Download as pdf or txt
Download as pdf or txt
You are on page 1of 70

PART XI CLINICAL CASES

These brief clinical case vignettes are typical presentations of common infectious diseases. Learning the most likely causative
organisms of these classic cases will help you answer the USMLE questions and improve your diagnostic skills. These cases are
presented in random order similar to the way they are on the USMLE. The important features of the case are written in
boldface.

CASE 1
A 22-year-old woman has a severe sore throat. Findings on physical examination include an inflamed throat, swollen cervical
lymph nodes, and an enlarged spleen. Her heterophile agglutinin test (Monospot test) is positive.
Diagnosis: Infectious mononucleosis caused by Epstein–Barr virus. Other viruses and bacteria, especially Streptococcus pyo-
genes, can cause pharyngitis and cervical lymphadenopathy, but an enlarged spleen and a positive Monospot test make infec-
tious mononucleosis the most likely diagnosis. See page 296 for additional information.

CASE 2
A 5-year-old boy with diabetic ketoacidosis has ptosis of his right eyelid, periorbital swelling, and a black, necrotic skin lesion
under his eye. Biopsy of the skin lesion shows nonseptate hyphae with wide-angle branching.
Diagnosis: Mucormycosis caused by Mucor or Rhizopus species. Diabetic ketoacidosis and renal acidosis predispose to mucor-
mycosis. Fungal spores are inhaled into the sinuses, resulting in lesions on the face. See page 419 for additional information.

CASE 3
A 40-year-old man complains of watery, foul-smelling diarrhea and flatulence for the past 2 weeks. He drank untreated water
on a camping trip about a month ago. See pear-shaped flagellated trophozoites in stool.
Diagnosis: Giardiasis caused by Giardia lamblia. Of the protozoa that are common causes of diarrhea, Giardia and Cryptospo-
ridium cause watery diarrhea, whereas Entamoeba causes bloody diarrhea. See page 428 for additional information on Giardia,
page 429 for additional information on Cryptosporidium, and page 424 for additional information on Entamoeba.

CASE 4
A 35-year-old man who is human immunodeficiency virus (HIV) antibody positive has had a persistent headache and a low-
grade fever (temperature, 100°F) for the past 2 weeks. See budding yeasts with a wide capsule in India ink preparation of
spinal fluid.
Diagnosis: Meningitis caused by Cryptococcus neoformans. The latex agglutination test, which detects the capsular polysac-
charide antigen of Cryptococcus in the spinal fluid, is a more sensitive and specific test than is the test with India ink. See
page xx for additional information. If acid-fast rods are seen in spinal fluid, think Mycobacterium tuberculosis. See page 417
for additional information.

691

mebooksfree.com
692 PART XI Clinical Cases

CASE 5
A 12-year-old boy has a painful arm that he thought he had injured while pitching in a Little League baseball game. The pain
has gotten worse over a 2-week period, and he now has a temperature of 100°F. X-ray of the humerus reveals raised periosteum.
Aspirate of lesion reveals gram-positive cocci in clusters.
Diagnosis: Osteomyelitis caused by Staphylococcus aureus. This organism is the most common cause of osteomyelitis in
children. Osteomyelitis in prosthetic joints is often caused by Staphylococcus epidermidis. See page 109 for additional informa-
tion on staphylococci.

CASE 6
A 50-year-old woman receiving chemotherapy via a subclavian catheter for acute leukemia has the sudden onset of blindness in
her right eye. Her total white blood cell (WBC) count is 120/μL. Blood cultures grew budding yeasts that formed germ tubes.
Diagnosis: Endophthalmitis (infection inside the eye) caused by Candida albicans. A catheter-related infection gave rise to an
embolus containing the organism, which traveled through the bloodstream to reach the eye. C. albicans is a member of the
normal flora of the skin and enters through a break in the skin at the catheter site. See page 414 for additional information.
If the blood culture grew colonies of gram-positive cocci in clusters that were coagulase-negative, think S. epidermidis, another
member of the skin flora that is also a common cause of catheter-associated infections. See page 115 for additional information.

CASE 7
A 60-year-old man has had a nonproductive cough and fever (temperature, 101°F) for 1 week. He received a kidney transplant
6 weeks ago and has had one episode of rejection that required increased prednisone. There was no response to erythromycin,
indicating that Legionella and Mycoplasma are unlikely causes. See owl’s-eye inclusion bodies within the nucleus of infected
cells in bronchoalveolar lavage fluid.
Diagnosis: Cytomegalovirus (CMV) pneumonia. These intranuclear inclusions are typical findings in CMV infections. Immu-
nosuppression predisposes to disseminated CMV infections. See page 295 for additional information.

CASE 8
A 45-year-old woman complains that her right arm has become increasingly weak during the past few days. This morning, she
had a generalized seizure. She recently finished a course of cancer chemotherapy. Magnetic resonance imaging (MRI) of the
brain reveals a lesion resembling an abscess. Brain biopsy shows gram-positive rods in long filaments. Organism is weakly
acid-fast.
Diagnosis: Brain abscess caused by Nocardia asteroides. Nocardia asteroides initially infects the lung, where it may or may not
cause symptoms in immunocompetent people. Dissemination to the brain is common in immunocompromised patients. See
page 195 for additional information.

CASE 9
A 20-year-old man has a severe headache and vomiting that began yesterday. He is now confused. On examination, his tem-
perature is 39°C and his neck is stiff. Spinal fluid reveals no bacteria on Gram stain, 25 lymphs, normal protein, and normal
glucose. Culture of the spinal fluid on blood agar shows no bacterial colonies.
Diagnosis: Viral meningitis, which is most often caused by Coxsackie virus. Can isolate the virus from spinal fluid. See
page 336 for additional information.

CASE 10
A 60-year-old man with a history of tuberculosis now has a cough productive of bloody sputum. Chest X-ray reveals a round
opaque mass within a cavity in his left upper lobe. Culture of the sputum grew an organism with septate hyphae that had
straight, parallel walls. The hyphae exhibited low-angle branching.
Diagnosis: “Fungus ball” caused by Aspergillus fumigatus. Fungal spores are inhaled into the lung, where they grow within a
preexisting cavity caused by infection with M. tuberculosis. See page 418 for additional information.

mebooksfree.com
PART XI Clinical Cases 693

CASE 11
A 3-month-old girl has watery, nonbloody diarrhea. Stool culture reveals only normal enteric flora.
Diagnosis: Think rotavirus, the most common cause of diarrhea in infants. The enzyme-linked immunosorbent assay
(ELISA) test for rotavirus antigen in the stool is positive, which confirms the diagnosis. See page 339 for additional
information.

CASE 12
A 30-year-old woman has a painless ulcer on her tongue. She is HIV antibody positive and has a CD4 count of 25. Her serum
is nonreactive in the VDRL test. Biopsy of the lesion revealed yeasts within macrophages.
Diagnosis: Disseminated histoplasmosis caused by Histoplasma capsulatum. Patients with a low CD4 count have severely
reduced cell-mediated immunity, which predisposes to disseminated disease caused by this dimorphic fungus. A negative
VDRL test indicates the ulcer was not caused by Treponema pallidum. See page 409 for additional information on
Histoplasma.

CASE 13
A 20-year-old man has a swollen, red, hot, tender ankle, accompanied by a temperature of 100°F for the past 2 days. There is
no history of trauma. See gram-negative diplococci in joint fluid aspirate. Organism is oxidase-positive.
Diagnosis: Arthritis caused by Neisseria gonorrhoeae, the most common cause of infectious arthritis in sexually active
adults. Sugar fermentation tests were used to identify the organism as N. gonorrhoeae. See page 132 for additional
information.

CASE 14
A 40-year-old woman has blurred vision and slurred speech. She is afebrile. She is famous in her neighborhood for her home-
canned vegetables and fruits.
Diagnosis: Botulism caused by Clostridium botulinum. Botulinum toxin causes a descending paralysis that starts with the
cranial nerves, typically appearing initially as diplopia. The toxin is a protease that cleaves the proteins involved in the release
of acetylcholine at the neuromuscular junction. Treat with antiserum immediately. Confirm diagnosis with mouse protec-
tion test or ELISA test using a sample of food suspected of containing the toxin. See page 139 for additional information.
Wound botulism occurs in heroin users (e.g., users of black tar heroin), especially in those who “skin pop.” Bacterial spores in
the heroin germinate in the anaerobic conditions in necrotic skin tissue.

CASE 15
A neonate was born with a small head (microcephaly), jaundice, and hepatosplenomegaly. Urine contained multinucleated
giant cells with intranuclear inclusions.
Diagnosis: Cytomegalovirus infection acquired in utero. Cytomegalovirus is the leading cause of congenital abnormalities.
For fetal infection to occur, the mother must be infected for the first time during pregnancy. She therefore would have no
preexisting antibodies to neutralize the virus prior to its infecting the placenta and the fetus. See page 295 for additional
information.

CASE 16
A 14-year-old girl has a rapidly spreading, painful, erythematous rash on her leg. The rash is warm and tender, and her tem-
perature is 38°C. Gram-positive cocci in chains were seen in an aspirate from the lesion. Culture of the aspirate on blood agar
grew colonies surrounded by clear (beta) hemolysis. Growth of the organism was inhibited by bacitracin.
Diagnosis: Cellulitis caused by S. pyogenes. The rapid spread of cellulitis caused by S. pyogenes is due to hyaluronidase (spread-
ing factor) that degrades hyaluronic acid in subcutaneous tissue. Acute glomerulonephritis (AGN) can follow skin infections
caused by S. pyogenes. AGN is an immunologic disease caused by antigen–antibody complexes. See page 120 for additional
information.

mebooksfree.com
694 PART XI Clinical Cases

CASE 17
A 4-year-old boy wakes up at night because his anal area is itching. See worm eggs in “Scotch tape” preparation.
Diagnosis: Pinworm infection (enterobiasis) caused by Enterobius vermicularis. Pinworm infection is the most common hel-
minth disease in the United States. See page 470 for additional information.

CASE 18
A 25-year-old woman has a painful, inflamed swollen hand. She was bitten by a cat about 8 hours ago. See small gram-negative
rods in the exudate from lesion.
Diagnosis: Cellulitis caused by Pasteurella multocida. Organism is normal flora in cat’s mouth. See page 180 for additional
information.

CASE 19
A 7-year-old girl has bloody diarrhea and fever (temperature, 38°C), but no nausea or vomiting. Only lactose-fermenting colo-
nies are seen on EMB agar.
Diagnosis: Think either Campylobacter jejuni or enterohemorrhagic strains of Escherichia coli (E. coli O157:H7). If Campylo-
bacter is the cause, see colonies on Campylobacter agar containing curved gram-negative rods, and the colonies on EMB agar
are likely to be nonpathogenic E. coli. If E. coli O157:H7 is the cause, the organism in the lactose-fermenting colonies on EMB
agar is unable to ferment sorbitol. The absence of non–lactose-fermenting colonies indicates that Shigella and Salmonella are not
the cause. See page 161 for additional information on Campylobacter and page 154 for additional information on E. coli O157:H7.

CASE 20
A 15-year-old girl has had a nonproductive cough and temperature of 100°F for the past 5 days. The symptoms came on gradu-
ally. Lung examination shows few scattered rales. Chest X-ray shows patchy infiltrate in left lower lobe but no consolidation.
Cold agglutinin test is positive.
Diagnosis: Atypical pneumonia caused by Mycoplasma pneumoniae. This organism is the most common cause of atypical
pneumonia in teenagers and young adults. In the cold agglutinin test, antibodies in the patient’s serum agglutinate human red
blood cells in the cold (4°C). These antibodies do not react with Mycoplasma. If sputum is available, a PCR test can confirm
Mycoplasma infection. See page 197 for additional information.

CASE 21
A 45-year-old man sustained a skull fracture in an automobile accident. The following day, he noted clear fluid dripping from
his nose, but he did not notify the hospital personnel. The following day, he spiked a fever to 39°C and complained of a severe
headache. Nuchal rigidity was found on physical examination. Spinal fluid analysis revealed a WBC count of 5200/μL, 90% of
which were neutrophils. Gram stain showed gram-positive diplococci.
Diagnosis: Meningitis caused by Streptococcus pneumoniae. Patients with a fracture of the cribriform plate who leak spinal
fluid into the nose are predisposed to meningitis by this organism. Pneumococci can colonize the nasal mucosa and enter the
subarachnoid space through the fractured cribriform plate. See page 123 for additional information.

CASE 22
A 7-year-old girl was well until about 3 weeks ago, when she began complaining of being “tired all the time.” On examination,
her temperature is 38°C and there is tenderness below the right knee. Hemoglobin: 10.2; WBC: 9600 with increased neutro-
phils. A sickle cell prep shows a moderate sickling tendency. Gram-negative rods grew in the blood culture.
Diagnosis: Osteomyelitis caused by Salmonella species. Sickle cell anemia predisposes to osteomyelitis caused by Salmo-
nella species. The abnormally shaped sickle cells are trapped in the small capillaries of the bone and cause microinfarcts. These
microinfarcts enhance the likelihood of infection by Salmonella. See page 155 for additional information.

CASE 23
A 3-month-old boy has a persistent cough and severe wheezing for the past 2 days. On physical examination, his temperature
is 39°C and coarse rhonchi are heard bilaterally. Chest X-ray shows interstitial infiltrates bilaterally. Diagnosis was made by
ELISA that detected viral antigen in nasal washings.

mebooksfree.com
PART XI Clinical Cases 695

Diagnosis: Think pneumonia caused by respiratory syncytial virus (RSV), the most common cause of pneumonia and bron-
chiolitis in infants. RSV causes giant cells (syncytia) that can be seen in respiratory secretions and in cell culture. See
page 321 for additional information.

CASE 24
A 34-year-old man was in his usual state of health until last night, when he felt feverish, had a shaking chill, and became short
of breath at rest. Temperature 39°C, blood pressure 110/60, pulse 104, respirations 18. Scattered rales were heard in both bases.
A new murmur consistent with tricuspid insufficiency was heard. Needle tracks were seen on both forearms. Gram-positive
cocci in clusters grew in blood culture.
Diagnosis: Acute endocarditis caused by S. aureus. This organism is the most common cause of acute endocarditis in intrave-
nous drug users. The valves on the right side of the heart are often involved. See page 109 for additional information.

CASE 25
A 2-week-old infant was well on discharge from the hospital 10 days ago and remained so until last night, when he appeared
drowsy and flushed. His skin felt hot to the touch. On physical examination, the infant was very difficult to arouse, but there
were no other positive findings. His temperature was 40°C. Blood culture grew gram-positive cocci in chains. A narrow zone
of clear (beta) hemolysis was seen around the colonies. Hippurate hydrolysis test was positive.
Diagnosis: Neonatal sepsis caused by Streptococcus agalactiae (group B streptococci). Group B streptococci are the most com-
mon cause of neonatal sepsis. Think E. coli if gram-negative rods are seen or Listeria monocytogenes if gram-positive rods are
seen. See page 121 for additional information on group B streptococci, page 153 for additional information on E. coli, and
page 144 for additional information on L. monocytogenes.

CASE 26
A 70-year-old woman had a hip replacement because of severe degenerative joint disease. She did well until a year later, when
a fall resulted in a fracture of the femur and the prosthesis had to be replaced. Three weeks later, bloody fluid began draining
from the wound site. The patient was afebrile, and the physical examination was otherwise unremarkable. Two days later,
because of increasing drainage, the wound was debrided and pus was obtained. Gram stain of the pus was negative, but an
acid-fast stain revealed red rods.
Diagnosis: Prosthetic joint infection caused by Mycobacterium fortuitum-chelonei complex. Think S. epidermidis if gram-
positive cocci in clusters are seen. See page 190 for additional information on M. fortuitum-chelonei complex and page 115 for
additional information on S. epidermidis.

CASE 27
An 80-year-old man complains of a painful rash on his left forehead. The rash is vesicular and only on that side. He is being
treated with chemotherapy for leukemia. Smear of material from the base of the vesicle reveals multinucleated giant cells with
intranuclear inclusions.
Diagnosis: Herpes zoster (shingles) caused by varicella-zoster virus. The rash of zoster follows the dermatome of the neuron
that was latently infected. Herpes simplex virus type 1 can cause a similar picture. These viruses can be distinguished using
fluorescent antibody assay. See page 293 for additional information.

CASE 28
A 55-year-old woman has an inflamed ulcer on her right hand and several tender nodules on the inner aspect of her right arm.
She is an avid gardener and especially enjoys pruning her roses. Biopsy of the lesion reveals budding yeasts.
Diagnosis: Sporotrichosis caused by Sporothrix schenckii. The organism is a mold in the soil and a yeast in the body (i.e., it is
dimorphic). Infection occurs when spores produced by the mold form are introduced into the skin by a penetrating injury.
See page 405 for additional information.

CASE 29
A 15-year-old boy sustained a broken tooth in a fist fight several weeks ago. He now has an inflamed area on the skin over the
broken tooth, in the center of which is a draining sinus tract. Gram stain of the drainage fluid reveals filamentous gram-
positive rods.

mebooksfree.com
696 PART XI Clinical Cases

Diagnosis: Actinomycosis caused by Actinomyces israelii. See “sulfur granules” in the sinus tract. These granules are particles
composed of interwoven filaments of bacteria. See page 194 for additional information.

CASE 30
A 24-year-old woman experienced the sudden onset of high fever, myalgias, vomiting, and diarrhea. Her vital signs were as
follows: temperature 40°C, blood pressure 70/30, pulse 140, respirations 30. A sunburn-like rash appeared over most of her
body. Blood cultures and stool cultures are negative. She is recovering from a surgical procedure on her maxillary sinus, and
the bleeding was being staunched with nasal tampons. Gram-positive cocci in clusters were seen in blood adherent to the nasal
tampon.
Diagnosis: Toxic shock syndrome caused by S. aureus. Toxic shock syndrome toxin is a superantigen that stimulates the
release of large amounts of cytokines from many helper T cells. See page 114 for additional information.

CASE 31
An 8-year-old girl has a pruritic rash on her chest. Lesions are round or oval with an inflamed border and central clearing. The
lesions contain both papules and vesicles. See hyphae in KOH prep of scrapings from the lesion.
Diagnosis: Tinea corporis (ringworm) caused by one of the dermatophytes, especially species of Microsporum, Trichophyton,
or Epidermophyton. Dermatophytes use keratin as a nutrient source, so lesions are limited to the skin. See page 404 for addi-
tional information.

CASE 32
A 25-year-old woman has a papular rash on her trunk, arms, and palms. She says the rash does not itch. Vaginal examination
reveals two flat, moist, slightly raised lesions on the labia. Material from a labial lesion examined in a dark field microscope
revealed spirochetes.
Diagnosis: Secondary syphilis caused by T. pallidum. The rash on the palms coupled with the vaginal lesions (condylomata
lata) is compatible with secondary syphilis. Serologic tests, such as the nonspecific test (VDRL) and the specific test (FTA-
ABS), were positive. See page 201 for additional information.

CASE 33
A 5-year-old girl complains of an earache for the past 2 days. On examination, she has a temperature of 39°C, the right external
canal contained dried blood, the drum was perforated, and a small amount of purulent fluid was seen. Gram stain of the pus
revealed gram-positive diplococci. Colonies formed green (alpha) hemolysis on blood agar. Growth was inhibited by
optochin.
Diagnosis: Otitis media caused by S. pneumoniae. Think Haemophilus influenzae if small gram-negative rods are seen. These
organisms colonize the oropharynx and enter the middle ear via the eustachian tube. See page 123 for additional information
on S. pneumoniae and page 171 for additional information on H. influenzae.

CASE 34
A 25-year-old woman was well until the sudden onset of high fever (temperature, 40°C) accompanied by several purple skin
lesions (ecchymoses, purpura). The lesions are scattered over the body, are irregularly shaped, and are not raised. Her blood
pressure is 60/10, and her pulse rate is 140. Blood culture grew gram-negative diplococci.
Diagnosis: Meningococcemia caused by Neisseria meningitidis. The endotoxin (lipopolysaccharide, or LPS) of the organism
triggers release of interleukin-1, tumor necrosis factor, and nitric oxide from macrophages. These cause the high fever and low
blood pressure. The purpuric lesions are a manifestation of disseminated intravascular coagulation (DIC). Endotoxin acti-
vates the coagulation cascade, causing DIC. Lipid A is the toxic part of LPS. See page 130 for additional information.

CASE 35
A 40-year-old woman was well until 2 days ago, when she experienced the sudden onset of fever, shaking chills, and profuse
sweating. Today, she also complains of headache and abdominal pain but no nausea, vomiting, or diarrhea. She does not have
a stiff neck, rash, or altered mental status. Travel history reveals she returned from an extended trip to several countries in
central Africa 1 week ago. Blood smear reveals ring-shaped trophozoites within red blood cells.

mebooksfree.com
PART XI Clinical Cases 697

Diagnosis: Malaria caused by Plasmodium species. If banana-shaped gametocytes seen in the blood smear, think Plasmodium
falciparum. Plasmodium falciparum is the species that causes the life-threatening complications of malaria, such as cerebral
malaria. The fever and chills experienced by the patient coincide with the release of merozoites from infected red blood cells
and occur in either a tertian or quartan pattern. See page 433 for additional information.

CASE 36
A 35-year-old man is seen in the emergency room (ER) complaining of severe headache and vomiting that began last night.
His temperature is 40°C. While in the ER, he is increasingly combative and has a grand mal seizure. He is “foaming at the
mouth” and cannot drink any liquids. Analysis of his spinal fluid reveals no abnormality, and no organisms are seen in the
Gram stain. Two days later, despite supportive measures, he dies. Pathologic examination of the brain reveals eosinophilic
inclusion bodies in the cytoplasm of neurons.
Diagnosis: Rabies (an encephalitis) caused by rabies virus. The inclusions are Negri bodies. Diagnosis can be confirmed by
using fluorescent antibody assays. The patient was a farm worker who was bitten by a bat about a month prior to the onset of
symptoms. Note the long incubation period, which can be as long as 6 months. People bitten by a bat (or any wild animal)
should receive rabies immunization consisting of the inactivated vaccine plus rabies immune globulins (passive–active immu-
nization). See page 326 for additional information.

CASE 37
A 70-year-old man was admitted to the hospital after suffering extensive third-degree burns. Three days later, he spiked a fever,
and there was pus on the dressing that had a blue–green color. Gram stain of the pus revealed gram-negative rods.
Diagnosis: Wound (burn) infection caused by Pseudomonas aeruginosa. The blue-green color is caused by pyocyanin, a pig-
ment produced by the organism. See page 165 for additional information.

CASE 38
A 65-year-old woman reports that she has had several episodes of confusion and memory loss during the past few weeks. On
examination, she is afebrile but has a staggering gait and myoclonus can be elicited. Over the next several months, her condition
deteriorates and death ensues. On autopsy, microscopic examination of the brain reveals many vacuoles but no viral inclusion
bodies.
Diagnosis: Creutzfeldt-Jakob disease (CJD) caused by prions. CJD is a spongiform encephalopathy. The vacuoles give the
brain a sponge-like appearance. See page 374 for additional information.

CASE 39
A 20-year-old man complains of several episodes of blood in his urine. He has no dysuria or urethral discharge. He is not sexu-
ally active. He is a college student but was born and raised in Egypt. Physical examination reveals no penile lesions. Urinalysis
shows many red cells, no white cells, and several large eggs with terminal spines.
Diagnosis: Schistosomiasis caused by Schistosoma haematobium. Schistosome eggs in venules of the bladder damage the blad-
der epithelium and cause bleeding. The eggs are excreted in the urine. See page 462 for additional information.

CASE 40
A 35-year-old man complains of night sweats, chills, and fatigue at varying intervals during the past 2 months. These episodes
began while he was traveling in Latin America. When questioned, he says that cheeses, especially the unpasteurized varieties,
are some of his favorite foods. On examination, his temperature is 39°C, and his liver and spleen are palpable. His hematocrit
is 30%, and his WBC count is 5000. Blood culture grew small gram-negative rods.
Diagnosis: Brucellosis caused by Brucella species. Domestic animals such as cows and goats are the main reservoir for
Brucella, and it is often transmitted in unpasteurized dairy products. This patient could also have typhoid fever caused by
Salmonella typhi, but S. typhi is only a human pathogen (i.e., there is no animal reservoir). See page 177 for additional infor-
mation on Brucella species and page 155 for additional information on S. typhi.

CASE 41
A 6-year-old girl has a rash on her face that appeared yesterday. The rash is erythematous and located over the malar emi-
nences bilaterally. The rash is macular; there are no papules, vesicles, or pustules. A few days prior to the appearance of the
rash, she had a runny nose and anorexia.

mebooksfree.com
698 PART XI Clinical Cases

Diagnosis: Slapped cheek syndrome caused by parvovirus B19. This virus also causes aplastic anemia because it prefer-
entially infects and kills erythroblasts. It also infects the fetus, causing hydrops fetalis, and causes an immune complex–
mediated arthritis, especially in adult women. See page 308 for additional information.

CASE 42
A 20-year-old man fell off his motorcycle and suffered a compound fracture of the femur. The fracture was surgically reduced
and the wound debrided. Forty-eight hours later, he spiked a fever (temperature, 40°C), and the wound area became necrotic.
Crepitus was felt, and a foul-smelling odor was perceived originating from the wound. Marked anemia and a WBC count of
22,800 were found. Gram stain of the exudate showed large gram-positive rods. Colonies grew on blood agar incubated
anaerobically but not aerobically.
Diagnosis: Gas gangrene (myonecrosis) caused by Clostridium perfringens. The main virulence factor produced by this organ-
ism is an exotoxin that is a lecithinase. It causes necrosis of tissue and lysis of red blood cells (causing hemolytic anemia). The
spores of the organism are in the soil and enter at the wound site. A foul-smelling exudate is characteristic of infections caused
by anaerobic bacteria. See page 140 for additional information.

CASE 43
A 30-year-old woman complains of a burning feeling in her mouth and pain on swallowing. Sexual history reveals she is a
commercial sex worker and has had unprotected vaginal, oral, and anal intercourse with multiple partners. On examination,
whitish lesions are seen on the tongue, palate, and pharynx. No vesicles are seen. The test for HIV antibody is positive, and her
CD4 count is 65. Gram stain of material from the lesions reveals budding yeasts and pseudohyphae.
Diagnosis: Thrush caused by C. albicans. This organism forms pseudohyphae when it invades tissue. The absence of vesicles
indicates that her symptoms are not caused by herpes simplex virus type 2. See page 414 for additional information.

CASE 44
You’re a physician at a refugee camp in sub-Saharan Africa, when an outbreak of diarrhea occurs. Massive amounts of watery
stool, without blood, are produced by the patients. Curved gram-negative rods are seen in a Gram stain of the stool.
Diagnosis: Cholera caused by Vibrio cholerae. There are three genera of curved gram-negative rods: Vibrio, Campylobacter,
and Helicobacter. V. cholerae causes watery, nonbloody diarrhea, whereas C. jejuni typically causes bloody diarrhea.
Helicobacter pylori causes gastritis and peptic ulcer, not diarrhea. Enterotoxigenic E. coli causes watery diarrhea by producing
an exotoxin that has the same mode of action as does the exotoxin produced by V. cholerae. However, E. coli is a straight gram-
negative rod, not a curved one. If an outbreak of bloody diarrhea had occurred in the refugee camp, then Shigella dysenteriae
would be the most likely cause. See the following pages for additional information: Vibrio, page 159; Campylobacter, page 161;
Helicobacter, page 162; Escherichia, page 153; and Shigella, page 158.

CASE 45
A 40-year-old man with low-grade fever and night sweats for the past 4 weeks now has increasing fatigue and shortness of
breath. He says he has difficulty climbing the one flight of stairs to his apartment. Pertinent past history includes rheumatic
fever when he was 15 years old and the extraction of two wisdom teeth about 3 weeks before his symptoms began. No che-
moprophylaxis was given at the time of the extractions. There is no history of intravenous drug use. His temperature is
38.5°C, and a loud holosystolic murmur can be heard over the precordium. His spleen is palpable. He is anemic, and his
WBC count is 13,500. Blood cultures grow gram-positive cocci in chains that produce green (alpha) hemolysis on blood
agar. Growth is not inhibited by optochin.
Diagnosis: Subacute bacterial endocarditis caused by one of the viridans group streptococci, such as Streptococcus sanguinis.
The laboratory findings are also compatible with Enterococcus faecalis, but the history of dental surgery makes the viridans
group streptococci more likely to be the cause. Endocarditis caused by E. faecalis is associated with gastrointestinal or genito-
urinary tract surgery. See page 121 for additional information on both viridans group streptococci and E. faecalis.

CASE 46
A 60-year-old woman is asymptomatic but has a lung nodule seen on chest X-ray. Pertinent past history includes her cigarette
smoking (2 packs per day for 40 years) and her occupation as an archaeologist, digging primarily in Arizona and New Mexico.
Because of concern that the nodule may be malignant, it was surgically removed. Pathologic examination revealed large
(25 μm) round structures with thick walls and many round spores inside. No malignant cells were seen.

mebooksfree.com
PART XI Clinical Cases 699

Diagnosis: Coccidioidomycosis caused by Coccidioides immitis. These structures are spherules, which are pathognomonic for
this disease. The mold form of the organism is found in the soil of the southwestern United States, and the organism is acquired
by inhalation of arthrospores produced by the mold. The inhaled arthrospores form spherules in the lung. Coccidioides immitis
is dimorphic and forms spherules at 37°C. See page 407 for additional information.

CASE 47
A 20-year-old woman in her 30th week of pregnancy had an ultrasound examination that revealed a growth-retarded fetus
with a large head (indicating hydrocephalus) and calcifications within the brain. Umbilical blood was cultured, and crescent-
shaped trophozoites were grown.
Diagnosis: Toxoplasmosis caused by Toxoplasma gondii. Detection of IgM antibody in the Sabin-Feldman dye test can also be
used to make a diagnosis. The main reservoir is domestic cats. Domestic farm animals, such as cattle, acquire the organism by
accidentally eating cat feces. Pregnant women should not be exposed to cat litter or eat undercooked meat. See page 438 for
additional information.

CASE 48
A 10-day-old neonate has several vesicles on the scalp and around the eyes. The child is otherwise well, afebrile, and feeding
normally. A Giemsa-stained smear of material from the base of a vesicle revealed multinucleated giant cells with intranuclear
inclusions.
Diagnosis: Neonatal infection caused by herpes simplex virus type 2. Infection is acquired during passage through the birth
canal. Life-threatening encephalitis and disseminated infection of the neonate also occur. See page 292 for additional
information.

CASE 49
A 40-year-old woman has just had a grand mal seizure. There is a history of headaches for the past week and one episode of
vertigo but no previous seizures. She is afebrile. She is a native of Honduras but has lived in the United States for the past 5
years. MRI reveals a mass in the parietal lobe. Surgical removal of the mass reveals a larva within a cystlike sac.
Diagnosis: Cysticercosis caused by the larva of Taenia solium. Infection is acquired by ingesting the tapeworm eggs, not by
ingesting undercooked pork. This clinical picture can also be caused by a brain abscess, a granuloma such as a tuberculoma,
or a brain tumor. See page 454 for additional information.

CASE 50
A 1-week-old neonate has a yellowish exudate in the corners of both eyes. The child is otherwise well, afebrile, and feeding
normally. Gram stain of the exudate reveals no gram-negative diplococci. A Giemsa-stained smear of the exudate reveals a
large cytoplasmic inclusion.
Diagnosis: Conjunctivitis caused by Chlamydia trachomatis. Confirm the diagnosis with direct fluorescent antibody test.
Infection is acquired during passage through the birth canal. The inclusion contains large numbers of the intracellular repli-
cating forms called reticulate bodies. See page 210 for additional information.

mebooksfree.com
mebooksfree.com
PART XII PEARLS FOR THE USMLE

Many questions on the USMLE can be answered by Table XII–4. Environmental Sources of Medically
knowing the meaning of the epidemiologic information Important Organisms
provided in the case description. In order to do this, the Table XII–5. Main Geographical Location of Medically
student should know the reservoir of the organism, its Important Organisms
mode of transmission, and the meaning of factors such as Table XII–6. Occupations and Avocations That Increase
travel, occupation, and exposure to pets, farm animals, or Exposure to Medically Important Organisms
wild animals. Knowledge of the microbes that typically Table XII–7. Hospital-Related Events That Predispose to
cause disease in individuals with specific immunodefi- Infection by Medically Important Organisms
ciencies will also be helpful. Table XII–8. Organisms That Commonly Cause Disease
In addition to being useful for the USMLE, this infor- in Patients with Immunodeficiencies or Reduced Host
mation will prove valuable to make the diagnosis of infec- Defenses
tious diseases on the wards and in your clinical practice. Table XII–9. Important Factors That Predispose to
The “Pearls” are presented in tables entitled: Infections by Specific Organisms
Table XII–1. Farm Animals and Household Pets as Res- Table XII–10. Maternal Infections That Pose Significant
ervoirs of Medically Important Organisms Risk to the Fetus or Neonate
Table XII–2. Wild Animals as Reservoirs of Medically Table XII–11. Important Skin Lesions Caused by
Important Organisms Microorganisms
Table XII–3. Insects as Vectors of Medically Important
Organisms

701

mebooksfree.com
702 PART XII Pearls for the USMLE

TABLE XII–1 Farm Animals and Household Pets as Reservoirs of Medically Important Organisms
Animal Mode of Transmission Important Organisms Disease
1
Cattle/cows 1. Ingestion of meat 1. Escherichia coli O157 Enterocolitis and hemolytic–uremic
syndrome
    2. Salmonella enterica Enterocolitis
    3. Prions Variant Creutzfeldt-Jakob disease
    4. Taenia saginata Taeniasis (intestinal tapeworm)
    5. Toxoplasma gondii Toxoplasmosis
2
  2. Ingestion of milk products 1. Listeria monocytogenes Neonatal sepsis
    2. Brucella species Brucellosis
    3. Mycobacterium bovis Intestinal tuberculosis
  3. Contact with animal hides Bacillus anthracis Anthrax
Sheep Inhalation of amniotic fluid Coxiella burnetii Q fever
Goats Ingestion of milk products2 Brucella species Brucellosis
1
Pigs Ingestion of meat 1. Taenia solium Taeniasis (intestinal tapeworm)3
    2. Trichinella spiralis Trichinosis
1
Poultry (chickens; turkeys) Ingestion of meat or eggs 1. S. enterica Enterocolitis
    2. Campylobacter jejuni Enterocolitis
Dogs 1. Ingestion of dog feces 1. Echinococcus granulosus Echinococcosis
    2. Toxocara canis Visceral larva migrans
  2. Ingestion of dog urine Leptospira interrogans Leptospirosis
  3. Dog bite 1. Rabies virus Rabies
    2. Capnocytophaga canimorsus Sepsis
  4. Direct contact Microsporum canis Tinea corporis
Cats 1. Ingestion of cat feces T. gondii Toxoplasmosis
  2. Cat bite/scratch 1. Pasteurella multocida Cellulitis
    2. Bartonella henselae Cat-scratch disease; bacillary
angiomatosis
    3. Rabies virus Rabies
1
Raw or undercooked.
2
Unpasteurized.
3
Ingestion of eggs in human feces, not ingestion of pork, results in cysticercosis.

TABLE XII–2 Wild Animals as Reservoirs of Medically Important Organisms


Animal Mode of Transmission Important Organisms Disease

Rats 1. Flea bite Yersinia pestis Plague


  2. Ingestion of urine Leptospira interrogans Leptospirosis
Mice 1. Tick bite Borrelia burgdorferi Lyme disease
  2. Inhale aerosol of droppings Hantavirus Hantavirus
      Pulmonary syndrome
Bats, skunks, raccoons, and foxes Bite Rabies virus Rabies
Rabbits Contact Francisella tularensis Tularemia
Civet cats, bats Inhale aerosol Coronavirus—SARS Pneumonia
Monkeys Mosquito bite Yellow fever virus Yellow fever

(Continued)

mebooksfree.com
PART XII Pearls for the USMLE 703

TABLE XII–2 Wild Animals as Reservoirs of Medically Important Organisms (Continued )


Animal Mode of Transmission Important Organisms Disease

Birds      
1. Psittacine birds (e.g., parrots) Inhale aerosol Chlamydia psittaci Psittacosis
2. Chickens Inhale aerosol Influenza virus Influenza
3. Pigeons Inhale aerosol Cryptococcus neoformans Meningitis, pneumonia
4. Starlings Inhale aerosol Histoplasma capsulatum Histoplasmosis
5. Sparrows Mosquito bite Encephalitis viruses (e.g., West Nile Encephalitis
virus)
Snakes, turtles Fecal-oral Salmonella enterica Enterocolitis
Beaver Fecal-oral Giardia lamblia Giardiasis
1
Fish Ingestion of fish Anisakis simplex Anisakiasis
    Diphyllobothrium latum Diphyllobothriasis
SARS = severe acute respiratory syndrome.
1
Raw or undercooked.

TABLE XII–3 Insects as Vectors of Medically Important Organisms


Insects Important Organisms Reservoir Disease

Ticks      
1. Ixodes (deer tick) 1. Borrelia burgdorferi Mice Lyme disease
  2. Babesia microti Mice Babesiosis
2. Dermacentor (dog tick) 1. Rickettsia rickettsii Rodents, dogs Rocky Mountain spotted fever
  2. Ehrlichia chaffeensis Dogs Ehrlichiosis
  3. Anaplasma phagocytophilum Rodents, dogs Anaplasmosis
Lice Rickettsia prowazekii Humans Typhus
Mosquitoes      
1. Anopheles Plasmodium falciparum, P. vivax, P. ovale, P. malariae Humans Malaria
2. Aedes Yellow fever virus Humans and monkeys Yellow fever
3. Aedes Dengue virus Humans Dengue
4. Culex Encephalitis viruses, such as West Nile virus Birds Encephalitis
5. Anopheles and Culex Wuchereria bancrofti Humans Filariasis, especially elephantiasis
Fleas      
Rat flea Yersinia pestis Rats Plague
Flies      
1. Sandfly Leishmania donovani Various animals Leishmaniasis
2. Tse-tse fly Trypanosoma brucei Humans and various Sleeping sickness
animals
3. Blackfly Onchocerca volvulus Humans Onchocerciasis
Bugs      
Reduviid bug Trypanosoma cruzi Various animals Chagas’ disease

mebooksfree.com
704 PART XII Pearls for the USMLE

TABLE XII–4 Environmental Sources of Medically Important Organisms


Environmental Source Important Organisms Mode of Transmission Disease

Water 1. Legionella pneumophila Inhale aerosol Pneumonia


  2. Pseudomonas aeruginosa Inhale aerosol or direct contact Pneumonia, burn, and wound
infections
  3. Mycobacterium marinum Skin abrasion Swimming pool granuloma
  4. Vibrio vulnificus Skin abrasion Cellulitis
  5. Schistosoma mansoni, S. hematobium Cercariae enter skin Schistosomiasis
  6. Naegleria fowleri Ameba enter nose while swimming Meningoencephalitis
Soil 1. Clostridium tetani Spores in soil enter wound Tetanus
  2. Clostridium botulinum Spores in soil contaminate food that Botulism
is improperly canned
  3. Clostridium perfringens Spores in soil enter wound Gas gangrene
  4. Bacillus anthracis Spores in soil enter wound Anthrax
  5. Atypical mycobacteria (e.g., Inhale aerosol Tuberculosis-like disease
Mycobacterium avium-intracellulare)
  6. Nocardia asteroides Inhale aerosol Nocardiosis
  7. Cryptococcus neoformans Inhale yeast in aerosol of soil contam- Meningitis, pneumonia
inated by pigeon guano
  8. Histoplasma capsulatum Inhale spores in aerosol of soil con- Histoplasmosis
taminated by starling guano
  9. Coccidioides immitis Inhale spores in aerosol of soil dust Coccidioidomycosis
  10. Sporothrix schenckii Spores in soil enter wound Sporotrichosis
  11. Ancylostoma duodenale and Filariform larvae enter skin Hookworm, especially anemia
Necator americanus
  12. Strongyloides stercoralis Filariform larvae enter skin Strongyloidiasis
  13. Ancylostoma caninum Filariform larvae enter skin Cutaneous larva migrans

TABLE XII–5 Main Geographical Location of Medically Important Organisms


Main Geographical Location Important Organism Disease

Within the United States    


1. South central states (e.g., North Carolina and Virginia) Rickettsia rickettsii Rocky Mountain spotted fever
2. Northeastern states (e.g., Connecticut, New York, and New Borrelia burgdorferi Lyme disease
Jersey)
3. Midwestern states in the Ohio and Mississippi River val- Histoplasma capsulatum Histoplasmosis
leys (e.g., Missouri and Illinois)
4. Southwestern states (e.g., California and Arizona) Coccidioides immitis Coccidioidomycosis
Outside the United States    
1. Tropical areas of Africa, Asia, and South America Plasmodium species Malaria
2. Central America Trypanosoma cruzi Chagas’ disease
3. Caribbean Islands and Africa Dengue virus Dengue fever
4. West Africa Ebola virus Ebola hemorrhagic fever
5. Tropical areas of Africa and South America Yellow fever virus Yellow fever
6. Sub-Saharan Africa Neisseria meningitidis Meningococcal meningitis
7. Central Africa Trypanosoma brucei African sleeping sickness
8. Middle East, Africa, and India Leishmania donovani Visceral leishmaniasis (kala-azar)
9. Middle East, Africa, and India Leishmania tropica Cutaneous leishmaniasis
10. Central and South America Leishmania brasiliensis Mucocutaneous Leishmaniasis

mebooksfree.com
PART XII Pearls for the USMLE 705

TABLE XII–6 Occupations and Avocations That Increase Exposure to Medically Important Organisms
Occupation/Avocation Predisposing Factor Important Organism Disease

Hiking/camping Tick exposure Borrelia burgdorferi Lyme disease


Rancher/farm worker Skin wound contaminated with soil Bacillus anthracis Anthrax
Sewer worker Exposure to rat urine Leptospira interrogans Leptospirosis
Cave explorer (spelunker) in bat- Exposure to aerosol of bat saliva Rabies virus Rabies
infested caves
Cave explorer (spelunker) or con- Exposure to aerosol of bat guano Histoplasma capsulatum Histoplasmosis
struction worker
Archaeologist or construction worker Exposure to soil dust containing Coccidioides immitis Coccidioidomycosis
digging in soil spores
Pigeon fancier Exposure to aerosol of bird guano Cryptococcus neoformans Cryptococcosis
Bear hunter in Alaska Ingestion of bear meat Trichinella spiralis Trichinosis
Aquarium personnel/swimming pool Abrasion of skin Mycobacterium marinum “Swimming pool granuloma”

TABLE XII–7 Hospital-Related Events That Predispose to Infection by Medically Important Organisms
Hospital-Related Event Important Organism Disease

Surgery Staphylococcus aureus Wound infection


Urinary catheter 1. Escherichia coli primarily, but also other enteric gram- Urinary tract infection
negative rods (e.g., Proteus, Serratia, and Pseudomonas)
  2. Enterococcus faecalis Urinary tract infection
Intravenous catheter Staphylococcus epidermidis, Candida albicans Catheter-related infection, bacteremia
Prosthetic device (e.g., hip or heart valve) 1. S. epidermidis Osteomyelitis or endocarditis
  2. Mycobacterium fortuitum-chelonei Osteomyelitis
Respiratory therapy Pseudomonas aeruginosa, Acinetobacter baumannii Pneumonia
Burn therapy P. aeruginosa Wound infection
Intracerebral electrodes Prion Creutzfeldt-Jakob disease
Needlestick 1. HBV, HCV Hepatitis B or C
  2. HIV AIDS
Premature nursery Respiratory syncytial virus Bronchiolitis or pneumonia
AIDS = acquired immunodeficiency syndrome; HBV = hepatitis B virus; HCV = hepatitis C virus; HIV = human immunodeficiency virus.

TABLE XII–8 Organisms That Commonly Cause Disease in Patients with Immunodeficiencies or Reduced Host
Defenses
Immunodeficiency or Reduced Host Defense Organisms

Reduced antibodies (e.g., agammaglobulinemia and IgA Encapsulated bacteria (e.g., Streptococcus pneumoniae, Haemophilus influenzae
deficiency) type b)
Reduced phagocytosis (e.g., chronic granulomatous disease, Staphylococcus aureus, Pseudomonas aeruginosa, Aspergillus fumigatus
cancer chemotherapy [neutropenia])
Reduced complement  
1. C3b S. pneumoniae, H. influenzae type b, S. aureus
2. C6,7,8,9 (membrane attack complex) Neisseria meningitidis

(Continued)

mebooksfree.com
706 PART XII Pearls for the USMLE

TABLE XII–8 Organisms That Commonly Cause Disease in Patients with Immunodeficiencies or Reduced Host
Defenses (Continued )
Immunodeficiency or Reduced Host Defense Organisms

Reduced cell-mediated immunity  


1. Thymic aplasia (DiGeorge’s syndrome) Candida albicans, Pneumocystis jiroveci
2. HIV infection (AIDS), corticosteroids Intracellular bacteria (e.g., Mycobacterium tuberculosis, MAI, Listeria, Salmonella)
  Opportunistic fungi (e.g., Candida, Cryptococcus)
  Herpesviruses (e.g., herpes simplex virus, varicella-zoster virus, cytomegalovirus)
  Protozoa (e.g., Toxoplasma, Cryptosporidium)
  Pneumocystis
Disrupted epithelial surface (e.g., burns) P. aeruginosa
Splenectomy S. pneumoniae, Babesia microti
Diabetes mellitus S. aureus, Mucor species, P. aeruginosa
AIDS = acquired immunodeficiency syndrome; HIV = human immunodeficiency virus; IgA = immunoglobulin A; MAI = Mycobacterium avium-intracellulare complex.

TABLE XII–9 Important Factors That Predispose to Infections by Specific Organisms


Predisposing Factor Organism Disease Pathogenetic Mechanism

Cystic fibrosis Pseudomonas aeruginosa Pneumonia Tenacious mucus traps bacteria in airways
Sickle cell anemia Salmonella enterica Osteomyelitis Abnormally shaped red cells block blood
vessels in bone and trap bacteria
  Streptococcus pneumoniae Sepsis Abnormally shaped red cells block blood
vessels in spleen causing infarction of
spleen
Intravenous drug use Staphylococcus aureus Right-sided endocarditis Skin flora enter venous blood at site of
needle
Antibiotic use Clostridium difficile Pseudomembranous colitis Antibiotics suppress enteric normal flora,
allowing C. difficile to grow
Aortic aneurysm S. enterica1 Vascular graft infection Uncertain
Tampon use (either vaginal or S. aureus Toxic shock syndrome Tampon blocks flow of blood, allowing
nasal tampon) S. aureus to grow and produce toxin
Dental surgery Viridans group streptococci Endocarditis These bacteria are normal flora in the
mouth and enter the blood at the site of
the surgical wound
Prosthetic heart valve Staphylococcus epidermidis Endocarditis Skin flora enter blood stream at site of
catheter or skin wound
Prosthetic joint S. epidermidis Osteomyelitis Skin flora enter blood stream at site of
catheter or skin wound
Motorcycle accident Clostridium perfringens Gas gangrene (myonecrosis) Spores in soil enter wound site
Contact lenses P. aeruginosa, Acanthamoeba Keratitis Abrasions caused by lenses provide entry
castellani site for organisms
1
Especially S. enterica serotype Choleraesuis and serotype Dublin.

mebooksfree.com
PART XII Pearls for the USMLE 707

TABLE XII–10 Maternal Infections That Pose Significant Risk to the Fetus or Neonate
Transplacental or Perina-
Microbe tal Transmission to Fetus Comment

A. Virus    
Cytomegalovirus Transplacental The leading cause of congenital abnormalities
Parvovirus B-19 Transplacental Important cause of congenital abnormalities, including hydrops fetalis
Rubella virus Transplacental Vaccine has greatly reduced the incidence of fetal infection
Human immunodeficiency virus Perinatal Most are perinatal but transplacental and via breast milk also occurs
Hepatitis B virus (HBV) Perinatal Neonatal HBV infection greatly increases the risk of chronic carrier state
Hepatitis C virus (HCV) Perinatal Neonatal HCV infection greatly increases the risk of chronic carrier state
Herpes simplex type 2 virus Perinatal Important cause of encephalitis
B. Bacteria    
Treponema pallidum Transplacental Causes congenital syphilis
Neisseria gonorrhoeae Perinatal Important cause of conjunctivitis (ophthalmia neonatorum)
Chlamydia trachomatis Perinatal Important cause of conjunctivitis and pneumonia
Streptococcus agalactiae (group B Perinatal Important cause of meningitis and sepsis
Streptococcus)
Escherichia coli Perinatal Important cause of meningitis and sepsis
Listeria monocytogenes Perinatal Important cause of meningitis and sepsis
C. Yeast    
Candida albicans Perinatal Causes thrush of the oropharynx
D. Protozoan    
Toxoplasma gondii Transplacental Important cause of congenital abnormalities, especially of eye and brain

TABLE XII–11 Important Skin Lesions Caused by Microorganisms


Name or Type of Lesion Causative Organism Description of Lesion Comment

A. Single or localized lesions      


Black eschar of anthrax Bacillus anthracis Crust over a necrotic ulcer Caused by lethal toxin of B. anthracis
Carbuncle Staphylococcus aureus Group of furuncles (see below), Poor personal hygiene predisposes
often on neck
Cellulitis Streptococcus pyogenes Red, hot, tender, rapidly spreading, Hyaluronidase is “spreading factor”
irregular shape
Chancre of primary syphilis Treponema pallidum Painless, moist, shallow ulcer Dark field microscopy shows motile
spirochetes
Cutaneous larva migrans Ancylostoma caninum Pruritic track, often on foot Larva of dog hookworm migrates in skin
Ecthyma gangrenosum Most often Pseudomonas Necrotic ulcer with black eschar Neutropenia predisposes
aeruginosa
Erysipelas S. pyogenes Raised, red, tender, with defined Rapid progression (minutes to hours);
border diabetes predisposes
Erythema chronicum migrans Borrelia burgdorferi Expanding erythematous macule1 Lesion is at site of tick bite
(ECM) of Lyme disease
Furuncle (boil, folliculitis) A. S. aureus Small pustule1 at hair follicle A. Contains neutrophils and gram-posi-
tive cocci
  B. P. aeruginosa   B. Causes “hot tub” folliculitis
Impetigo S. pyogenes and S. aureus Vesicles1 with honey-colored crust S. pyogenes skin infections predispose to
acute glomerulonephritis
(Continued)

mebooksfree.com
708 PART XII Pearls for the USMLE

TABLE XII–11 Important Skin Lesions Caused by Microorganisms (Continued )


Name or Type of Lesion Causative Organism Description of Lesion Comment

Malignant otitis externa P. aeruginosa Necrotic lesion on pinna of ear Diabetes predisposes
Papilloma (warts) Human papilloma virus (HPV) Raised, dry, noninflamed papules1 Benign tumors except HPV 16 and 18
cause carcinoma of cervix
Ringworm Trichophyton, Epidermophyton, Oval, inflamed, pruritic border with See hyphae in KOH prep
Microsporum central clearing
Scabies Sarcoptes scabiei Pruritic track or papule1 S. scabiei is called the “itch mite”
Slapped cheeks syndrome Parvovirus B19 Erythematous, macular, nontender  
rash on cheeks
Zoster (Shingles) Varicella-zoster virus (VZV) Painful, vesicles1 along sensory Reactivation of latent VZV infection
nerve
B. Multiple or disseminated      
lesions
Disseminated gonococcal Neisseria gonorrhoeae Scattered pustules and inflamed  
infection (DGI) tendons, especially of wrists and
fingers (tenosynovitis)
Erythema nodosum Systemic fungi (e.g., Coccidioi- Erythematous, tender nodules on Immunologic response to circulating
des) and Mycobacteria (e.g., skin over tibia or ulna antigen; no organisms in lesion
Mycobacterium tuberculosis
and Mycobacterium leprae)
Hand, foot, and mouth disease Coxsackie virus Vesicles in those locations  
Measles Measles virus Maculopapular splotchy (morbil- See Koplik’s spots on buccal mucosa;
liform) rash, especially on head rash caused by cytotoxic T-cell attack
and trunk on virus-infected cells
Petechial hemorrhage Many bacteria, (e.g., Neisseria Small area of bleeding into the skin A sign of disseminated intravascular
meningitidis) and viruses coagulation (DIC) that occurs in sepsis;
(e.g., Ebola virus) can enlarge to form purpuric (ecchy-
motic) lesions
Rocky Mountain spotted Rickettsia rickettsiae Petechial hemorrhages including Rickettsia infect and kill vascular endo-
fever on palms and soles thelium, resulting in hemorrhage into
skin
Rubella Rubella virus Maculopapular, nonconfluent rash Milder disease than measles
on face and trunk
Scalded skin syndrome S. aureus Desquamation over large area of Protease that cleaves desmoglein
body causes desquamation
Scarlet fever S. pyogenes Diffuse, macular, red (scarlet) rash; Caused by strains of S. pyogenes that
also strawberry tongue and cir- produces erythrogenic toxin that is a
cumoral pallor superantigen
Secondary syphilis T. pallidum Maculopapular rash on trunk,  
palms, and soles
Splinter hemorrhage Viridans streptococci, S. aureus Linear, black “splinters” under nails Sign of emboli from vegetation on heart
and other causes of valve
endocarditis
Toxic shock syndrome S. aureus Macular “sunburn-like” rash that Toxic shock syndrome toxin (TSST) is a
desquamates later superantigen
Varicella (chickenpox) VZV Pruritic vesicles on face and trunk  
1
Description of certain important skin lesions: Macule is a flat, erythematous lesion. Papule is a raised, erythematous lesion with no visible fluid inside; resembles a mosquito
bite. Vesicle is a raised, erythematous lesion with yellowish fluid (resembling plasma) inside; approximately the same size as a papule. Pustule is a raised, erythematous lesion
with cloudy fluid (pus) inside; typically larger than a papule or vesicle.

mebooksfree.com
PART XIII USMLE (NATIONAL BOARD)
PRACTICE QUESTIONS

These practice questions are presented in the format used by the are presented in the ONE-BEST-ANSWER format, and no
United States Medical Licensing Examination (USMLE) Step 1. EXCEPT type questions are used.
Note that in the computerized version of the USMLE, all ques- After the questions regarding the specific content areas (i.e.,
tions are of the “ONE-BEST-ANSWER” type. There are no bacteriology, virology, mycology, parasitology, and immunol-
questions of the “EXCEPT” or “LEAST ACCURATE” type in ogy), there are two additional sections, one containing questions
which you are asked to determine the one wrong answer. Nev- in an extended matching format and the other containing ques-
ertheless, for studying purposes, the EXCEPT or LEAST ACCU- tions based on infectious disease cases. The questions in the
RATE type of questions are excellent learning tools because they computerized version of the USMLE have 4 to 10 answer
provide you with several correct statements and only one incor- choices. Although the format of the questions in the extended
rect statement rather than several incorrect ones. In view of this matching section of this book is different from the format used
learning advantage, many practice questions in Part XIII of this in the USMLE, the questions in this section are designed to be a
book are of the EXCEPT or LEAST ACCURATE type. However, highly time-effective way of transmitting the important
in Part XIV, the questions in the USMLE Practice Examination information.

BASIC BACTERIOLOGY 4. Which one of the statements is the MOST accurate com-
parison of human, bacterial, and fungal cells?
DIRECTIONS (Questions 1–39): Select the ONE lettered answer that (A) Human cells undergo mitosis, whereas neither bacteria
is BEST in each question. nor fungi do.
1. Each of the following statements concerning the surface struc- (B) Human and fungal cells have a similar cell wall, in con-
tures of bacteria is correct EXCEPT: trast to bacteria, whose cell wall contains peptidoglycan.
(A) Pili mediate the interaction of bacteria with mucosal epithelium. (C) Human and bacterial cells have plasmids, whereas fun-
(B) Polysaccharide capsules retard phagocytosis. gal cells do not.
(C) Both gram-negative rods and cocci have lipopolysaccharide (D) Human and fungal cells have similar ribosomes,
(“endotoxin”) in their cell wall. whereas bacterial ribosomes are significantly different.
(D) Bacterial flagella are nonantigenic in humans because they 5. Which statement is MOST accurate regarding the drug
closely resemble human flagella in chemical composition. depicted in the diagram?
2. Each of the following statements concerning peptidoglycan is
correct EXCEPT:
(A) It has a backbone composed of alternating units of muramic
acid and acetylglucosamine.
(B) Cross-links between the tetrapeptides involve d-alanine.
(C) It is thinner in gram-positive than in gram-negative cells.
(D) It can be degraded by lysozyme.
3. Each of the following statements concerning bacterial spores is
correct EXCEPT:
(A) Their survival ability is based on their enhanced metabolic
activity.
(B) They are formed by gram-positive rods.
(C) They can be killed by being heated to 121°C for 15 minutes.
(D) They are formed primarily when nutrients are limited.

709

mebooksfree.com
710 PART XIII USMLE (National Board) Practice Questions

(A) It inhibits DNA synthesis. (C) Resistance to penicillin is known to be due to cleavage by
(B) It is bacteriostatic. β-lactamase.
(C) It binds to 30S ribosomes. (D) Resistance to tetracycline is known to be due to an enzyme
(D) It prevents formation of folic acid. that hydrolyzes the ester linkage.
6. Each of the following statements regarding the selective action 13. Of the following choices, the MOST important function of anti-
of antibiotics on bacteria is correct EXCEPT: body in host defenses against bacteria is:
(A) Chloramphenicol affects the large subunit of the bacterial (A) Activation of lysozyme that degrades the cell wall
ribosome, which is different from the large subunit of the (B) Acceleration of proteolysis of exotoxins
human ribosome. (C) Facilitation of phagocytosis
(B) Isoniazid affects the DNA polymerase of bacteria but not (D) Inhibition of bacterial protein synthesis
that of human cells. 14. Which of the following events is MOST likely to be due to bac-
(C) Sulfonamides affect folic acid synthesis in bacteria, a path- terial conjugation?
way that does not occur in human cells. (A) A strain of Corynebacterium diphtheriae produces a toxin
(D) Penicillins affect bacteria rather than human cells because encoded by a prophage.
bacteria have a cell wall, whereas human cells do not. (B) A strain of Pseudomonas aeruginosa produces β-lactamase
7. Each of the following statements concerning endotoxins is cor- encoded by a plasmid similar to a plasmid of another gram-
rect EXCEPT: negative organism.
(A) They are less toxic (i.e., less active on a weight basis) than (C) An encapsulated strain of Streptococcus pneumoniae acquires
exotoxins. the gene for capsule formation from an extract of DNA from
(B) They are more stable on heating than exotoxins. another encapsulated strain.
(C) They bind to specific cell receptors, whereas exotoxins do (D) A gene encoding resistance to gentamicin in the Escherichia
not. coli chromosome appears in the genome of a bacteriophage
(D) They are part of the bacterial cell wall, whereas exotoxins are that has infected E. coli.
not. 15. Which one of the following BEST describes the mode of action
8. The MAIN host defense against bacterial exotoxins is: of endotoxin?
(A) Activated macrophages secreting proteases (A) Degrades lecithin in cell membranes
(B) IgG and IgM antibodies (B) Inactivates elongation factor-2
(C) Helper T cells (C) Blocks release of acetylcholine
(D) Modulation of host cell receptors in response to the toxin (D) Causes the release of tumor necrosis factor
9. Which one of the following processes involves a sex pilus? 16. The identification of bacteria by serologic tests is based on the
(A) Transduction of a chromosomal gene presence of specific antigens. Which one of the following bac-
(B) Transposition of a mobile genetic element terial components is LEAST likely to contain useful antigens?
(C) Integration of a temperate bacteriophage (A) Capsule
(D) Conjugation resulting in transfer of an R (resistance) factor (B) Flagella
10. Each of the following statements concerning the normal flora is (C) Exotoxins
correct EXCEPT: (D) Ribosomes
(A) The most common organism found on the skin is Staphylo- 17. Each of the following statements concerning bacterial spores is
coccus epidermidis. correct EXCEPT:
(B) Escherichia coli is a prominent member of the normal flora of (A) Spores are formed under adverse environmental conditions
the throat. such as the absence of a carbon source.
(C) The major site where Bacteroides fragilis is found is the colon. (B) Spores are resistant to boiling.
(D) One of the most common sites where Staphylococcus aureus (C) Spores are metabolically inactive and contain dipicolinic
is found is the nose. acid, a calcium chelator.
11. Each of the following statements concerning the mechanism of (D) Spores are formed primarily by organisms of the genus
action of antimicrobial drugs is correct EXCEPT: Neisseria.
(A) Vancomycin acts by inhibiting peptidoglycan synthesis. 18. Each of the following statements concerning the mechanism of
(B) Quinolones, such as ciprofloxacin, act by inhibiting the DNA action of antibacterial drugs is correct EXCEPT:
gyrase of bacteria. (A) Cephalosporins are bactericidal drugs that inhibit the trans-
(C) Erythromycin is a bactericidal drug that disrupts cell mem- peptidase reaction and prevent cell wall synthesis.
branes by a detergent-like action. (B) Tetracyclines are bacteriostatic drugs that inhibit protein
(D) Aminoglycosides such as streptomycin are bactericidal drugs synthesis by blocking tRNA binding.
that inhibit protein synthesis. (C) Aminoglycosides are bacteriostatic drugs that inhibit pro-
12. Each of the following statements concerning the resistance of tein synthesis by activating ribonuclease, which degrades
bacteria to antimicrobial drugs is correct EXCEPT: mRNA.
(D) Erythromycin is a bacteriostatic drug that inhibits protein
(A) Resistance to chloramphenicol is known to be due to an
synthesis by blocking translocation of the polypeptide.
enzyme that acetylates the drug.
(B) Resistance to penicillin is known to be due to reduced affin-
ity of transpeptidases.

mebooksfree.com
PART XIII USMLE (National Board) Practice Questions 711

19. Each of the following is a typical property of obligate anaerobes 27. The effects of antibody on bacteria include each of the following
EXCEPT: EXCEPT:
(A) They generate energy by using the cytochrome system. (A) Lysis of gram-negative bacteria in conjunction with
(B) They grow best in the absence of air. complement
(C) They lack superoxide dismutase. (B) Augmentation of phagocytosis
(D) They lack catalase. (C) Increase in the frequency of lysogeny
20. Each of the following statements concerning the Gram stain is (D) Inhibition of adherence of bacteria to mucosal surfaces
correct EXCEPT: 28. Each of the following statements concerning exotoxins is cor-
(A) Escherichia coli stains pink because it has a thin peptidogly- rect EXCEPT:
can layer. (A) When treated chemically, some exotoxins lose their toxicity
(B) Streptococcus pyogenes stains blue because it has a thick pep- and can be used as immunogens in vaccines.
tidoglycan layer. (B) Some exotoxins are capable of causing disease in purified
(C) Mycobacterium tuberculosis stains blue because it has a thick form, free of any bacteria.
lipid layer. (C) Some exotoxins act in the gastrointestinal tract to cause
(D) Mycoplasma pneumoniae is not visible in the Gram stain diarrhea.
because it does not have a cell wall. (D) Some exotoxins contain lipopolysaccharides as the toxic
21. Each of the following statements concerning the killing of bac- component.
teria is correct EXCEPT: 29. Each of the following statements concerning bacterial and
(A) Lysozyme in tears can hydrolyze bacterial cell walls. human cells is correct EXCEPT:
(B) Silver nitrate can inactivate bacterial enzymes. (A) Bacteria are prokaryotic (i.e., they have one molecule of
(C) Detergents can disrupt bacterial cell membranes. DNA, are haploid, and have no nuclear membrane), whereas
(D) Ultraviolet light can degrade bacterial capsules. human cells are eukaryotic (i.e., they have multiple chromo-
22. In the Gram stain, the decolorization of gram-negative bacteria somes, are diploid, and have a nuclear membrane).
by acetone-alcohol is MOST closely related to: (B) Bacteria derive their energy by oxidative phosphorylation
(A) Proteins encoded by F plasmids within mitochondria in a manner similar to human cells.
(B) Lipids in the outer cell membrane (C) Bacterial and human ribosomes are of different sizes and
(C) 70S ribosomes chemical compositions.
(D) Branched polysaccharides in the capsule (D) Bacterial cells possess peptidoglycan, whereas human cells
do not.
23. Chemical modification of benzylpenicillin (penicillin G) has
resulted in several beneficial changes in the clinical use of this 30. Each of the following statements concerning penicillin is cor-
drug. Which one of the following is NOT one of those beneficial rect EXCEPT:
changes? (A) An intact β-lactam ring of penicillin is required for its
(A) Lowered frequency of anaphylaxis activity.
(B) Increased activity against gram-negative rods (B) The structure of penicillin resembles that of a dipeptide of
(C) Increased resistance to stomach acid alanine, which is a component of peptidoglycan.
(D) Reduced cleavage by penicillinase (C) Penicillin is a bacteriostatic drug because autolytic enzymes
are not activated.
24. Each of the following statements concerning resistance to anti-
(D) Penicillin inhibits transpeptidases, which are required for
biotics is correct EXCEPT:
cross-linking peptidoglycan.
(A) Resistance to aminoglycosides can be due to phosphorylat-
31. Each of the following statements concerning the mechanisms of
ing enzymes encoded by R plasmids.
resistance to antimicrobial drugs is correct EXCEPT:
(B) Resistance to sulfonamides can be due to enzymes that
hydrolyze the five-membered ring structure. (A) R factors are plasmids that carry the genes for enzymes that
(C) Resistance to penicillins can be due to alterations in binding modify one or more drugs.
proteins in the cell membrane. (B) Resistance to some drugs is due to a chromosomal mutation
(D) Resistance to cephalosporins can be due to cleavage of the that alters the receptor for the drug.
β-lactam ring. (C) Resistance to some drugs is due to transposon genes that
code for enzymes that inactivate the drugs.
25. The effects of endotoxin include each of the following EXCEPT:
(D) Resistance genes are rarely transferred by conjugation.
(A) Opsonization
32. Each of the following statements concerning endotoxins is cor-
(B) Fever
rect EXCEPT:
(C) Activation of the coagulation cascade
(D) Hypotension (A) The toxicity of endotoxins is due to the lipid portion of the
molecule.
26. Bacterial surface structures that show antigenic diversity
(B) Endotoxins are found in most gram-positive bacteria.
include each of the following EXCEPT:
(C) Endotoxins are located outside of the cell wall
(A) Pili peptidoglycan.
(B) Capsules (D) The antigenicity of somatic (O) antigen is due to repeating
(C) Flagella oligosaccharides.
(D) Peptidoglycan

mebooksfree.com
712 PART XIII USMLE (National Board) Practice Questions

33. Each of the following statements concerning exotoxins is cor- 39. Each of the following statements concerning cholera toxin is
rect EXCEPT: correct EXCEPT:
(A) Exotoxins are polypeptides. (A) Cholera toxin inhibits elongation factor-2 in the mucosal
(B) Exotoxins are more easily inactivated by heat than are epithelium.
endotoxins. (B) Binding of cholera toxin to the mucosal epithelium occurs
(C) Exotoxins are less toxic than the same amount of endotoxins. via interaction of the B subunit of the toxin with a ganglio-
(D) Exotoxins can be converted to toxoids. side in the cell membrane.
34. Each of the following statements concerning the killing of bac- (C) Cholera toxin acts by adding ADP-ribose to a G protein.
teria is correct EXCEPT: (D) Cholera toxin activates the enzyme adenylate cyclase in the
(A) A 70% solution of ethanol kills more effectively than absolute enterocyte.
(100%) ethanol. Answers (Questions 1–39)
(B) An autoclave uses steam under pressure to reach the killing 1. (D) 9. (D) 17. (D) 25. (A) 33. (C)
temperature of 121°C.
2. (C) 10. (B) 18. (C) 26. (D) 34. (D)
(C) The pasteurization of milk kills pathogens but allows many
organisms and spores to survive. 3. (A) 11. (C) 19. (A) 27. (C) 35. (A)
(D) Iodine kills by causing the formation of thymine dimers in 4. (D) 12. (D) 20. (C) 28. (D) 36. (C)
bacterial DNA. 5. (C) 13. (C) 21. (D) 29. (B) 37. (D)
35. Each of the following statements concerning the drug depicted 6. (B) 14. (B) 22. (B) 30. (C) 38. (B)
in the diagram is correct EXCEPT: 7. (C) 15. (D) 23. (A) 31. (D) 39. (A)
8. (B) 16. (D) 24. (B) 32. (B)

DIRECTIONS (Questions 40–51): Select the ONE lettered option that


is MOST closely associated with the numbered items. Each lettered
option may be selected once, more than once, or not at all.
Questions 40–43
(A) The drug is bacteriostatic. (A) Penicillins
(B) The drug inhibits cell wall synthesis. (B) Aminoglycosides
(C) The drug is made by a fungus. (C) Chloramphenicol
(D) The portion of the molecule required for activity is labeled B. (D) Rifampin
36. Each of the following statements concerning the normal flora is (E) Sulfonamides
correct EXCEPT: 40. Inhibit(s) bacterial RNA polymerase
(A) The normal flora of the colon consists predominantly of 41. Inhibit(s) cross-linking of peptidoglycan
anaerobic bacteria. 42. Inhibit(s) protein synthesis by binding to the 30S ribosomal
(B) The presence of the normal flora prevents certain pathogens subunit
from colonizing the upper respiratory tract. 43. Inhibit(s) folic acid synthesis
(C) Fungi (e.g., yeasts) are not members of the normal flora.
Questions 44–46
(D) Organisms of the normal flora are permanent residents of
the body surfaces. (A) Transduction
37. Each of the following statements concerning the structure and (B) Conjugation
chemical composition of bacteria is correct EXCEPT: (C) DNA transformation
(D) Transposition
(A) Some gram-positive cocci contain teichoic acid external to
the peptidoglycan. 44. During an outbreak of gastrointestinal disease caused by an
(B) Some gram-positive rods produce spores that are resistant to Escherichia coli strain sensitive to ampicillin, tetracycline, and
boiling. chloramphenicol, a stool sample from one patient yields E. coli
(C) Some gram-negative rods contain lipid A in their outer cell with the same serotype resistant to the three antibiotics.
membrane. 45. A mutant cell line lacking a functional thymidine kinase gene
(D) Some mycoplasmas contain pentaglycine in their was exposed to a preparation of DNA from normal cells; under
peptidoglycan. appropriate growth conditions, a colony of cells was isolated
that makes thymidine kinase.
38. Each of the following statements concerning the normal flora is
46. A retrovirus without an oncogene does not induce leukemia in
correct EXCEPT:
mice; after repeated passages through mice, viruses recovered
(A) Streptococcus mutans is found in the mouth and contributes from a tumor were highly oncogenic and contained a new gene.
to the formation of dental caries.
(B) The predominant organisms in the alveoli are viridans Questions 47–51
streptococci.
(A) Diphtheria toxin
(C) Bacteroides fragilis is found in greater numbers than Escherichia
(B) Tetanus toxin
coli in the colon.
(C) Botulinum toxin
(D) Candida albicans is part of the normal flora of both men and
(D) Toxic shock syndrome toxin
women.
(E) Cholera toxin

mebooksfree.com
PART XIII USMLE (National Board) Practice Questions 713

47. Causes paralysis by blocking release of acetylcholine Answers (Questions 40–51)


48. Inhibits protein synthesis by blocking elongation factor-2 40. (D) 43. (E) 46. (A) 49. (D)
49. Stimulates T cells to produce cytokines 41. (A) 44. (B) 47. (C) 50. (E)
50. Stimulates the production of cyclic AMP by adding ADP-ribose
42. (B) 45. (C) 48. (A) 51. (B)
to a G protein
51. Inhibits the release of inhibitory neurotransmitters causing
muscle spasms

CLINICAL BACTERIOLOGY
DIRECTIONS (Questions 52–136): Select the ONE lettered answer (C) Mycobacterium tuberculosis appears as a red rod in Gram-
that is BEST in each question. stained specimens.
52. An outbreak of sepsis caused by Staphylococcus aureus has (D) Mycobacterium tuberculosis appears as a red rod in acid-fast
occurred in the newborn nursery. You are called upon to inves- stained specimens.
tigate. According to your knowledge of the normal flora, what 58. A 50-year-old homeless alcoholic has a fever and is coughing up
is the MOST likely source of the organism? 1 cup of green, foul-smelling sputum per day. You suspect that
(A) Colon he may have a lung abscess. Which one of the following pairs of
(B) Nose organisms is MOST likely to be the cause?
(C) Throat (A) Listeria monocytogenes and Legionella pneumophila
(D) Vagina (B) Nocardia asteroides and Mycoplasma pneumoniae
53. Each of the statements about the classification of streptococci is (C) Fusobacterium nucleatum and Peptostreptococcus intermedius
correct EXCEPT: (D) Clostridium perfringens and Chlamydia psittaci
(A) Pneumococci (Streptococcus pneumoniae) are α-hemolytic 59. Which one of the following diseases is BEST diagnosed by sero-
and can be serotyped on the basis of their polysaccharide logic means?
capsules. (A) Q fever
(B) Enterococci are group D streptococci and can be classified by (B) Pulmonary tuberculosis
their ability to grow in 6.5% sodium chloride. (C) Gonorrhea
(C) Although pneumococci and the viridans streptococci are (D) Actinomycosis
α-hemolytic, they can be differentiated by the bile solubility 60. Your patient has subacute bacterial endocarditis caused by a
test and their susceptibility to optochin. member of the viridans group of streptococci. Which one of the
(D) Viridans streptococci are identified by Lancefield grouping, following sites is MOST likely to be the source of the organism?
which is based on the C carbohydrate in the cell wall. (A) Skin
54. Each of the following agents is a recognized cause of diarrhea (B) Colon
EXCEPT: (C) Oropharynx
(A) Clostridium perfringens (D) Urethra
(B) Enterococcus faecalis 61. A culture of skin lesions from a patient with pyoderma
(C) Escherichia coli (impetigo) shows numerous colonies surrounded by a zone
(D) Vibrio cholerae of β-hemolysis on a blood agar plate. A Gram-stained smear
55. Each of the following organisms is an important cause of uri- shows gram-positive cocci. If you found the catalase test to
nary tract infections EXCEPT: be negative, which one of the following organisms would you
(A) Escherichia coli MOST probably have isolated?
(B) Proteus mirabilis (A) Streptococcus pyogenes
(C) Klebsiella pneumoniae (B) Staphylococcus aureus
(D) Bacteroides fragilis (C) Staphylococcus epidermidis
56. Your patient is a 30-year-old woman with nonbloody diarrhea (D) Streptococcus pneumoniae
for the past 14 hours. Which one of the following organisms is 62. The coagulase test, in which the bacteria cause plasma to clot, is
LEAST likely to cause this illness? used to distinguish:
(A) Clostridium difficile (A) Streptococcus pyogenes from Enterococcus faecalis
(B) Streptococcus pyogenes (B) Streptococcus pyogenes from Staphylococcus aureus
(C) Shigella dysenteriae (C) Staphylococcus aureus from Staphylococcus epidermidis
(D) Salmonella enteritidis (D) Staphylococcus epidermidis from Neisseria meningitidis
57. Each of the following statements concerning Mycobacterium 63. Which one of the following is a virulence factor for Staphylococ-
tuberculosis is correct EXCEPT: cus aureus?
(A) After being stained with carbolfuchsin, M. tuberculosis (A) A heat-labile toxin that inhibits glycine release at the inter-
resists decolorization with acid alcohol. nuncial neuron
(B) Mycobacterium tuberculosis has a large amount of mycolic (B) An oxygen-labile hemolysin
acid in its cell wall. (C) Resistance to novobiocin
(D) Protein A that binds to the Fc portion of IgG

mebooksfree.com
714 PART XIII USMLE (National Board) Practice Questions

64. Which one of the following host defense mechanisms is 72. Each of the following statements concerning chlamydiae is cor-
the MOST important for preventing dysentery caused by rect EXCEPT:
Salmonella? (A) Chlamydiae are strict intracellular parasites because they
(A) Gastric acid cannot synthesize sufficient adenosine triphosphate (ATP).
(B) Salivary enzymes (B) Chlamydiae possess both DNA and RNA and are bounded
(C) Normal flora of the mouth by a cell wall.
(D) Alpha interferon (C) Chlamydia trachomatis has multiple serotypes that can cause
65. The MOST important protective function of the antibody stim- different diseases.
ulated by tetanus immunization is: (D) Most chlamydiae are transmitted by arthropods.
(A) To opsonize the pathogen (Clostridium tetani) 73. For which one of the following bacterial vaccines are toxic side
(B) To prevent growth of the pathogen effects an important concern?
(C) To prevent adherence of the pathogen (A) The vaccine containing pneumococcal polysaccharide
(D) To neutralize the toxin of the pathogen (B) The vaccine containing killed Bordetella pertussis
66. Five hours after eating reheated rice at a restaurant, a 24-year- (C) The vaccine containing tetanus toxoid
old woman and her husband both developed nausea, vomit- (D) The vaccine containing diphtheria toxoid
ing, and diarrhea. Which one of the following organisms is the 74. Each of the following statements concerning Staphylococcus
MOST likely to be involved? aureus is correct EXCEPT:
(A) Clostridium perfringens (A) Gram-positive cocci in grapelike clusters are seen on Gram-
(B) Enterotoxigenic Escherichia coli stained smear.
(C) Bacillus cereus (B) The coagulase test is positive.
(D) Salmonella typhi (C) Treatment should include a β-lactamase–resistant penicillin.
67. Which one of the following bacteria has the LOWEST 50% (D) Endotoxin is an important pathogenetic factor.
infectious dose (ID50)? 75. Your patient is a 70-year-old man who underwent bowel surgery
(A) Shigella sonnei for colon cancer 3 days ago. He now has a fever and abdomi-
(B) Vibrio cholerae nal pain. You are concerned that he may have peritonitis. Which
(C) Salmonella typhi one of the following pairs of organisms is MOST likely to be the
(D) Campylobacter jejuni cause?
68. For which one of the following enteric illnesses is a chronic car- (A) Bacteroides fragilis and Klebsiella pneumoniae
rier state MOST likely to develop? (B) Bordetella pertussis and Salmonella enteritidis
(A) Campylobacter enterocolitis (C) Actinomyces israelii and Campylobacter jejuni
(B) Shigella enterocolitis (D) Clostridium botulinum and Shigella dysenteriae
(C) Cholera 76. A 65-year-old man develops dysuria and hematuria. A Gram
(D) Typhoid fever stain of a urine sample shows gram-negative rods. Culture of
69. Which one of the following zoonotic illnesses has NO arthro- the urine on EMB agar reveals lactose-negative colonies with-
pod vector? out evidence of swarming motility. Which one of the following
organisms is MOST likely to be the cause of his urinary tract
(A) Plague
infection?
(B) Lyme disease
(C) Brucellosis (A) Enterococcus faecalis
(D) Epidemic typhus (B) Pseudomonas aeruginosa
(C) Proteus vulgaris
70. Which one of the following organisms principally infects vascu-
(D) Escherichia coli
lar endothelial cells and as a result often causes a petechial rash?
77. A 25-year-old man complains of a urethral discharge. You per-
(A) Salmonella typhi
form a Gram stain on a specimen of the discharge and see neu-
(B) Rickettsia rickettsii
trophils but no bacteria. Of the organisms listed, the one MOST
(C) Haemophilus influenzae
likely to cause the discharge is:
(D) Coxiella burnetii
(A) Treponema pallidum
71. Which one of the following statements MOST accurately
(B) Chlamydia trachomatis
depicts the ability of the organism to be cultured in the
(C) Candida albicans
laboratory?
(D) Coxiella burnetii
(A) Treponema pallidum from a chancre can be grown on a spe-
78. Two hours after a delicious Thanksgiving dinner of barley soup,
cial artificial medium supplemented with cholesterol.
roast turkey, stuffing, sweet potato, green beans, cranberry sauce,
(B) Mycobacterium leprae can be grown in the armadillo and the
and pumpkin pie topped with whipped cream, the Smith family
mouse footpad but not on any artificial media.
of four experience vomiting and diarrhea. Which one of the fol-
(C) Mycobacterium tuberculosis can be grown on enriched artifi-
lowing organisms is MOST likely to cause these symptoms?
cial media and produces visible colonies in 48 to 96 hours.
(D) Atypical mycobacteria are found widely in soil and water (A) Shigella flexneri
but cannot be cultured on artificial media in the (B) Campylobacter jejuni
laboratory. (C) Staphylococcus aureus
(D) Salmonella enteritidis

mebooksfree.com
PART XIII USMLE (National Board) Practice Questions 715

79. Your patient has a brain abscess that was detected 1 month after (C) Anaerobic conditions at the wound site are not required to
a dental extraction. Which one of the following organisms is cause tetanus, because spores will form in the presence of
MOST likely to be involved? oxygen.
(A) Anaerobic streptococci (D) Botulism, which is caused by ingesting preformed toxin, can
(B) Mycobacterium smegmatis be prevented by boiling food prior to eating.
(C) Lactobacillus acidophilus 87. Each of the following statements concerning Bacteroides fragilis
(D) Mycoplasma pneumoniae is correct EXCEPT:
80. The MOST important contribution of the capsule of Streptococ- (A) Bacteroides fragilis is a gram-negative rod that is part of the
cus pneumoniae to virulence is: normal flora of the colon.
(A) To prevent dehydration of the organisms on mucosal (B) Bacteroides fragilis forms endospores, which allow it to sur-
surfaces vive in the soil.
(B) To retard phagocytosis by polymorphonuclear leukocytes (C) The capsule of B. fragilis is an important virulence factor.
(C) To inhibit polymorphonuclear leukocyte chemotaxis (D) Bacteroides fragilis infections are characterized by foul-
(D) To accelerate tissue invasion by its collagenase-like activity smelling pus.
81. The MOST important way the host counteracts the function of 88. Each of the following statements concerning staphylococci is
the pneumococcal polysaccharide capsule is via: correct EXCEPT:
(A) T lymphocytes sensitized to polysaccharide antigens (A) Staphylococcus aureus is differentiated from Staphylococcus
(B) Polysaccharide-degrading enzymes epidermidis by the production of coagulase.
(C) Anticapsular antibody (B) Staphylococcus aureus infections are often associated with
(D) Activated macrophages abscess formation.
82. The pathogenesis of which one of the following organisms is (C) The majority of clinical isolates of S. aureus produce penicil-
MOST likely to involve invasion of the intestinal mucosa? linase; therefore, penicillin G should not be used for antibi-
otic therapy for S. aureus infections.
(A) Vibrio cholerae
(D) Scalded skin syndrome caused by S. aureus is due to enzy-
(B) Shigella sonnei
matic degradation of epidermal desmosomes by catalase.
(C) Enterotoxigenic Escherichia coli
(D) Clostridium botulinum 89. Acute glomerulonephritis is a nonsuppurative complication
that follows infection by which one of the following organisms?
83. Which one of the following organisms that infects the gastroin-
testinal tract is the MOST frequent cause of bacteremia? (A) Enterococcus faecalis
(B) Streptococcus pyogenes
(A) Shigella flexneri
(C) Streptococcus pneumoniae
(B) Campylobacter jejuni
(D) Streptococcus agalactiae
(C) Vibrio cholerae
(D) Salmonella typhi 90. Each of the following statements concerning gram-negative
rods is correct EXCEPT:
84. A 30-year-old woman with systemic lupus erythematosus is
found to have a positive serologic test for syphilis (VDRL test). (A) Escherichia coli is part of the normal flora of the colon; there-
She denies having had sexual contact with a partner who had fore, it does not cause diarrhea.
symptoms of a venereal disease. The next best step would be to: (B) Escherichia coli ferments lactose, whereas the enteric patho-
gens Shigella and Salmonella do not.
(A) Reassure her that the test is a false-positive reaction related
(C) Klebsiella pneumoniae, although a cause of pneumonia, is
to her autoimmune disorder
part of the normal flora of the colon.
(B) Trace her sexual contacts for serologic testing
(D) Proteus species are highly motile organisms that are found in
(C) Treat her with penicillin
the human colon and cause urinary tract infections.
(D) Perform a fluorescent treponemal antibody-absorbed (FTA-
ABS) test on a specimen of her serum 91. A 70-year-old man is found to have a hard mass in his prostate,
which is suspected to be a carcinoma. Twenty-four hours after
85. Each of the following statements concerning Treponema is cor-
surgical removal of the mass, he develops fever to 39°C and has
rect EXCEPT:
several shaking chills. Of the organisms listed, which one is
(A) Treponema pallidum produces an exotoxin that stimulates LEAST likely to be involved?
adenylate cyclase.
(A) Escherichia coli
(B) Treponema pallidum cannot be grown on conventional labo-
(B) Enterococcus faecalis
ratory media.
(C) Klebsiella pneumoniae
(C) Treponemes are members of the normal flora of the human
(D) Legionella pneumophila
oropharynx.
(D) Patients infected with T. pallidum produce antibodies that 92. Five days ago a 65-year-old woman with a lower urinary tract
react with beef heart cardiolipin. infection began taking ampicillin. She now has a fever and
severe diarrhea. Of the organisms listed, which one is MOST
86. Each of the following statements concerning clostridia is cor-
likely to be the cause of the diarrhea?
rect EXCEPT:
(A) Clostridium difficile
(A) Pathogenic clostridia are found both in the soil and in the
(B) Bacteroides fragilis
normal flora of the colon.
(C) Proteus mirabilis
(B) Antibiotic-associated (pseudomembranous) colitis is due to
(D) Bordetella pertussis
a toxin produced by Clostridium difficile.

mebooksfree.com
716 PART XIII USMLE (National Board) Practice Questions

93. The pathogenesis of which one of the following diseases does 102. Three organisms, Streptococcus pneumoniae, Neisseria men-
NOT involve an exotoxin? ingitidis, and Haemophilus influenzae, cause the vast majority
(A) Scarlet fever of cases of bacterial meningitis. What is the MOST important
(B) Typhoid fever pathogenic component they share?
(C) Toxic shock syndrome (A) Protein A
(D) Botulism (B) Capsule
94. Regarding the effect of benzylpenicillin (penicillin G) on bacte- (C) Endotoxin
ria, which one of the following organisms is LEAST likely to be (D) β-Lactamase
resistant? 103. Diarrhea caused by which one of the following agents is charac-
(A) Staphylococcus aureus terized by the presence of fecal leukocytes?
(B) Enterococcus faecalis (A) Campylobacter jejuni
(C) Streptococcus pyogenes (B) Rotavirus
(D) Neisseria gonorrhoeae (C) Clostridium perfringens
95. Which one of the following organisms is MOST likely to be the (D) Enterotoxigenic Escherichia coli
cause of pneumonia in an immunocompetent young adult? 104. Each of the following statements concerning Chlamydia tracho-
(A) Nocardia asteroides matis is correct EXCEPT:
(B) Serratia marcescens (A) It is an important cause of nongonococcal urethritis.
(C) Mycoplasma pneumoniae (B) It is the cause of lymphogranuloma venereum.
(D) Legionella pneumophila (C) It is an important cause of subacute bacterial endocarditis.
96. Each of the following statements concerning chlamydial genital (D) It is an important cause of conjunctivitis.
tract infections is correct EXCEPT: 105. Each of the following statements concerning Actinomyces and
(A) Infection can be diagnosed by finding antichlamydial anti- Nocardia is correct EXCEPT:
body in a serum specimen. (A) Actinomysis israelii is an anaerobic rod found as part of the
(B) Infection can persist after administration of penicillin. normal flora in the mouth.
(C) Symptomatic infections can be associated with urethral or cervi- (B) Both Actinomyces and Nocardia are branching, filamentous
cal discharge containing many polymorphonuclear leukocytes. rods.
(D) There is no vaccine against these infections. (C) Nocardia asteroides causes infections primarily in immuno-
97. Which one of the following illnesses is NOT a zoonosis? compromised patients.
(A) Typhoid fever (D) Infections are usually diagnosed by detecting a significant
(B) Q fever rise in antibody titer.
(C) Tularemia 106. Which one of the following types of organisms is NOT an obli-
(D) Rocky Mountain spotted fever gate intracellular parasite and therefore can replicate on bacte-
98. Which one of the following is NOT a characteristic of the Staph- riologic media?
ylococcus associated with toxic shock syndrome? (A) Chlamydia
(A) Release of a superantigen (B) Mycoplasma
(B) Coagulase production (C) Adenovirus
(C) Appearance of the organism in grapelike clusters on Gram- (D) Rickettsia
stained smear 107. Tissue-degrading enzymes play an important role in the patho-
(D) Catalase-negative reaction genesis of several bacteria. Which one of the following is NOT
99. Which one of the following is NOT an important characteristic involved in tissue or cell damage?
of either Neisseria gonorrhoeae or Neisseria meningitidis? (A) Lecithinase of Clostridium perfringens
(A) Polysaccharide capsule (B) Hyaluronidase of Streptococcus pyogenes
(B) IgA protease (C) M protein of Streptococcus pneumoniae
(C) M protein (D) Leukocidin of Staphylococcus aureus
(D) Pili 108. The soil is the natural habitat for certain microorganisms of
100. Which one of the following is NOT an important characteristic medical importance. Which one of the following is LEAST
of Streptococcus pyogenes? likely to reside there?
(A) Protein A (A) Clostridium tetani
(B) M protein (B) Mycobacterium avium-intracellulare
(C) β-hemolysin (C) Bacillus anthracis
(D) Polysaccharide group-specific substance (D) Chlamydia trachomatis
101. Each of the following is associated with the Lancefield group B 109. Which one of the following organisms is the MOST frequent
streptococci (S. agalactiae) EXCEPT: bacterial cause of pharyngitis?
(A) Pyoderma (impetigo) (A) Staphylococcus aureus
(B) Vaginal carriage in 5% to 25% of normal women of child- (B) Streptococcus pneumoniae
bearing age (C) Streptococcus pyogenes
(C) Neonatal sepsis and meningitis (D) Neisseria meningitidis
(D) β-hemolysis

mebooksfree.com
PART XIII USMLE (National Board) Practice Questions 717

110. Several pathogens are transmitted either during gestation or at 116. Each of the following statements concerning enterotoxins is
birth. Which one of the following is LEAST likely to be trans- correct EXCEPT:
mitted at these times? (A) Enterotoxins typically cause bloody diarrhea with leukocytes
(A) Haemophilus influenzae in the stool.
(B) Treponema pallidum (B) Staphylococcus aureus produces an enterotoxin that causes
(C) Neisseria gonorrhoeae vomiting and diarrhea.
(D) Chlamydia trachomatis (C) Vibrio cholerae causes cholera by producing an enterotoxin
111. Each of the following statements concerning exotoxins is cor- that increases adenylate cyclase activity within the
rect EXCEPT: enterocyte.
(A) Some strains of Escherichia coli produce an enterotoxin that (D) Escherichia coli enterotoxin mediates ADP-ribosylation of a
causes diarrhea. G protein.
(B) Cholera toxin acts by stimulating adenylate cyclase. 117. Each of the following statements concerning plague is correct
(C) Diphtheria is caused by an exotoxin that inhibits protein EXCEPT:
synthesis by inactivating an elongation factor. (A) Plague is caused by a gram-negative rod that can be cultured
(D) Botulism is caused by a toxin that hydrolyzes lecithin (leci- on blood agar.
thinase), thereby destroying nerve cells. (B) Plague is transmitted from the animal reservoir to humans
112. Each of the following statements concerning the VDRL test for by flea bite.
syphilis is correct EXCEPT: (C) The main reservoirs in nature are small rodents.
(A) The antigen is composed of inactivated Treponema (D) Plague is of concern in many underdeveloped countries but
pallidum. has not occurred in the United States since 1968.
(B) The test is usually positive in secondary syphilis. 118. Which one of the following statements concerning the organ-
(C) False-positive results are more frequent than with the fluo- isms that cause brucellosis is CORRECT?
rescent treponemal antibody-absorbed (FTA-ABS) test. (A) Brucellae are transmitted primarily by tick bite.
(D) The antibody titer declines with appropriate therapy. (B) The principal reservoirs of Brucellae are small rodents.
113. Each of the following statements concerning the fluorescent (C) Brucellae infect reticuloendothelial cells in the liver, spleen,
treponemal antibody-absorbed (FTA-ABS) test for syphilis is and bone marrow.
correct EXCEPT: (D) Brucellae are obligate intracellular parasites that are usually
(A) The test is specific for Treponema pallidum. identified by growth in human cell culture.
(B) The patient’s serum is absorbed with saprophytic 119. Each of the following statements concerning epidemic typhus is
treponemes. correct EXCEPT:
(C) Once positive, the test remains so despite appropriate (A) The disease is characterized by a rash.
therapy. (B) The Weil-Felix test can aid in diagnosis of the disease.
(D) The test is rarely positive in primary syphilis. (C) The disease is caused by a Rickettsia.
114. Each of the following statements concerning Corynebacterium (D) The causative organism is transmitted from rodents to
diphtheriae is correct EXCEPT: humans by a tick.
(A) Corynebacterium diphtheriae is a gram-positive rod that does 120. Which one of the following organisms causes diarrhea by pro-
not form spores. ducing an enterotoxin that increases adenylate cyclase activity
(B) Toxin production is dependent on the organism’s being within enterocytes?
lysogenized by a bacteriophage. (A) Escherichia coli
(C) Diphtheria toxoid should not be given to children younger (B) Bacteroides fragilis
than 3 years because the incidence of complications is too (C) Staphylococcus aureus
high. (D) Enterococcus faecalis
(D) Antitoxin should be used to treat patients with diphtheria. 121. Each of the following statements concerning Rocky Mountain
115. Each of the following statements concerning certain gram- spotted fever is correct EXCEPT:
negative rods is correct EXCEPT: (A) The causative organism forms β-hemolytic colonies on
(A) Pseudomonas aeruginosa causes wound infections that are blood agar.
characterized by blue-green pus as a result of pyocyanin (B) Headache, fever, and rash are characteristic features of the
production. disease.
(B) In unimmunized individuals, invasive disease caused by (C) The disease occurs primarily east of the Mississippi.
Haemophilus influenzae is most often due to strains possess- (D) The disease is caused by a Rickettsia.
ing a type b polysaccharide capsule. 122. Each of the following statements concerning Clostridium per-
(C) Legionella pneumophila infection is acquired by inhalation of fringens is correct EXCEPT:
aerosols from environmental water sources. (A) It causes gas gangrene.
(D) Whooping cough, which is caused by Bordetella pertussis, is (B) It causes food poisoning.
on the rise because changing antigenicity of the organism has (C) It produces an exotoxin that degrades lecithin and causes
made the vaccine relatively ineffective. necrosis and hemolysis.
(D) It is a gram-negative rod that does not ferment lactose.

mebooksfree.com
718 PART XIII USMLE (National Board) Practice Questions

123. Each of the following statements concerning Clostridium tetani 129. Which one of the following statements concerning Legionella
is correct EXCEPT: pneumophila is CORRECT?
(A) It is a gram-positive, spore-forming rod. (A) It is part of the normal flora of the colon.
(B) Pathogenesis is due to the production of an exotoxin that (B) It cannot be grown on laboratory media.
blocks inhibitory neurotransmitters. (C) It does not have a cell wall.
(C) It is a facultative organism; it will grow on a blood agar plate (D) It causes atypical pneumonia, especially in those with
in the presence of room air. reduced cell-mediated immunity.
(D) Its natural habitat is primarily the soil. 130. Each of the following statements concerning wound infections
124. Each of the following statements concerning spirochetes is cor- caused by Clostridium perfringens is correct EXCEPT:
rect EXCEPT: (A) An exotoxin plays a role in pathogenesis.
(A) Species of Treponema are part of the normal flora of the (B) Gram-positive rods are found in the exudate.
mouth. (C) The organism grows only in human cell culture.
(B) Species of Borrelia cause a tick-borne disease called relapsing (D) Anaerobic culture of the wound site should be ordered.
fever. 131. Each of the following statements concerning infection with
(C) The species of Leptospira that cause leptospirosis grow pri- Chlamydia psittaci is correct EXCEPT:
marily in humans and are usually transmitted by human-to- (A) Chylamydia psittaci can be isolated by growth in cell culture
human contact. and will not grow in blood agar.
(D) Species of Treponema cause syphilis and yaws. (B) The organism appears purple in Gram-stained smears of
125. Each of the following statements concerning gonorrhea is cor- sputum.
rect EXCEPT: (C) The infection is more readily diagnosed by serologic tests
(A) Infection in men is more frequently symptomatic than in than by isolation of the organism.
women. (D) The infection is more commonly acquired from a nonhuman
(B) A presumptive diagnosis can be made by finding gram- source than from another human.
negative kidney bean-shaped diplococci within neutrophils 132. Ticks are vectors for the transmission of each of the following
in a urethral discharge. diseases EXCEPT:
(C) The definitive diagnosis can be made by detecting antibodies (A) Rocky Mountain spotted fever
to Neisseria gonorrhoeae in the patient’s serum. (B) Epidemic typhus
(D) Gonococcal conjunctivitis of the newborn rarely occurs in (C) Tularemia
the United States, because silver nitrate or erythromycin is (D) Lyme disease
commonly used as prophylaxis.
133. Each of the following statements concerning pneumonia caused
126. Each of the following statements concerning Mycobacterium by Mycoplasma pneumoniae is correct EXCEPT:
tuberculosis is correct EXCEPT:
(A) Pneumonia caused by M. pneumoniae is associated with a
(A) Some strains of M. tuberculosis isolated from patients exhibit rise in the titer of cold agglutinins.
multiple drug resistance (i.e., they are resistant to both iso- (B) Pneumonia caused by M. pneumoniae occurs primarily in
niazid and rifampin). immunocompetent individuals.
(B) Mycobacterium tuberculosis contains a small amount of lipid (C) Pneumonia caused by M. pneumoniae is an “atypical”
in its cell wall and therefore stains poorly with the Gram pneumonia.
stain. (D) Mycoplasma pneumoniae cannot be cultured in vitro because
(C) Mycobacterium tuberculosis grows slowly, often requiring 3 it has no cell wall.
to 6 weeks before colonies appear.
134. Each of the following statements concerning Neisseria menin-
(D) The antigen in the tuberculin skin test is a protein extracted
gitidis is correct EXCEPT:
from the organism.
(A) It is an oxidase-positive, gram-negative diplococcus.
127. Which one of the following statements concerning immuniza-
(B) It contains endotoxin in its cell wall.
tion against diseases caused by clostridia is CORRECT?
(C) It produces an exotoxin that stimulates adenylate cyclase.
(A) Antitoxin against tetanus protects against botulism as well, (D) It has a polysaccharide capsule that is antiphagocytic.
because the two toxins share antigenic sites.
135. Each of the following statements concerning Q fever is correct
(B) Vaccines containing alpha toxin (lecithinase) are effective in
EXCEPT:
protecting against gas gangrene.
(C) The toxoid vaccine against Clostridium difficile infection (A) Rash is a prominent feature.
should be administered to immunocompromised patients. (B) It is transmitted by respiratory aerosol.
(D) Immunization with tetanus toxoid induces effective protec- (C) Farm animals are an important reservoir.
tion against tetanus toxin. (D) It is caused by Coxiella burnetii.
128. Each of the following statements concerning neisseriae is cor- 136. Each of the following statements concerning Mycobacterium
rect EXCEPT: leprae is correct EXCEPT:
(A) They are gram-negative diplococci. (A) In lepromatous leprosy, large numbers of organisms are usu-
(B) They produce IgA protease as a virulence factor. ally seen in acid-fast-stained smears.
(C) They are oxidase-positive. (B) The organism will grow on bacteriologic media in 3 to 6
(D) They grow best under anaerobic conditions. weeks.

mebooksfree.com
PART XIII USMLE (National Board) Practice Questions 719

(C) Prolonged therapy (9 months or longer) is required to pre- (C) Bacillus anthracis
vent recurrence. (D) Clostridium botulinum
(D) Loss of sensation due to nerve damage is often seen in 144. Causes both skin lesions and a severe pneumonia
leprosy. 145. Causes flaccid paralysis
146. Causes a pseudomembrane in the throat, which can cause
Answers (Questions 52–136)
respiratory tract obstruction
52. (B) 69. (C) 86. (C) 103. (A) 120. (A) 147. Causes meningitis in neonates and the immunosuppressed
53. (D) 70. (B) 87. (B) 104. (C) 121. (A) Questions 148–150
54. (B) 71. (B) 88. (D) 105. (D) 122. (D)
(A) Escherichia coli
55. (D) 72. (D) 89. (B) 106. (B) 123. (C) (B) Klebsiella pneumoniae
56. (B) 73. (B) 90. (A) 107. (C) 124. (C) (C) Salmonella enteritidis
57. (C) 74. (D) 91. (D) 108. (D) 125. (C) (D) Proteus mirabilis
58. (C) 75. (A) 92. (A) 109. (C) 126. (B) 148. Is frequently implicated in nosocomial infections, is an impor-
59. (A) 76. (B) 93. (B) 110. (A) 127. (D) tant cause of community-acquired pneumonia in adults, and
60. (C) 77. (B) 94. (C) 111. (D) 128. (D) has a thick, mucoid capsule
149. Is the most common cause of urinary tract infections
61. (A) 78. (C) 95. (C) 112. (A) 129. (D)
150. Pathogenicity associated primarily with urinary tract infec-
62. (C) 79. (A) 96. (A) 113. (D) 130. (C) tions; produces urease
63. (D) 80. (B) 97. (A) 114. (C) 131. (B)
Questions 151–154
64. (A) 81. (C) 98. (D) 115. (D) 132. (B)
65. (D) 82. (B) 99. (C) 116. (A) 133. (D) (A) Staphylococcus aureus
66. (C) 83. (D) 100. (A) 117. (D) 134. (C) (B) Streptococcus pyogenes
(C) Enterococcus faecalis
67. (A) 84. (D) 101. (A) 118. (C) 135. (A)
(D) Streptococcus pneumoniae
68. (D) 85. (A) 102. (B) 119. (D) 136. (B)
151. Grows in 6.5% sodium chloride
152. Is bile soluble
DIRECTIONS (Questions 137–158): Select the ONE lettered option 153. Produces enterotoxin
that is MOST closely associated with the numbered items. Each lettered 154. Is associated with rheumatic fever
option may be selected once, more than once, or not at all. Questions 155–158
Questions 137–140 (A) Bacteroides fragilis
(A) Mycobacterium avium-intracellulare (B) Haemophilus influenzae
(B) Treponema pallidum (C) Pseudomonas aeruginosa
(C) Rickettsia prowazekii (D) Chlamydia pneumoniae
(D) Mycoplasma pneumoniae 155. Coccobacillary gram-negative rod that causes meningitis in
137. Is an obligate intracellular parasite young children
138. Is found primarily in the soil 156. Oxidase-positive gram-negative rod that is an important cause
139. Has no cell wall of wound and burn infections
140. Is an acid-fast rod 157. Causes atypical pneumonia in immunocompetent adults
158. Anaerobic gram-negative rod that is an important cause of
Questions 141–143 peritonitis
(A) Borrelia burgdorferi Answers (Questions 137–158)
(B) Helicobacter pylori
(C) Pasteurella multocida 137. (C) 142. (C) 147. (B) 152. (D) 157. (D)
(D) Brucella melitensis 138. (A) 143. (A) 148. (B) 153. (A) 158. (A)
141. Peptic ulcer in a 45-year-old salesman 139. (D) 144. (C) 149. (A) 154. (B)
142. Cellulitis of the hand following a cat bite 140. (A) 145. (D) 150. (D) 155. (B)
143. Expanding, bull’s eye–shaped red rash in a 6-year-old boy after 141. (B) 146. (A) 151. (C) 156. (C)
a camping trip
Questions 144–147
(A) Corynebacterium diphtheriae
(B) Listeria monocytogenes

mebooksfree.com
720 PART XIII USMLE (National Board) Practice Questions

BASIC VIROLOGY
DIRECTIONS (Questions 159–192): Select the ONE lettered answer 165. Which one of the following forms of immunity to viruses would
that is BEST in each question. be LEAST likely to be lifelong?
159. Viruses enter cells by adsorbing to specific sites on the outer (A) Passive immunity
membrane of cells. Each of the following statements regarding (B) Passive–active immunity
this event is correct EXCEPT: (C) Active immunity
(A) The interaction determines the specific target organs for (D) Cell-mediated immunity
infection. 166. Which one of the following statements concerning alpha, beta,
(B) The interaction determines whether the purified genome of and gamma interferons is LEAST accurate?
a virus is infectious. (A) Interferons inhibit a broad range of viruses, not just the virus
(C) The interaction can be prevented by neutralizing antibody. that induced the interferon.
(D) If the sites are occupied, interference with virus infection (B) Interferons are synthesized only by virus-infected cells.
occurs. (C) Interferons induce the synthesis of a protein kinase that
160. Many viruses mature by budding through the outer membrane phosphorylates an elongation factor, thereby inactivating
of the host cell. Each of the following statements regarding protein synthesis.
these viruses is correct EXCEPT: (D) Interferons induce the synthesis of a ribonuclease that
(A) Some of these viruses cause multinucleated giant cell degrades viral mRNA.
formation. 167. You have isolated a virus from the stool of a patient with diar-
(B) Some new viral antigens appear on the surface of the host rhea and shown that its genome is composed of multiple pieces
cell. of double-stranded RNA. Which one of the following is LEAST
(C) Some of these viruses contain host cell lipids. LIKELY to be true?
(D) Some of these viruses do not have an envelope. (A) Each piece of RNA encodes a different protein.
161. Biochemical analysis of a virus reveals the genome to be composed (B) The virus encodes an RNA-directed RNA polymerase.
of eight unequally sized pieces of single-stranded RNA, each of (C) The virion contains an RNA polymerase.
which is complementary to viral mRNA in infected cells. Which (D) The genome integrates into the host chromosome.
one of the following statements is UNLIKELY to be correct? 168. A temperate bacteriophage has been induced from a new patho-
(A) Different proteins are encoded by each segment of the viral genic strain of Escherichia coli that produces a toxin. Which one
genome. of the following is the MOST convincing way to show that the
(B) The virus particle contains a virus-encoded enzyme that can phage encodes the toxin?
copy the genome into its complement. (A) Carry out conjugation of the pathogenic strain with a non-
(C) Purified RNA extracted from the virus particle is infectious. pathogenic strain.
(D) The virus can acquire new antigens via reassortment of its (B) Infect an experimental animal with the phage.
RNA segments. (C) Lysogenize a nonpathogenic strain with the phage.
162. Latency is an outcome particularly characteristic of which one (D) Look for transposable elements in the phage DNA.
of the following virus groups? 169. Each of the following statements concerning retroviruses is cor-
(A) Polioviruses rect EXCEPT:
(B) Herpesviruses (A) The virion carries an RNA-directed DNA polymerase
(C) Rhinoviruses encoded by the viral genome.
(D) Influenza viruses (B) The viral genome consists of three segments of double-
163. Each of the following statements concerning viral serotypes is stranded RNA.
correct EXCEPT: (C) The virion is enveloped and enters cells via an interaction
(A) In naked nucleocapsid viruses, the serotype is usually deter- with specific receptors on the host cell.
mined by the outer capsid proteins. (D) During infection, the virus synthesizes a DNA copy of its
(B) In enveloped viruses, the serotype is usually determined by RNA, and this DNA becomes covalently integrated into host
the outer envelope proteins, especially the spike proteins. cell DNA.
(C) Some viruses have multiple serotypes. 170. A stock of virus particles has been found by electron micros-
(D) Some viruses have an RNA polymerase that determines the copy to contain 108 particles/mL, but a plaque assay reveals only
serotype. 105 plaque-forming units/mL. The BEST interpretation of these
164. The ability of a virus to produce disease can result from a vari- results is that:
ety of mechanisms. Which one of the following mechanisms is (A) Only one particle in 1000 is infectious.
LEAST likely? (B) A nonpermissive cell line was used for the plaque assay.
(A) Cytopathic effect in infected cells (C) Several kinds of viruses were present in the stock.
(B) Malignant transformation of infected cells (D) The virus is a temperature-sensitive mutant.
(C) Immune response to virus-induced antigens on the surface 171. Reasonable mechanisms for viral persistence in infected indi-
of infected cells viduals include all of the following EXCEPT:
(D) Production of an exotoxin that activates adenylate cyclase (A) Generation of defective-interfering particles
(B) Virus-mediated inhibition of host DNA synthesis

mebooksfree.com
PART XIII USMLE (National Board) Practice Questions 721

(C) Integration of a provirus into the genome of the host (C) Some viruses contain lipoprotein envelopes derived from the
(D) Host tolerance to viral antigens plasma membrane of the host cell.
172. Each of the following statements concerning viral surface pro- (D) The nucleic acid of some viruses is single-stranded DNA and
teins is correct EXCEPT: that of others is double-stranded RNA.
(A) They elicit antibody that neutralizes infectivity of the virus. 179. Which one of the following statements about virion structure
(B) They determine the species specificity of the virus–cell and assembly is CORRECT?
interaction. (A) Most viruses acquire surface glycoproteins by budding
(C) They participate in active transport of nutrients across the through the nuclear membrane.
viral envelope membrane. (B) Helical nucleocapsids are found primarily in DNA viruses.
(D) They protect the genetic material against nucleases. (C) The symmetry of virus particles prevents inclusion of any
173. Each of the following statements concerning viral vaccines is nonstructural proteins, such as enzymes.
correct EXCEPT: (D) Enveloped viruses use a matrix protein to mediate interac-
(A) In live, attenuated vaccines, the virus has lost its ability to tions between viral glycoproteins in the plasma membrane
cause disease but has retained its ability to induce neutraliz- and structural proteins in the nucleocapsid.
ing antibody. 180. Each of the following statements concerning viruses is correct
(B) In live, attenuated vaccines, the possibility of reversion to EXCEPT:
virulence is of concern. (A) Viruses can reproduce only within cells.
(C) With inactivated vaccines, IgA mucosal immunity is usually (B) The proteins on the surface of the virus mediate the entry of
induced. the virus into host cells.
(D) With inactivated vaccines, protective immunity is due mainly (C) Neutralizing antibody is directed against proteins on the
to the production of IgG. surface of the virus.
174. The major barrier to the control of rhinovirus upper respiratory (D) Viruses replicate by binary fission.
infections by immunization is: 181. Viruses are obligate intracellular parasites. Each of the follow-
(A) The poor local and systemic immune response to these viruses ing statements concerning this fact is correct EXCEPT:
(B) The large number of serotypes of the rhinoviruses (A) Viruses cannot generate energy outside of cells.
(C) The side effects of the vaccine (B) Viruses cannot synthesize proteins outside of cells.
(D) The inability to grow the viruses in cell culture (C) Viruses must degrade host cell DNA in order to obtain
175. The feature of the influenza virus genome that contributes nucleotides.
MOST to the antigenic variation of the virus is: (D) Enveloped viruses require host cell membranes to obtain
(A) A high G + C content, which augments binding to nucleo- their envelopes.
proteins 182. Each of the following statements concerning lysogeny is correct
(B) Inverted repeat regions, which create “sticky ends” EXCEPT:
(C) Segmented nucleic acid (A) Viral genes replicate independently of bacterial genes.
(D) Unique methylated bases (B) Viral genes responsible for lysis are repressed.
176. Which one of the following is the BEST explanation for the (C) Viral DNA is integrated into bacterial DNA.
selective action of acyclovir (acycloguanosine) in herpes sim- (D) Some lysogenic bacteriophages encode toxins that cause
plex virus (HSV)-infected cells? human disease.
(A) Acyclovir binds specifically to viral receptors only on the 183. Each of the following viruses possesses an outer envelope of
surface of the HSV-infected cell. lipoprotein EXCEPT:
(B) Acyclovir is phosphorylated by a virus-encoded phosphoki- (A) Varicella-zoster virus
nase only within HSV-infected cells. (B) Papillomavirus
(C) Acyclovir selectively inhibits the RNA polymerase in the (C) Influenza virus
HSV virion. (D) Human immunodeficiency virus
(D) Acyclovir specifically blocks the matrix protein of HSV, 184. Which one of the following viruses possesses a genome of
thereby preventing release of progeny HSV. single-stranded RNA that is infectious when purified?
177. Each of the following statements concerning interferon is cor- (A) Influenza virus
rect EXCEPT: (B) Rotavirus
(A) Interferon inhibits the growth of both DNA and RNA viruses. (C) Measles virus
(B) Interferon is induced by double-stranded RNA. (D) Poliovirus
(C) Interferon made by cells of one species acts more effectively 185. Each of the following viruses possesses an RNA polymerase in
in the cells of that species than in the cells of other species. the virion EXCEPT:
(D) Interferon acts by preventing viruses from entering the cell. (A) Hepatitis A virus
178. Each of the following statements concerning the viruses that (B) Smallpox virus
infect humans is correct EXCEPT: (C) Mumps virus
(A) Only viruses with a negative polarity RNA genome have a (D) Rotavirus
polymerase in the virion.
(B) The purified nucleic acid of some viruses is infectious, but at
a lower efficiency than the intact virions.

mebooksfree.com
722 PART XIII USMLE (National Board) Practice Questions

186. Each of the following viruses possesses a DNA polymerase in DIRECTIONS (Questions 193–211): Select the one lettered option
the virion EXCEPT: that is MOST CLOSELY associated with the numbered items. Each let-
(A) Human immunodeficiency virus tered option may be selected once, more than once, or not at all.
(B) Human T-cell lymphotropic virus Questions 193–196
(C) Epstein–Barr virus
(D) Hepatitis B virus (A) DNA enveloped virus
(B) DNA nonenveloped virus
187. Each of the following viruses possesses double-stranded nucleic
(C) RNA enveloped virus
acid as its genome EXCEPT:
(D) RNA nonenveloped virus
(A) Coxsackie virus
193. Herpes simplex virus
(B) Herpes simplex virus
194. Human T-cell lymphotropic virus
(C) Rotavirus
195. Human papillomavirus
(D) Adenovirus
196. Rotavirus
188. Regarding viroids, which one of the following statements is the
MOST accurate? Questions 197–201
(A) They are defective viruses that are missing the DNA coding (A) Attachment and penetration of virion
for the matrix protein. (B) Viral mRNA synthesis
(B) They consist of RNA without a protein or lipoprotein outer (C) Viral protein synthesis
coat. (D) Viral genome DNA synthesis
(C) They cause tumors in experimental animals. (E) Assembly and release of progeny virus
(D) They require an RNA polymerase in the particle for replica- 197. Main site of action of acyclovir
tion to occur. 198. Main site of action of amantadine
189. Each of the following statements about both measles virus and 199. Function of virion polymerase of influenza virus
rubella virus is correct EXCEPT: 200. Main site of action of antiviral antibody
(A) They are RNA enveloped viruses. 201. Step at which budding occurs
(B) Their virions contain an RNA polymerase.
Questions 202–206
(C) They have a single antigenic type.
(D) They are transmitted by respiratory aerosol. (A) Poliovirus
190. Each of the following statements about both influenza virus and (B) Epstein–Barr virus
rabies virus is correct EXCEPT: (C) Prions
(D) Hepatitis B virus
(A) They are enveloped RNA viruses.
(E) Respiratory syncytial virus
(B) Their virions contain an RNA polymerase.
(C) A killed vaccine is available for both viruses. 202. Part of the genome DNA is synthesized by the virion polymerase.
(D) They each have a single antigenic type. 203. The translation product of viral mRNA is a polyprotein that is
cleaved to form virion structural proteins.
191. Each of the following statements about both poliovirus and rhi-
204. It is remarkably resistant to ultraviolet light.
noviruses is correct EXCEPT:
205. It causes latent infection of B cells.
(A) They are nonenveloped RNA viruses. 206. An envelope protein induces the formation of giant cells.
(B) They have multiple antigenic types.
(C) Their virions contain an RNA polymerase. Questions 207–211
(D) They do not integrate their genome into host cell DNA. (A) Hepatitis A virus
192. Each of the following statements about human immunodefi- (B) Hepatitis B virus
ciency virus (HIV) is correct EXCEPT: (C) Hepatitis C virus
(A) HIV is an enveloped RNA virus. (D) Hepatitis D virus
(B) The virion contains an RNA-dependent DNA polymerase. 207. Enveloped DNA virus that is transmitted by blood
(C) A DNA copy of the HIV genome integrates into host cell 208. Enveloped RNA virus that has the surface antigen of another
DNA. virus
(D) Acyclovir inhibits HIV replication. 209. Enveloped RNA virus that is the most common cause of non-A,
non-B hepatitis
Answers (Questions 159–192)
210. Nonenveloped RNA virus that is transmitted by the fecal–oral
159. (B) 166. (B) 173. (C) 180. (D) 187. (A) route
160. (D) 167. (D) 174. (B) 181. (C) 188. (B) 211. Purified surface protein of this virus is the immunogen in a
161. (C) 168. (C) 175. (C) 182. (A) 189. (B) vaccine
162. (B) 169. (B) 176. (B) 183. (B) 190. (D)
Answers (Questions 193–211)
163. (D) 170. (A) 177. (D) 184. (D) 191. (C)
164. (D) 171. (B) 178. (A) 185. (A) 192. (D) 193. (A) 198. (A) 203. (A) 208. (D)
165. (A) 172. (C) 179. (D) 186. (C) 194. (C) 199. (B) 204. (C) 209. (C)
195. (B) 200. (A) 205. (B) 210. (A)
196. (D) 201. (E) 206. (E) 211. (B)
197. (D) 202. (D) 207. (B)

mebooksfree.com
PART XIII USMLE (National Board) Practice Questions 723

CLINICAL VIROLOGY
DIRECTIONS (Questions 212–275): Select the ONE lettered answer 219. Each of the following statements concerning influenza is
that is BEST in each question. correct EXCEPT:
212. Which one of the following outcomes is MOST common fol- (A) Major epidemics of the disease are caused by influenza A
lowing a primary herpes simplex virus infection? viruses rather than influenza B and C viruses.
(B) Likely sources of new antigens for influenza A viruses are the
(A) Complete eradication of virus and virus-infected cells
viruses that cause influenza in animals.
(B) Persistent asymptomatic viremia
(C) Major antigenic changes (shifts) of viral surface proteins are
(C) Establishment of latent infection
seen primarily in influenza A viruses rather than in influenza B
(D) Persistent cytopathic effect in infected cells
and C viruses.
213. Each of the following pathogens is likely to establish chronic or (D) The antigenic changes that occur with antigenic drift are due
latent infection EXCEPT: to reassortment of the multiple pieces of the influenza virus
(A) Cytomegalovirus genome.
(B) Hepatitis A virus 220. Each of the following statements concerning the prevention and
(C) Hepatitis B virus treatment of influenza is correct EXCEPT:
(D) Herpes simplex virus
(A) The inactivated influenza vaccine contains H1N1 virus, whereas
214. Each of the following statements regarding poliovirus and its the live, attenuated influenza vaccine contains H3N2 virus.
vaccine is correct EXCEPT: (B) The vaccine is recommended to be given each year because
(A) Poliovirus is transmitted by the fecal–oral route. the antigenicity of the virus drifts.
(B) Pathogenesis by poliovirus primarily involves the death of (C) Oseltamivir (Tamiflu) is effective against both influenza A
sensory neurons. and influenza B viruses.
(C) The live, attenuated vaccine contains all three serotypes of (D) The main antigen in the vaccine that induces protective anti-
poliovirus. body is the hemagglutinin.
(D) An unimmunized adult traveling to countries where there is 221. A 6-month-old child develops a persistent cough and a fever.
a known risk of being infected with poliovirus should receive Physical examination and chest X-ray suggest pneumonia.
the inactivated vaccine. Which one of the following organisms is LEAST likely to cause
215. Which one of the following strategies is MOST likely to induce this infection?
lasting intestinal mucosal immunity to poliovirus?
(A) Respiratory syncytial virus
(A) Parenteral (intramuscular) immunization with inactivated (B) Adenovirus
vaccine (C) Parainfluenza virus
(B) Oral administration of poliovirus immune globulin (D) Rotavirus
(C) Parenteral immunization with live vaccine
222. A 45-year-old man was attacked by a bobcat and bitten repeat-
(D) Oral immunization with live vaccine
edly about the face and neck. The animal was shot by a com-
216. Each of the following clinical syndromes is associated with panion and brought back to the public health authorities. Once
infection by picornaviruses EXCEPT: you decide to immunize against rabies virus, how would you
(A) Myocarditis/pericarditis proceed?
(B) Hepatitis (A) Use hyperimmune serum only
(C) Mononucleosis (B) Use active immunization only
(D) Meningitis (C) Use hyperimmune serum and active immunization
217. Each of the following statements concerning rubella vaccine is (D) Use active immunization and follow this with hyperimmune
correct EXCEPT: serum if adequate antibody titers are not obtained in the
(A) The vaccine prevents reinfection, thereby limiting the spread patient’s serum
of virulent virus. 223. Each of the following statements concerning mumps is correct
(B) The immunogen in the vaccine is killed rubella virus. EXCEPT:
(C) The vaccine induces antibodies that prevent dissemination (A) Mumps virus is a paramyxovirus and hence has a single-
of the virus by neutralizing it during the viremic stage. stranded RNA genome.
(D) The incidence of both childhood rubella and congenital (B) Meningitis is a recognized complication of mumps.
rubella syndrome has decreased significantly since the (C) Mumps orchitis in children prior to puberty often causes
advent of the vaccine. sterility.
218. Each of the following statements concerning the rabies vaccine (D) During mumps, the virus spreads through the bloodstream
for use in humans is correct EXCEPT: (viremia) to various internal organs.
(A) The vaccine contains live, attenuated rabies virus. 224. Each of the following statements concerning respiratory syncy-
(B) If your patient is bitten by a wild animal (e.g., a skunk) the tial virus (RSV) is correct EXCEPT:
rabies vaccine should be given.
(A) RSV has a single-stranded RNA genome.
(C) When the vaccine is used for postexposure prophylaxis,
(B) RSV induces the formation of multinucleated giant cells.
rabies immune globulin should also be given.
(C) RSV causes pneumonia primarily in children.
(D) The virus in the vaccine is grown in human cell cultures, thus
(D) RSV infections can be effectively treated with acyclovir.
decreasing the risk of allergic encephalomyelitis.

mebooksfree.com
724 PART XIII USMLE (National Board) Practice Questions

225. The principal reservoir for the antigenic shift variants of influ- 232. The slow virus disease that MOST clearly has immunosuppres-
enza virus appears to be: sion as an important factor in its pathogenesis is:
(A) People in isolated communities such as the Arctic (A) Progressive multifocal leukoencephalopathy
(B) Animals, specifically pigs, horses, and fowl (B) Subacute sclerosing panencephalitis
(C) Soil, especially in the tropics (C) Creutzfeldt-Jakob disease
(D) Sewage (D) Kuru
226. The role of an infectious agent in the pathogenesis of kuru was 233. You think your patient may be in the “window period” of
BEST demonstrated by which one of the following observations? hepatitis B virus (HBV) infection because his blood tests for
(A) A 16-fold rise in antibody titer to the agent was observed. HBs antigen and anti-HBs antibody are negative. Which one of
(B) The viral genome was isolated from infected neurons. the following additional tests is MOST useful to establish that
(C) Electron micrographs of the brains of infected individuals he has been infected with HBV and is in the “window period.”
demonstrated intracellular structures resembling paramyxo- (A) HBe antigen
virus nucleocapsids. (B) Anti-HBc antibody
(D) The disease was serially transmitted to experimental animals. (C) Anti-HBe antibody
227. A 64-year-old man with chronic lymphatic leukemia develops (D) Delta antigen
progressive deterioration of mental and neuromuscular func- 234. Which one of the following is the MOST reasonable explanation
tion. At autopsy the brain shows enlarged oligodendrocytes for the ability of hepatitis B virus to cause chronic infection?
whose nuclei contain naked, icosahedral virus particles. The (A) Infection does not elicit the production of antibody.
MOST likely diagnosis is: (B) The liver is an “immunologically sheltered” site.
(A) Herpes encephalitis (C) Viral DNA can persist within the host cell.
(B) Creutzfeldt-Jakob disease (D) Many humans are immunologically tolerant to HBs antigen.
(C) Subacute sclerosing panencephalitis 235. The routine screening of transfused blood has greatly reduced
(D) Progressive multifocal leukoencephalopathy the problem of post-transfusion hepatitis. For which one of the
(E) Rabies following viruses has screening eliminated a large number of
228. A 20-year-old man, who for many years had received daily cases of post-transfusion hepatitis?
injections of growth hormone prepared from human pitu- (A) Hepatitis A virus
itary glands, develops ataxia, slurred speech, and dementia. At (B) Hepatitis C virus
autopsy the brain shows widespread neuronal degeneration, a (C) Cytomegalovirus
spongy appearance due to many vacuoles between the cells, no (D) Epstein–Barr virus
inflammation, and no evidence of virus particles. The MOST 236. A 35-year-old man addicted to intravenous drugs has been a
likely diagnosis is: carrier of HBs antigen for 10 years. He suddenly develops acute
(A) Herpes encephalitis fulminant hepatitis and dies within 10 days. Which one of the fol-
(B) Creutzfeldt-Jakob disease lowing laboratory tests would contribute MOST to a diagnosis?
(C) Subacute sclerosing panencephalitis (A) Anti-HBs antibody
(D) Progressive multifocal leukoencephalopathy (B) HBe antigen
(E) Rabies (C) Anti-HBc antibody
229. A 24-year-old woman has had fever and a sore throat for the (D) Anti-delta virus antibody
past week. Moderately severe pharyngitis and bilateral cervical 237. Which one of the following is the BEST evidence on which to
lymphadenopathy are seen on physical examination. Which one base a decisive diagnosis of acute mumps disease?
of the following viruses is LEAST likely to cause this picture?
(A) A positive skin test
(A) Norovirus (B) A fourfold rise in antibody titer to mumps antigen
(B) Adenovirus (C) A history of exposure to a child with mumps
(C) Coxsackie virus (D) Orchitis in young adult male
(D) Epstein–Barr virus
238. Varicella-zoster virus and herpes simplex virus share many
230. Scrapie and kuru possess all of the following characteristics characteristics. Which one of the following characteristics is
EXCEPT: NOT shared?
(A) A histologic picture of spongiform encephalopathy (A) Inapparent disease, manifested only by virus shedding, is
(B) Transmissibility to animals associated with a long incubation common
period (B) Persistence of latent virus after recovery from acute disease
(C) Slowly progressive deterioration of brain function (C) Vesicular rash
(D) Prominent intranuclear inclusions in oligodendrocytes (D) Linear, double-stranded DNA genome
231. Each of the following statements concerning subacute scleros- 239. Herpes simplex virus and cytomegalovirus share many features.
ing panencephalitis is correct EXCEPT: Which one of the following features is LEAST likely to be shared?
(A) Immunosuppression is a frequent predisposing factor. (A) Important cause of morbidity and mortality in the newborn
(B) Aggregates of helical nucleocapsids are found in infected (B) Congenital abnormalities due to transplacental passage
cells. (C) Important cause of serious disease in immunosuppressed
(C) High titers of measles antibody are found in cerebrospinal individuals
fluid. (D) Mild or inapparent infection
(D) Gradual progressive deterioration of brain function occurs.

mebooksfree.com
PART XIII USMLE (National Board) Practice Questions 725

240. The eradication of smallpox was facilitated by several features 247. Herpes simplex virus type 1 (HSV-1) is distinct from HSV-2 in
of the virus. Which one of the following contributed LEAST to several different ways. Which one of the following is the LEAST
eradication? accurate statement?
(A) It has one antigenic type. (A) HSV-1 causes lesions above the umbilicus more frequently
(B) Inapparent infection is rare. than HSV-2 does.
(C) Administration of live vaccine reliably induces immunity. (B) Infection by HSV-1 is not associated with any tumors in
(D) It multiplies in the cytoplasm of infected cells. humans.
241. Which one of the following statements concerning infectious (C) Antiserum to HSV-1 neutralizes HSV-1 much more effec-
mononucleosis is the MOST accurate? tively than HSV-2.
(A) Multinucleated giant cells are found in the skin lesions. (D) HSV-1 causes frequent recurrences, whereas HSV-2 infec-
(B) Infected T lymphocytes are abundant in peripheral blood. tion rarely recurs.
(C) Isolation of virus is necessary to confirm the diagnosis. 248. Which one of the following statements about the src gene and
(D) Infectious mononucleosis is transmitted by virus in saliva. src protein of Rous sarcoma virus is INCORRECT?
242. Which one of the following statements about genital herpes is (A) The src protein inactivates a protein encoded by p53, a tumor
LEAST accurate? suppressor gene.
(A) Acyclovir reduces the number of recurrent disease episodes (B) The src protein is a protein kinase that preferentially phos-
by eradicating latently infected cells. phorylates tyrosine in cellular proteins.
(B) Genital herpes can be transmitted in the absence of apparent (C) The src protein is required to maintain neoplastic transfor-
lesions. mation of infected cells.
(C) Multinucleated giant cells with intranuclear inclusions are (D) The viral src gene is derived from a cellular gene found in
found in the lesions. many vertebrate species.
(D) Initial disease episodes are generally more severe than recur- 249. Each of the following statements supports the idea that cel-
rent episodes. lular proto-oncogenes participate in human carcinogenesis
243. There are several influenza vaccines administered in the United EXCEPT:
States. Regarding these vaccines, which one of the following (A) The c-abl gene is rearranged on the Philadelphia chromo-
statements is LEAST accurate? some in myeloid leukemias and encodes a protein with
(A) One of the vaccines contains purified peptide subunits of increased tyrosine kinase activity.
neuraminidase produced in yeast. (B) The N-myc gene is amplified as much as 100-fold in many
(B) One of the vaccines is an inactivated vaccine consisting of advanced cases of neuroblastoma.
formaldehyde-treated influenza virions. (C) The receptor for platelet-derived growth factor is a trans-
(C) One of the vaccines contains a temperature-sensitive mutant membrane protein that exhibits tyrosine kinase activity.
of influenza virus that replicates in the nose but not in the (D) The c-Ha-ras gene is mutated at specific codons in several
lungs. types of human cancer.
(D) Influenza vaccines contain influenza A and B strains but not 250. Each of the following statements concerning human immuno-
C strains. deficiency virus (HIV) is correct EXCEPT:
244. Which of the following is the MOST common lower respiratory (A) Screening tests for antibodies are useful to prevent transmis-
pathogen in infants? sion of HIV through transfused blood.
(A) Respiratory syncytial virus (B) The opportunistic infections seen in AIDS are primarily the
(B) Adenovirus result of a loss of cell-mediated immunity.
(C) Rhinovirus (C) Zidovudine (azidothymidine) inhibits the RNA-dependent
(D) Coxsackie virus DNA polymerase.
(D) The presence of circulating antibodies that neutralize HIV is
245. Which of the following conditions is LEAST likely to be caused
evidence that an individual is protected against HIV-induced
by adenoviruses?
disease.
(A) Conjunctivitis
251. Which one of the following statements concerning viral menin-
(B) Pneumonia
gitis and viral encephalitis is CORRECT?
(C) Pharyngitis
(D) Glomerulonephritis (A) Herpes simplex virus type 2 is the leading cause of viral
meningitis.
246. Regarding the serologic diagnosis of infectious mononucleosis,
(B) Herpes simplex virus type 1 is an important cause of viral
which one of the following is CORRECT?
encephalitis.
(A) A heterophil antibody is formed that reacts with a capsid (C) The spinal fluid protein is usually decreased in viral
protein of Epstein–Barr virus. meningitis.
(B) A heterophil antibody is formed that agglutinates sheep or (D) The diagnosis of viral meningitis can be made by using the
horse red blood cells. India ink stain on a sample of spinal fluid.
(C) A heterophil antigen occurs that cross-reacts with Proteus
OX19 strains.
(D) A heterophil antigen occurs following infection with
cytomegalovirus.

mebooksfree.com
726 PART XIII USMLE (National Board) Practice Questions

252. Each of the following statements is correct EXCEPT: (C) In a 6-year-old child, rubella is a mild, self-limited disease
(A) Coxsackie viruses are enteroviruses and can replicate in both with few complications.
the respiratory and gastrointestinal tracts. (D) Acyclovir is effective in the treatment of congenital rubella
(B) Influenza viruses have multiple serotypes based on hemag- syndrome.
glutinin and neuraminidase proteins located on the envelope 258. Each of the following statements concerning rabies and rabies
surface. virus is correct EXCEPT:
(C) Flaviviruses are RNA enveloped viruses that replicate in ani- (A) The virus has a lipoprotein envelope and single-stranded
mals as well as humans. RNA as its genome.
(D) Adenoviruses are RNA enveloped viruses that are an impor- (B) The virus has a single antigenic type (serotype).
tant cause of sexually transmitted disease. (C) In the United States, dogs are the most common reservoir.
253. Which one of the following statements concerning the preven- (D) The incubation period is usually long (several weeks) rather
tion of viral disease is CORRECT? than short (several days).
(A) Adenovirus vaccine contains purified penton fibers and is 259. Each of the following statements concerning arboviruses is
usually given to children in conjunction with polio vaccine. correct EXCEPT:
(B) Coxsackie virus vaccine contains live virus that induces IgA, (A) The pathogenesis of dengue hemorrhagic shock syndrome is
which prevents reinfection by homologous serotypes. associated with the heterotypic anamnestic response.
(C) Flavivirus immunization consists of hyperimmune serum (B) Wild birds are the reservoir for encephalitis viruses but not
plus a vaccine consisting of subunits containing the surface for yellow fever virus.
glycoprotein. (C) Ticks are the main mode of transmission for both encephali-
(D) One of the influenza virus vaccines contains killed virus that tis viruses and yellow fever virus.
induces neutralizing antibody directed against the (D) There is a live, attenuated vaccine that effectively prevents
hemagglutinin. yellow fever.
254. Each of the following statements concerning hepatitis C virus 260. Each of the following statements concerning rhinoviruses is
(HCV) and hepatitis D virus (HDV) is correct EXCEPT: correct EXCEPT:
(A) HCV is an RNA virus that causes post-transfusion hepatitis. (A) Rhinoviruses are picornaviruses (i.e., small, nonenveloped
(B) HDV is a defective virus that can replicate only in a cell that viruses with an RNA genome).
is also infected with hepatitis B virus. (B) Rhinoviruses are an important cause of lower respiratory
(C) HDV is transmitted primarily by the fecal–oral route. tract infections, especially in patients with chronic obstruc-
(D) People infected with HCV commonly become chronic carri- tive pulmonary disease.
ers of HCV and are predisposed to hepatocellular (C) Rhinoviruses do not infect the gastrointestinal tract because
carcinoma. they are inactivated by the acid pH in the stomach.
255. Each of the following statements concerning measles virus is (D) There is no vaccine against rhinoviruses because they have
correct EXCEPT: too many antigenic types.
(A) Measles virus is an enveloped virus with a single-stranded 261. Each of the following statements concerning herpes simplex
RNA genome. virus type 2 (HSV-2) is correct EXCEPT:
(B) One of the important complications of measles is (A) Primary infection with HSV-2 does not confer immunity to
encephalitis. primary infection with HSV-1.
(C) The initial site of measles virus replication is the upper respi- (B) HSV-2 causes vesicular lesions, typically in the genital area.
ratory tract, from which it spreads via the blood to the skin. (C) HSV-2 can cause alterations of the cell membrane, leading to
(D) Latent infection by measles virus can be explained by the cell fusion and the formation of multinucleated giant cells.
integration of provirus into the host cell DNA. (D) Recurrent disease episodes due to reactivation of latent
256. Each of the following statements concerning measles vaccine is HSV-2 are usually more severe than the primary episode.
correct EXCEPT: 262. Each of the following statements concerning Epstein–Barr virus
(A) The vaccine contains live, attenuated virus. is correct EXCEPT:
(B) The vaccine should not be given at the same time as the (A) Many infections are mild or inapparent.
mumps vaccine because the immune system cannot respond (B) The earlier in life primary infection is acquired, the more
to two viral antigens given at the same time. likely the typical picture of infectious mononucleosis will be
(C) Virus in the vaccine contains only one serotype. manifest.
(D) The vaccine should not be given prior to 15 months of age (C) Latently infected lymphocytes regularly persist following an
because maternal antibodies can prevent an immune acute episode of infection.
response. (D) Infection confers immunity against second episodes of infec-
257. Each of the following statements concerning rubella is correct tious mononucleosis.
EXCEPT: 263. Each of the following statements regarding rotaviruses is correct
(A) Congenital abnormalities occur primarily when a pregnant EXCEPT:
woman is infected during the first trimester. (A) The rotavirus vaccine contains recombinant RNA polymer-
(B) Women who say that they have never had rubella can, nev- ase as the immunogen.
ertheless, have neutralizing antibody in their serum. (B) Rotaviruses are a leading cause of diarrhea in young
children.

mebooksfree.com
PART XIII USMLE (National Board) Practice Questions 727

(C) Rotaviruses are transmitted primarily by the fecal–oral route. (C) A skin test is available to determine prior exposure to the
(D) Rotaviruses belong to the reovirus family, which have a virus.
double-stranded, segmented RNA genome. (D) Oseltamivir is an effective preventive agent.
264. Each of the following statements concerning the antigenicity of 270. Each of the following statements concerning yellow fever is correct
influenza A virus is correct EXCEPT: EXCEPT:
(A) Antigenic shifts, which represent major changes in anti- (A) Yellow fever virus is transmitted by the Aedes aegypti mos-
genicity, occur infrequently and are due to the reassortment quito in the urban form of yellow fever.
of segments of the viral genome. (B) Infection by yellow fever virus causes significant damage to
(B) Antigenic shifts affect both the hemagglutinin and the hepatocytes.
neuraminidase. (C) Nonhuman primates in the jungle are a major reservoir of
(C) The worldwide epidemics caused by influenza A virus are yellow fever virus.
due to antigenic shifts. (D) Acyclovir is an effective treatment for yellow fever.
(D) The protein involved in antigenic drift is primarily the inter- 271. Which one of the following statements concerning mumps is
nal ribonucleoprotein. CORRECT?
265. Each of the following statements concerning adenoviruses is (A) Although the salivary glands are the most obvious sites of
correct EXCEPT: infection, the testes, ovaries, and pancreas can be involved as
(A) Adenoviruses are composed of a double-stranded DNA well.
genome and a capsid without an envelope. (B) Because there is no vaccine against mumps, passive immuni-
(B) Adenoviruses cause both sore throat and pneumonia. zation is the only means of preventing the disease.
(C) Adenoviruses have only one serologic type. (C) The diagnosis of mumps is made on clinical grounds because
(D) Adenoviruses are implicated as a cause of tumors in animals the virus cannot be grown in cell culture and serologic tests
but not humans. are inaccurate.
266. Each of the following statements concerning the prevention of (D) Second episodes of mumps can occur because there are two
viral respiratory tract disease is correct EXCEPT: serotypes of the virus and protection is type-specific.
(A) To prevent disease caused by adenoviruses, a live enteric- 272. Many of the oncogenic retroviruses carry oncogenes closely
coated vaccine that causes asymptomatic enteric infection is related to normal cellular genes, called proto-oncogenes. Which
used in the military. one of the following statements concerning proto-oncogenes is
(B) To prevent disease caused by influenza A virus, an inacti- INCORRECT?
vated vaccine is available for the civilian population. (A) Several proto-oncogenes have been found in mutant form in
(C) There is no vaccine available against respiratory syncytial human cancers that lack evidence for viral etiology.
virus. (B) Several viral oncogenes and their progenitor proto-oncogenes
(D) To prevent disease caused by rhinoviruses, a vaccine contain- encode protein kinases specific for tyrosine.
ing purified capsid proteins is used. (C) Some proto-oncogenes encode cellular growth factors and
267. Each of the following statements concerning herpesvirus receptors for growth factors.
latency is correct EXCEPT: (D) Proto-oncogenes are closely related to transposons found in
(A) Exogenous stimuli can cause reactivation of herpesvirus bacteria.
replication in latently-infected cells. 273. Each of the following statements concerning human immuno-
(B) During latency, antiviral antibody is not demonstrable in the deficiency virus is correct EXCEPT:
sera of infected individuals. (A) The CD4 protein on the T-cell surface is one of the receptors
(C) Reactivation of latent herpesviruses is more common in for the virus.
patients with impaired cell-mediated immunity than in (B) There is appreciable antigenic diversity in the envelope gly-
immunocompetent patients. coprotein of the virus.
(D) Herpesvirus genome DNA persists in latently infected cells. (C) One of the viral genes codes for a protein that augments the
268. Each of the following statements concerning rhinoviruses is activity of the viral transcriptional promoter.
correct EXCEPT: (D) A major problem with testing for antibody to the virus is its
(A) Rhinoviruses are the most common cause of the common cross-reactivity with human T-cell lymphoma virus type I.
cold. 274. Each of the following statements concerning human immuno-
(B) Rhinoviruses grow better at 33°C than at 37°C; hence they deficiency virus (HIV) is correct EXCEPT:
tend to cause disease in the upper respiratory tract rather (A) Patients infected with HIV typically form antibodies against
than the lower respiratory tract. both the envelope glycoproteins (gp120 and gp41) and the
(C) Rhinoviruses are members of the picornaviruses family and internal group-specific antigen (p24).
hence resemble poliovirus in their structure and replication. (B) HIV probably arose as an endogenous virus of humans
(D) The immunity provided by the rhinovirus vaccine is excel- because HIV proviral DNA is found in the DNA of certain
lent because there is only one serotype. normal human cells.
269. Which one of the following statements concerning poliovirus (C) Transmission of HIV occurs primarily by the transfer of
infection is CORRECT? blood or semen in adults, and neonates can be infected at the
(A) Congenital infection of the fetus is an important time of delivery.
complication. (D) The Western blot test is more specific for HIV infection than
(B) The virus replicates extensively in the gastrointestinal tract. the ELISA is.

mebooksfree.com
728 PART XIII USMLE (National Board) Practice Questions

275. Each of the following statements concerning hepatitis A virus Questions 280–284
(HAV) is correct EXCEPT:
(A) Bronchiolitis
(A) The hepatitis A vaccine contains inactivated HAV as the (B) Meningitis
immunogen. (C) Pharyngitis
(B) HAV commonly causes asymptomatic infection in children. (D) Shingles
(C) The diagnosis of hepatitis A is usually made by isolating (E) Subacute sclerosing panencephalitis
HAV in cell culture.
280. Adenovirus
(D) Gamma globulin is used to prevent hepatitis A in exposed
281. Measles virus
persons.
282. Respiratory syncytial virus
Answers (Questions 212–275) 283. Coxsackie virus
284. Varicella-zoster virus
212. (C) 225. (B) 238. (A) 251. (B) 264. (D)
213. (B) 226. (D) 239. (B) 252. (D) 265. (C) Questions 285–289
214. (B) 227. (D) 240. (D) 253. (D) 266. (D) (A) Adenovirus
215. (D) 228. (B) 241. (D) 254. (C) 267. (B) (B) Parainfluenza virus
216. (C) 229. (A) 242. (A) 255. (D) 268. (D) (C) Rhinovirus
(D) Coxsackie virus
217. (B) 230. (D) 243. (A) 256. (B) 269. (B)
(E) Epstein–Barr virus
218. (A) 231. (A) 244. (A) 257. (D) 270. (D)
285. Causes myocarditis and pleurodynia
219. (D) 232. (A) 245. (D) 258. (C) 271. (A) 286. Grows better at 33°C than 37°C
220. (A) 233. (B) 246. (B) 259. (C) 272. (D) 287. Causes tumors in laboratory rodents
221. (D) 234. (C) 247. (D) 260. (B) 273. (D) 288. Causes croup in young children
222. (C) 235. (B) 248. (A) 261. (D) 274. (B) 289. Causes infectious mononucleosis
223. (C) 236. (D) 249. (C) 262. (B) 275. (C) Questions 290–294
224. (D) 237. (B) 250. (D) 263. (A)
(A) Hepatitis C virus
(B) Cytomegalovirus
DIRECTIONS (Questions 276–294): Select the ONE lettered option (C) Human papillomavirus
that is MOST closely associated with the numbered items. Each lettered (D) Dengue virus
option may be selected once, more than once, or not at all. (E) St. Louis encephalitis virus
290. It is implicated as the cause of carcinoma of the cervix.
Questions 276–279 291. Wild birds are an important reservoir.
(A) Yellow fever virus 292. It is an important cause of pneumonia in immunocompromised
(B) Rabies virus patients.
(C) Rotavirus 293. Donated blood containing antibody to this RNA virus should
(D) Rubella virus not be used for transfusion.
(E) Rhinovirus 294. It causes a hemorrhagic fever that can be life-threatening.
276. Diarrhea Answers (Questions 276–294)
277. Jaundice
276. (C) 280. (C) 284. (D) 288. (B) 292. (B)
278. Congenital abnormalities
277. (A) 281. (E) 285. (D) 289. (E) 293. (A)
279. Encephalitis
278. (D) 282. (A) 286. (C) 290. (C) 294. (D)
279. (B) 283. (B) 287. (A) 291. (E)

MYCOLOGY
DIRECTIONS (Questions 295–317): Select the ONE lettered answer
Which one of the following statements is LEAST accurate or
that is BEST in each question.
appropriate?
295. Which one of the following fungi is MOST likely to be found (A) A Gram stain of the discharge should reveal budding yeasts.
within reticuloendothelial cells? (B) Culture of the discharge on Sabouraud’s agar should produce
(A) Histoplasma capsulatum a white mycelium with aerial conidia.
(B) Candida albicans (C) The clinical laboratory can use germ tube formation to iden-
(C) Cryptococcus neoformans tify the isolate as C. albicans.
(D) Sporothrix schenckii (D) Antibiotics predispose to Candida vaginitis by killing the
296. Your patient is a woman with a vaginal discharge. You suspect, normal flora lactobacilli that keep the vaginal pH low.
on clinical grounds, that it may be due to Candida albicans.

mebooksfree.com
PART XIII USMLE (National Board) Practice Questions 729

297. You have made a clinical diagnosis of meningitis in a (C) Adherence of the organism to perspiration-moist skin
50-year-old immunocompromised woman. A latex agglu- (D) Fecal–oral transmission
tination test on the spinal fluid for capsular polysaccharide 305. Aspergillosis is recognized in tissue by the presence of:
antigen is positive. Of the following organisms, which one (A) Budding cells
is the MOST likely cause? (B) Septate hyphae
(A) Histoplasma capsulatum (C) Metachromatic granules
(B) Cryptococcus neoformans (D) Pseudohyphae
(C) Aspergillus fumigatus 306. Which one of the following is NOT a characteristic of
(D) Candida albicans histoplasmosis?
298. Fungi often colonize lesions due to other causes. Which (A) Person-to-person transmission
one of the following is LEAST likely to be present as a (B) Specific geographic distribution
colonizer? (C) Yeasts in the tissue
(A) Aspergillus (D) Mycelial phase in the soil
(B) Mucor 307. Each of the following statements concerning mucormyco-
(C) Sporothrix sis is correct EXCEPT:
(D) Candida
(A) The fungi that cause mucormycosis are transmitted by
299. Your patient complains of an “itching rash” on her abdomen. airborne asexual spores.
On examination, you find that the lesions are red, circular, (B) Tissue sections from a patient with mucormycosis show
with a vesiculated border and a healing central area. You budding yeasts.
suspect tinea corporis. Of the following choices, the MOST (C) Hyphae typically invade blood vessels and cause necro-
appropriate laboratory procedure to make the diagnosis is a: sis of tissue.
(A) Potassium hydroxide mount of skin scrapings (D) Ketoacidosis in diabetic patients is a predisposing fac-
(B) Giemsa stain for multinucleated giant cells tor to mucormycosis.
(C) Fluorescent antibody stain of the vesicle fluid 308. Each of the following statements concerning fungi is correct
(D) Fourfold rise in antibody titer against the organism EXCEPT:
300. Each of the following statements concerning Cryptococcus (A) Yeasts are fungi that reproduce by budding.
neoformans is correct EXCEPT: (B) Molds are fungi that have elongated filaments called
(A) Its natural habitat is the soil, especially associated with hyphae.
pigeon feces. (C) Thermally dimorphic fungi exist as yeasts at 37°C and
(B) Pathogenesis is related primarily to the production of as molds at 25°C.
exotoxin A. (D) Both yeasts and molds have a cell wall made of
(C) Budding yeasts are found in the lesions. peptidoglycan.
(D) The initial site of infection is usually the lung. 309. Each of the following statements concerning yeasts is cor-
301. A woman who pricked her finger while pruning some rose rect EXCEPT:
bushes develops a local pustule that progresses to an ulcer. (A) Yeasts have chitin in their cell walls and ergosterol in
Several nodules then develop along the local lymphatic their cell membranes.
drainage. The MOST likely agent is: (B) Yeasts form ascospores when they invade tissue.
(A) Cryptococcus neoformans (C) Yeasts have eukaryotic nuclei and contain mitochon-
(B) Candida albicans dria in their cytoplasm.
(C) Sporothrix schenckii (D) Yeasts produce neither endotoxin nor exotoxins.
(D) Aspergillus fumigatus 310. Each of the following statements concerning fungi and
302. Several fungi are associated with disease in immuno- protozoa is correct EXCEPT:
compromised patients. Which one of the following is the (A) Both fungi and protozoa are eukaryotic organisms.
LEAST frequently associated? (B) Fungi possess a cell wall, whereas protozoa do not.
(A) Cryptococcus neoformans (C) Both fungi and protozoa use flagella as their organ of
(B) Aspergillus fumigatus motility.
(C) Malassezia furfur (D) Both fungi and protozoa generate energy in mitochondria.
(D) Mucor species 311. You suspect that your patient’s disease may be caused
303. Fungal cells that reproduce by budding are seen in the by Cryptococcus neoformans. Which one of the follow-
infected tissues of patients with: ing findings would be MOST useful in establishing the
(A) Candidiasis, cryptococcosis, and sporotrichosis diagnosis?
(B) Mycetoma, candidiasis, and mucormycosis (A) A positive heterophil agglutination test for the presence
(C) Tinea corporis, tinea unguium, and tinea versicolor of antigen
(D) Sporotrichosis, mycetoma, and aspergillosis (B) A history of recent travel in the Mississippi River valley
304. Infection by a dermatophyte is MOST often associated with: area
(A) Intravenous drug abuse (C) The finding of encapsulated budding cells in spinal fluid
(B) Inhalation of the organism from contaminated bird (D) Recovery of an acid-fast organism from the patient’s
feces sputum

mebooksfree.com
730 PART XIII USMLE (National Board) Practice Questions

312. Each of the following statements concerning Candida albi- (C) Colonization of tuberculous cavities in the lung
cans is correct EXCEPT: (D) Thrush
(A) Candida albicans is a budding yeast that forms pseudo-
hyphae when it invades tissue. Answers (Questions 295–317)
(B) Candida albicans is transmitted primarily by respira-
295. (A) 300. (B) 305. (B) 310. (C) 315. (C)
tory aerosol.
296. (B) 301. (C) 306. (A) 311. (C) 316. (C)
(C) Candida albicans causes thrush.
297. (B) 302. (C) 307. (B) 312. (B) 317. (D)
(D) Impaired cell-mediated immunity is an important pre-
298. (C) 303. (A) 308. (D) 313. (D)
disposing factor to disease.
299. (A) 304. (C) 309. (B) 314. (D)
313. Each of the following statements concerning Coccidioides
immitis is correct EXCEPT:
(A) The mycelial phase of the organism grows primarily in
the soil, which is its natural habitat. DIRECTIONS (Questions 318–325): Select the ONE lettered option
(B) In the body, spherules containing endospores are that is MOST closely associated with the numbered items. Each lettered
formed. option may be selected once, more than once, or not at all.
(C) A rising titer of complement-fixing antibody indicates
disseminated disease. Questions 318–321
(D) Most infections are symptomatic and require treatment (A) Histoplasma capsulatum
with amphotericin B. (B) Candida albicans
314. Each of the following statements concerning Histoplasma (C) Aspergillus fumigatus
capsulatum is correct EXCEPT: (D) Sporothrix schenckii
(A) The natural habitat of H. capsulatum is the soil, where 318. A budding yeast that is a member of the normal flora of the
it grows as a mold. vagina
(B) Histoplasma capsulatum is transmitted by airborne 319. A dimorphic organism that is transmitted by trauma to the skin
conidia, and its initial site of infection is the lung. 320. A dimorphic fungus that typically is acquired by inhalation of
(C) Within the body, H. capsulatum grows primarily intra- asexual spores
cellularly within macrophages. 321. A mold that causes pneumonia in immunocompromised
(D) Passive immunity in the form of high titer antibodies patients
should be given to those known to be exposed.
Questions 322–325
315. Each of the following statements concerning infection
caused by Coccidioides immitis is correct EXCEPT: (A) Coccidioides immitis
(A) Coccidioides immitis is a dimorphic fungus. (B) Rhizopus nigricans
(B) Coccidioides immitis is acquired by inhalation of (C) Blastomyces dermatitidis
arthrospores. (D) Cryptococcus neoformans
(C) More than 50% of clinical isolates are resistant to 322. A yeast acquired by inhalation that causes meningitis primarily
amphotericin B. in immunocompromised patients
(D) Infection occurs primarily in the southwestern states 323. A mold that invades blood vessels primarily in patients with
and California. diabetic ketoacidosis
316. Each of the following statements concerning Blastomyces 324. A dimorphic fungus that is acquired by inhalation by people
dermatitidis is correct EXCEPT: living in certain areas of the southwestern states in the United
States
(A) Blastomyces dermatitidis grows as a mold in the soil in
325. A dimorphic fungus that causes granulomatous skin lesions in
North America.
people living in many areas of North America
(B) Blastomyces dermatitidis is a dimorphic fungus that
forms yeast cells in tissue.
(C) Blastomyces dermatitidis infection is commonly diag- Answers (Questions 318–325)
nosed by serologic tests because it does not grow in
culture. 318. (B) 320. (A) 322. (D) 324. (A)
(D) Blastomyces dermatitidis causes granulomatous skin 319. (D) 321. (C) 323. (B) 325. (C)
lesions.
317. Aspergillus fumigatus can be involved in a variety of clini-
cal conditions. Which one of the following is LEAST likely
to occur?
(A) Tissue invasion in immunocompromised host
(B) Allergy following inhalation of airborne particles of the
fungus

mebooksfree.com
PART XIII USMLE (National Board) Practice Questions 731

PARASITOLOGY
DIRECTIONS (Questions 326–352): Select the ONE lettered answer 335. Each of the following parasites is transmitted by eating inad-
that is BEST in each question. equately cooked fish or seafood EXCEPT:
326. Children at day care centers in the United States have a high rate (A) Diphyllobothrium latum
of infection with which one of the following? (B) Ancylostoma duodenale
(C) Paragonimus westermani
(A) Ascaris lumbricoides
(D) Clonorchis sinensis
(B) Entamoeba histolytica
(C) Enterobius vermicularis 336. Laboratory diagnosis of a patient with a suspected liver abscess
(D) Necator americanus due to Entamoeba histolytica should include:
327. The main anatomic location of Schistosoma mansoni adult (A) Stool examination and indirect hemagglutination test
worms is: (B) Stool examination and blood smear
(C) Indirect hemagglutination test and skin test
(A) Lung alveoli
(D) Xenodiagnosis and string test
(B) Intestinal venules
(C) Renal tubules 337. Each of the following statements concerning Toxoplasma gondii
(D) Bone marrow is correct EXCEPT:
328. In malaria, the form of plasmodia that is transmitted from mos- (A) Toxoplasma gondii can be transmitted across the placenta to
quito to human is the: the fetus.
(B) Toxoplasma gondii can be transmitted by cat feces.
(A) Sporozoite
(C) Toxoplasma gondii can cause encephalitis in immunocom-
(B) Gametocyte
promised patients.
(C) Merozoite
(D) Toxoplasma gondii can be diagnosed by finding trophozoites
(D) Hypnozoite
in the stool.
329. Which one of the following protozoa primarily infects
338. Each of the following statements concerning Giardia lamblia is
macrophages?
correct EXCEPT:
(A) Plasmodium vivax
(A) Giardia lamblia has both a trophozoite and a cyst stage in its
(B) Leishmania donovani
life cycle.
(C) Trypanosoma cruzi
(B) Giardia lamblia is transmitted by the fecal–oral route from
(D) Trichomonas vaginalis
both human and animal sources.
330. Each of the following parasites has an intermediate host as part (C) Giardia lamblia causes hemolytic anemia.
of its life cycle EXCEPT: (D) Giardia lamblia can be diagnosed by the string test in which
(A) Trichomonas vaginalis a weighted string is swallowed and passes into the upper GI
(B) Taenia solium tract.
(C) Echinococcus granulosus 339. Each of the following statements concerning malaria is correct
(D) Toxoplasma gondii EXCEPT:
331. Each of the following parasites passes through the lung during (A) The female Anopheles mosquito is the vector.
human infection EXCEPT: (B) Early in infection, sporozoites enter hepatocytes.
(A) Strongyloides stercoralis (C) Release of merozoites from red blood cells causes periodic
(B) Necator americanus fever and chills.
(C) Enterobius vermicularis (D) The principal site of gametocyte formation is the human
(D) Ascaris lumbricoides gastrointestinal tract.
332. Each of the following parasites is transmitted by flies EXCEPT: 340. Each of the following statements concerning Trichomonas vagi-
(A) Schistosoma mansoni nalis is correct EXCEPT:
(B) Onchocerca volvulus (A) Trichomonas vaginalis is transmitted sexually.
(C) Trypanosoma gambiense (B) Trichomonas vaginalis can be diagnosed by visualizing the
(D) Loa loa trophozoite.
333. Each of the following parasites is transmitted by mosquitoes (C) Trichomonas vaginalis can be treated effectively with
EXCEPT: metronidazole.
(A) Leishmania donovani (D) Trichomonas vaginalis causes bloody diarrhea.
(B) Wuchereria bancrofti 341. Which one of the following agents can be used to prevent
(C) Plasmodium vivax malaria?
(D) Plasmodium falciparum (A) Mebendazole
334. Pigs or dogs are the source of human infection by each of the (B) Chloroquine
following parasites EXCEPT: (C) Inactivated vaccine
(A) Echinococcus granulosus (D) Praziquantel
(B) Taenia solium
(C) Ascaris lumbricoides
(D) Trichinella spiralis

mebooksfree.com
732 PART XIII USMLE (National Board) Practice Questions

342. Each of the following statements concerning Pneumocystis cari- 349. Each of the following statements concerning hookworm infec-
nii is correct EXCEPT: tion is correct EXCEPT:
(A) Pneumocystis carinii infections primarily involve the respira- (A) Hookworm infection can cause anemia.
tory tract. (B) Hookworm infection is acquired by humans when filariform
(B) Pneumocystis carinii can be diagnosed by seeing cysts in larvae penetrate the skin.
tissue. (C) Hookworm infection is caused by Necator americanus.
(C) Pneumocystis carinii infections are symptomatic primarily in (D) Hookworm infection can be diagnosed by finding the tro-
immunocompromised patients. phozoite in the stool.
(D) Pneumocystis carinii symptomatic infections can be pre- 350. Each of the following statements concerning Ascaris lumbricoi-
vented by administering penicillin orally. des is correct EXCEPT:
343. Each of the following statements concerning Trypanosoma (A) Ascaris lumbricoides is one of the largest nematodes.
cruzi is correct EXCEPT: (B) Ascaris lumbricoides is transmitted by ingestion of eggs.
(A) Trypanosoma cruzi is transmitted by the reduviid bug. (C) Both dogs and cats are intermediate hosts of A. lumbricoides.
(B) Trypanosoma cruzi occurs primarily in tropical Africa. (D) Ascaris lumbricoides can cause pneumonia.
(C) Trypanosoma cruzi can be diagnosed by seeing amastigotes 351. Each of the following statements concerning Strongyloides
in a bone marrow aspirate. stercoralis is correct EXCEPT:
(D) Trypanosoma cruzi typically affects heart muscle, leading to (A) Strongyloides stercoralis is acquired by ingestion of eggs.
cardiac failure. (B) Strongyloides stercoralis undergoes a free-living life cycle in
344. Each of the following statements concerning sleeping sickness soil.
is correct EXCEPT: (C) Migrating larvae of S. stercoralis induce a marked
(A) Sleeping sickness is caused by a trypanosome. eosinophilia.
(B) Sleeping sickness is transmitted by tsetse flies. (D) Strongyloides stercoralis produces filariform larvae.
(C) Sleeping sickness can be diagnosed by finding eggs in the 352. Each of the following statements concerning trichinosis is cor-
stool. rect EXCEPT:
(D) Sleeping sickness occurs primarily in tropical Africa. (A) Trichinosis is acquired by eating undercooked pork.
345. Each of the following statements concerning kala-azar is correct (B) Trichinosis is caused by a protozoan that has both a tropho-
EXCEPT: zoite and a cyst stage in its life cycle.
(A) Kala-azar is caused by Leishmania donovani. (C) Trichinosis can be diagnosed by seeing cysts in muscle
(B) Kala-azar is transmitted by the bite of sandflies. biopsy specimens.
(C) Kala-azar occurs primarily in rural Latin America. (D) Eosinophilia is a prominent finding.
(D) Kala-azar can be diagnosed by finding amastigotes in bone
marrow. Answers (Questions 326–352)
346. Each of the following statements concerning Diphyllobothrium 326. (C) 332. (A) 338. (C) 344. (C) 350. (C)
latum is correct EXCEPT: 327. (B) 333. (A) 339. (D) 345. (C) 351. (A)
(A) Diphyllobothrium latum is transmitted by undercooked fish. 328. (A) 334. (C) 340. (D) 346. (D) 352. (B)
(B) Diphyllobothrium latum has operculated eggs. 329. (B) 335. (B) 341. (B) 347. (D)
(C) Diphyllobothrium latum causes a megaloblastic anemia due 330. (A) 336. (A) 342. (D) 348. (C)
to vitamin B12 deficiency. 331. (C) 337. (D) 343. (B) 349. (D)
(D) Diphyllobothrium latum is a tapeworm that has a scolex with
a circle of hooks.
347. Each of the following statements concerning hydatid cyst dis- DIRECTIONS (Questions 353–386): Select the ONE lettered
ease is correct EXCEPT: option that is MOST closely associated with the numbered items.
(A) The disease is caused by Echinococcus granulosus. Each lettered option may be selected once, more than once, or not
(B) The cysts occur primarily in the liver. at all.
(C) The disease is caused by a parasite whose adult form lives in Questions 353–360
dogs’ intestines.
(D) The disease occurs primarily in tropical Africa. (A) Dracunculus medinensis
(B) Loa loa
348. Each of the following statements concerning Schistosoma hae-
(C) Onchocerca volvulus
matobium is correct EXCEPT:
(D) Wuchereria bancrofti
(A) Schistosoma haematobium is acquired by humans when cer- (E) Toxocara canis
cariae penetrate the skin.
353. Causes river blindness
(B) Snails are intermediate hosts of S. haematobium.
354. Transmitted by mosquito
(C) Schistosoma haematobium eggs have no spine.
355. Acquired by drinking contaminated water
(D) Schistosoma haematobium infection predisposes to bladder
356. Treated by extracting worm from skin ulcer
carcinoma.
357. Transmitted by the bite of a deer fly or mango fly
358. Causes visceral larva migrans

mebooksfree.com
PART XIII USMLE (National Board) Practice Questions 733

359. Causes filariasis 373. A cestode (tapeworm) acquired by eating undercooked pork
360. Acquired by ingestion of worm eggs 374. A nematode (roundworm) acquired when filariform larvae
penetrate the skin
Questions 361–372
375. A protozoan transmitted by the fecal–oral route
(A) Giardia lamblia 376. A trematode (fluke) acquired by eating undercooked crab meat
(B) Plasmodium vivax 377. A protozoan that infects red blood cells
(C) Taenia saginata 378. Laboratory diagnosis based on finding eggs in sputum
(D) Clonorchis sinensis 379. Causes cysticercosis in humans
(E) Enterobius vermicularis 380. Chloroquine-resistant strains occur
361. A trematode (fluke) acquired by eating undercooked fish 381. Autoinfection within humans, especially in immunocompro-
362. A cestode (tapeworm) acquired by eating undercooked beef mised patients
363. A nematode (roundworm) transmitted primarily from child to 382. Causes blackwater fever
child 383. Causes bloody diarrhea and liver abscesses
364. A protozoan transmitted by mosquito 384. Produces “banana-shaped” gametocytes
365. A protozoan transmitted by the fecal–oral route 385. Produces cysts with four nuclei
366. Primarily affects the biliary ducts 386. Has a scolex with suckers and a circle of hooks
367. Causes diarrhea as the most prominent symptom Answers (Questions 353–386)
368. Causes perianal itching as the most prominent symptom
369. Causes fever, chills, and anemia 353. (C) 360. (E) 367. (A) 374. (E) 380. (E)
370. Can be treated with metronidazole 354. (D) 361. (D) 368. (E) 375. (A) 382. (B)
371. Can be treated with mebendazole or pyrantel pamoate 355. (A) 362. (C) 369. (B) 376. (D) 383. (A)
372. Can be treated with chloroquine and primaquine 356. (A) 363. (E) 370. (A) 377. (B) 384. (B)
357. (B) 364. (B) 371. (E) 378. (D) 385. (A)
Questions 373–386 358. (E) 365. (A) 372. (B) 379. (C) 386. (C)
(A) Entamoeba histolytica 359. (D) 366. (D) 373. (C) 380. (B)
(B) Plasmodium falciparum
(C) Taenia solium
(D) Paragonimus westermani
(E) Strongyloides stercoralis

IMMUNOLOGY
DIRECTIONS (Questions 387–474): Select the ONE lettered answer started on treatment with penicillin, and the sore throat resolves
that is BEST in each question. within several days. However, 7 days after initiation of penicillin
therapy, the patient develops a fever of 103°F, a generalized rash,
387. Which category of hypersensitivity BEST describes hemolytic
and proteinuria. This MOST probably resulted from:
disease of the newborn caused by Rh incompatibility?
(A) Recurrence of the rheumatic fever
(A) Atopic or anaphylactic
(B) A different infectious disease
(B) Cytotoxic
(C) An IgE response to penicillin
(C) Immune complex
(D) An IgG-IgM response to penicillin
(D) Delayed
(E) A delayed hypersensitivity reaction to penicillin
388. The principal difference between cytotoxic (type II) and
391. A kidney biopsy specimen taken from a patient with acute glo-
immune complex (type III) hypersensitivity is:
merulonephritis and stained with fluorescein-conjugated anti-
(A) The class (isotype) of antibody human IgG antibody would probably show:
(B) Whether the antibody reacts with the antigen on the cell or
(A) No fluorescence
reacts with antigen before it interacts with the cell
(B) Uniform fluorescence of the glomerular basement
(C) The participation of complement
membrane
(D) The participation of T cells
(C) Patchy, irregular fluorescence of the glomerular basement
389. A child stung by a bee experiences respiratory distress within membrane
minutes and lapses into unconsciousness. This reaction is prob- (D) Fluorescent B cells
ably mediated by: (E) Fluorescent macrophages
(A) IgE antibody 392. A patient with severe asthma gets no relief from antihistamines.
(B) IgG antibody The symptoms are MOST likely to be caused by:
(C) Sensitized T cells
(A) Interleukin-2
(D) Complement
(B) Slow-reacting substance A (leukotrienes)
(E) IgM antibody
(C) Serotonin
390. A patient with rheumatic fever develops a sore throat from (D) Bradykinin
which β-hemolytic streptococci are cultured. The patient is

mebooksfree.com
734 PART XIII USMLE (National Board) Practice Questions

393. Hypersensitivity to penicillin and hypersensitivity to poison (C) Are very strong antigens
oak are both: (D) Do not induce an immune response
(A) Mediated by IgE antibody 401. Polyomavirus (a DNA virus) causes tumors in “nude mice”
(B) Mediated by IgG and IgM antibody (nude mice do not have a thymus because of a genetic defect)
(C) Initiated by haptens but not in normal mice. The BEST interpretation is that:
(D) Initiated by Th-2 cells (A) Macrophages are required to reject polyomavirus-induced
394. A recipient of a two-haplotype MHC-matched kidney from a tumors.
relative still needs immunosuppression to prevent graft rejec- (B) Natural killer cells can reject polyomavirus-induced tumors
tion because: without help from T lymphocytes.
(A) Graft-versus-host disease is a problem (C) T lymphocytes play an important role in the rejection of
(B) Class II MHC antigens will not be matched polyomavirus-induced tumors.
(C) Minor histocompatibility antigens will not be matched (D) B lymphocytes play no role in rejection of polyomavirus-
(D) Complement components will not be matched induced tumors.
395. Bone marrow transplantation in immunocompromised patients 402. C3 is cleaved to form C3a and C3b by C3 convertase. C3b is
presents which major problem? involved in all of the following EXCEPT:
(A) Potentially lethal graft-versus-host disease (A) Increasing vascular permeability
(B) High risk of T-cell leukemia (B) Promoting phagocytosis
(C) Inability to use a live donor (C) Forming alternative-pathway C3 convertase
(D) Delayed hypersensitivity (D) Forming C5 convertase
396. What is the role of class II MHC proteins on donor cells in graft 403. After binding to its specific antigen, a B lymphocyte may switch its:
rejection? (A) Immunoglobulin light chain isotype
(A) They are the receptors for interleukin-2, which is produced (B) Immunoglobulin heavy chain class
by macrophages when they attack the donor cells. (C) Variable region of the immunoglobulin heavy chain
(B) They are recognized by helper T cells, which then activate (D) Constant region of the immunoglobulin light chain
cytotoxic T cells to kill the donor cells. 404. Diversity is an important feature of the immune system. Which
(C) They induce the production of blocking antibodies that pro- one of the following statements about it is INCORRECT?
tect the graft. (A) Humans can make antibodies with about 108 different VH ×
(D) They induce IgE, which mediates graft rejection. VL combinations.
397. Grafts between genetically identical individuals (i.e., identical (B) A single cell can synthesize IgM antibody then switch to IgA
twins): antibody.
(A) Are rejected slowly as a result of minor histocompatibility (C) The hematopoietic stem cell carries the genetic potential to
antigens create more than 104 immunoglobulin genes.
(B) Are subject to hyperacute rejection (D) A single B lymphocyte can produce antibodies of many dif-
(C) Are not rejected, even without immunosuppression ferent specificities, but a plasma cell is monospecific.
(D) Are not rejected if a kidney is grafted, but skin grafts are 405. C3a and C5a can cause:
rejected (A) Bacterial lysis
398. Penicillin is a hapten in both humans and mice. To explore (B) Vascular permeability
the hapten–carrier relationship, a mouse was injected with (C) Phagocytosis of IgE-coated bacteria
penicillin covalently bound to bovine serum albumin and, at (D) Aggregation of C4 and C2
the same time, with egg albumin to which no penicillin was 406. Neutrophils are attracted to an infected area by:
bound. Of the following, which one will induce a secondary (A) IgM
response to penicillin when injected into the mouse 1 month (B) C1
later? (An explanation of the answer to this question is given on (C) C5a
page 741.) (D) C8
(A) Penicillin 407. Complement fixation refers to:
(B) Penicillin bound to egg albumin
(A) The ingestion of C3b-coated bacteria by macrophages
(C) Egg albumin
(B) The destruction of complement in serum by heating at 56°C
(D) Bovine serum albumin
for 30 minutes
399. AIDS is caused by a human retrovirus that kills: (C) The binding of complement components by antigen–
(A) B lymphocytes antibody complexes
(B) Lymphocyte stem cells (D) The interaction of C3b with mast cells
(C) CD4-positive T lymphocytes 408. The classic complement pathway is initiated by interaction of
(D) CD8-positive T lymphocytes C1 with:
400. Chemically induced tumors have tumor-associated transplan- (A) Antigen
tation antigens that: (B) Factor B
(A) Are always the same for a given carcinogen (C) Antigen–IgG complexes
(B) Are different for two tumors of different histologic type even (D) Bacterial lipopolysaccharides
if induced by the same carcinogen

mebooksfree.com
PART XIII USMLE (National Board) Practice Questions 735

409. Patients with severely reduced C3 levels tend to have: 419. The membrane IgM and IgD on the surface of an individual
(A) Increased numbers of severe viral infections B cell:
(B) Increased numbers of severe bacterial infections (A) Have identical heavy chains but different light chains
(C) Low gamma globulin levels (B) Are identical except for their CH regions
(D) Frequent episodes of hemolytic anemia (C) Are identical except for their VH regions
410. Individuals with a genetic deficiency of C6 have: (D) Have different VH and VL regions
(A) Decreased resistance to viral infections 420. During the maturation of a B lymphocyte, the first immuno-
(B) Increased hypersensitivity reactions globulin heavy chain synthesized is the:
(C) Increased frequency of cancer (A) Mu chain
(D) Increased frequency of Neisseria bacteremia (B) Gamma chain
411. Natural killer cells are: (C) Epsilon chain
(A) B cells that can kill without complement (D) Alpha chain
(B) Cytotoxic T cells 421. In the immune response to a hapten–protein conjugate, in
(C) Increased by immunization order to get anti-hapten antibodies, it is essential that:
(D) Able to kill virus-infected cells without prior sensitization (A) The hapten be recognized by helper T cells
412. A positive tuberculin skin test (a delayed hypersensitivity reac- (B) The protein be recognized by helper T cells
tion) indicates that: (C) The protein be recognized by B cells
(A) A humoral immune response has occurred. (D) The hapten be recognized by suppressor T cells
(B) A cell-mediated immune response has occurred. 422. In the determination of serum insulin levels by radioimmuno-
(C) Both the T-and B-cell systems are functional. assay, which one of the following is NOT needed?
(D) Only the B-cell system is functional. (A) Isotope-labeled insulin
413. Reaction to poison ivy or poison oak is: (B) Anti-insulin antibody made in goats
(A) An IgG-mediated response (C) Anti-goat gamma globulin made in rabbits
(B) An IgE-mediated response (D) Isotope-labeled anti-insulin antibody made in goats
(C) A cell-mediated response 423. Which one of the following sequences is appropriate for testing
(D) An Arthus reaction a patient for antibody against the AIDS virus with the ELISA
414. A child disturbs a wasp nest, is stung repeatedly, and goes into procedure? (The assay is carried out in a plastic plate with an
shock within minutes, manifesting respiratory failure and vas- incubation and a wash step after each addition except the final
cular collapse. This is MOST likely to be due to: one.)
(A) Systemic anaphylaxis (A) Patient’s serum/enzyme substrate/HIV antigen/enzyme-
(B) Serum sickness labeled antibody against HIV
(C) An Arthus reaction (B) HIV antigen/patient’s serum/enzyme-labeled antibody
(D) Cytotoxic hypersensitivity against human gamma globulin/enzyme substrate
(C) Enzyme-labeled antibody against human gamma globulin/
415. “Isotype switching” of immunoglobulin classes by B cells involves:
patient’s serum/HIV antigen/enzyme substrate
(A) Simultaneous insertion of VH genes adjacent to each CH gene (D) Enzyme-labeled antibody against HIV/HIV antigen/patient’s
(B) Successive insertion of a VH gene adjacent to different CH serum/enzyme substrate
genes
424. The BEST method to demonstrate IgG on the glomerular base-
(C) Activation of homologous genes on chromosome 6
ment membrane in a kidney tissue section is the:
(D) Switching of light chain types (kappa and lambda)
(A) Precipitin test
416. Which one of the following pairs of genes is linked on a single
(B) Complement fixation test
chromosome?
(C) Agglutination test
(A) V gene for lambda chain and C gene for kappa chain (D) Indirect fluorescent-antibody test
(B) C gene for gamma chain and C gene for kappa chain
425. A woman had a high fever, hypotension, and a diffuse macular
(C) V gene for lambda chain and V gene for heavy chain
rash. When all cultures showed no bacterial growth, a diagnosis
(D) C gene for gamma chain and C gene for alpha chain
of toxic shock syndrome was made. Regarding the mechanism
417. Idiotypic determinants are located within: by which the toxin causes this disease, which one of the follow-
(A) Hypervariable regions of heavy and light chains ing is LEAST accurate?
(B) Constant regions of light chains (A) The toxin is not processed within the macrophage.
(C) Constant regions of heavy chains (B) The toxin binds to both the class II MHC protein and the
(D) The hinge region T-cell receptor.
418. A primary immune response in an adult human requires (C) The toxin activates many CD4-positive T cells, and large
approximately how much time to produce detectable antibody amounts of interleukins are released.
levels in the blood? (D) The toxin has an A-B subunit structure—the B subunit binds
(A) 12 hours to a receptor, and the A subunit enters the cells and activates
(B) 3 days them.
(C) 1 week
(D) 3 weeks

mebooksfree.com
736 PART XIII USMLE (National Board) Practice Questions

426. A patient with a central nervous system disorder is maintained 433. An antigen found in relatively high concentration in the plasma
on the drug methyldopa. Hemolytic anemia develops, which of normal fetuses and a high proportion of patients with pro-
resolves shortly after the drug is withdrawn. This is MOST gressive carcinoma of the colon is:
probably an example of: (A) Viral antigen
(A) Atopic hypersensitivity (B) Carcinoembryonic antigen
(B) Cytotoxic hypersensitivity (C) α-Fetoprotein
(C) Immune-complex hypersensitivity (D) Heterophil antigen
(D) Cell-mediated hypersensitivity 434. An antibody directed against the idiotypic determinants of a
427. Which one of the following substances is NOT released by acti- human IgG antibody would react with:
vated helper T cells? (A) The Fc part of the IgG
(A) Alpha interferon (B) An IgM antibody produced by the same plasma cell that
(B) Gamma interferon produced the IgG
(C) Interleukin-2 (C) All human kappa chains
(D) Interleukin-4 (D) All human gamma chains
428. A delayed hypersensitivity reaction is characterized by: 435. Which one of the following is NOT true of the gene segments
(A) Edema without a cellular infiltrate that combine to make up a heavy chain gene?
(B) An infiltrate composed of neutrophils (A) Many V region segments are available.
(C) An infiltrate composed of helper T cells and macrophages (B) Several J segments and several D segments are available.
(D) An infiltrate composed of eosinophils (C) V, D, and J segments combine to encode the antigen-binding site.
429. Two dissimilar inbred strains of mice, A and B, are crossed to (D) A V segment and a J segment are preselected by an antigen
yield an F1 hybrid strain, AB. If a large dose of spleen cells from to make up the variable-region portion of the gene.
an adult A mouse is injected into an adult AB mouse, which one 436. When immune complexes from the serum are deposited on
of the following is MOST likely to occur? (An explanation of the glomerular basement membrane, damage to the membrane is
answer to this question is given on page 741.) caused mainly by:
(A) The spleen cells will be destroyed. (A) Gamma interferon
(B) The spleen cells will survive and will have no effect in the (B) Phagocytosis
recipient. (C) Cytotoxic T cells
(C) The spleen cells will induce a graft-versus-host reaction in (D) Enzymes released by polymorphonuclear cells
the recipient. 437. If an individual was genetically unable to make J chains, which
(D) The spleen cells will survive and induce tolerance of strain A immunoglobulin(s) would be affected?
grafts in the recipient. (A) IgG
430. This question is based on the same strains of mice described in (B) IgM
the previous question. If adult AB spleen cells are injected into a (C) IgA
newborn B mouse, which one of the following is MOST likely to (D) IgG and IgM
occur? (An explanation of the answer to this question is given (E) IgM and IgA
on page 741.) 438. The antigen-binding site on antibodies is formed primarily by:
(A) The spleen cells will be destroyed. (A) The constant regions of H and L chains
(B) The spleen cells will survive without any effect on the (B) The hypervariable regions of H and L chains
recipient. (C) The hypervariable regions of H chains
(C) The spleen cells will induce a graft-versus-host reaction in (D) The variable regions of H chains
the recipient. (E) The variable regions of L chains
(D) The spleen cells will survive and induce tolerance of strain A
439. The class of immunoglobulin present in highest concentration
grafts in the recipient.
in the blood of a human newborn is:
431. The minor histocompatibility antigens on cells:
(A) IgG
(A) Are detected by reaction with antibodies and complement (B) IgM
(B) Are controlled by several genes in the major histocompatibil- (C) IgA
ity complex (D) IgD
(C) Are unimportant in human transplantation (E) IgE
(D) Induce reactions that can cumulatively lead to a strong rejec-
440. Individuals of blood group type AB:
tion response
(A) Are Rh(D)-negative
432. Which one of the following is NOT true of class I MHC antigens?
(B) Are “universal recipients” of transfusions
(A) They can be assayed by a cytotoxic test that uses antibody (C) Have circulating anti-A and anti-B antibodies
and complement. (D) Have the same haplotype
(B) One of their two polypeptide chains is a beta-2 microglobulin.
441. Cytotoxic T cells induced by infection with virus A will kill target
(C) They are encoded by at least three gene loci in the major
cells:
histocompatibility complex.
(D) They are found mainly on B cells, macrophages, and acti- (A) From the same host infected with any virus
vated T cells. (B) Infected by virus A and identical at class I MHC loci of the
cytotoxic T cells

mebooksfree.com
PART XIII USMLE (National Board) Practice Questions 737

(C) Infected by virus A and identical at class II MHC loci of the (C) Insertion of complement proteins into the cell membrane
cytotoxic T cells (D) Inhibition of elongation factor-2
(D) Infected with a different virus and identical at class I MHC 448. Graft and tumor rejection are mediated primarily by:
loci of the cytotoxic cells (A) Non–complement-fixing antibodies
(E) Infected with a different virus and identical at class II MHC (B) Phagocytic cells
loci of the cytotoxic cells (C) Helper T cells
442. Antigen-presenting cells that activate helper T cells must (D) Cytotoxic T cells
express which one of the following on their surfaces? 449. Which one of the following properties of antibodies is NOT
(A) IgE dependent on the structure of the heavy chain constant region?
(B) Gamma interferon (A) Ability to cross the placenta
(C) Class I MHC antigens (B) Isotype (class)
(D) Class II MHC antigens (C) Ability to fix complement
443. Which one of the following does NOT contain C3b? (D) Affinity for antigen
(A) Classic-pathway C5 convertase 450. In which one of the following situations would a graft-versus-
(B) Alternative-pathway C5 convertase host reaction be MOST likely to occur? (Mouse strains A and
(C) Classic-pathway C3 convertase B are highly inbred; AB is an F1 hybrid between strain A and
(D) Alternative-pathway C3 convertase strain B.)
444. Which one of the following is NOT true regarding the alterna- (A) Newborn strain A spleen cells injected into a strain B adult
tive complement pathway? (B) X-irradiated adult strain A spleen cells injected into a strain B
(A) It can be triggered by infectious agents in absence of adult
antibody. (C) Adult strain A spleen cells injected into an x-irradiated strain
(B) It does not require C1, C2, or C4. AB adult
(C) It cannot be initiated unless C3b fragments are already (D) Adult strain AB spleen cells injected into a strain A
present. newborn
(D) It has the same terminal sequence of events as the classic 451. In a mixed-lymphocyte culture, lymphocytes from person X,
pathway. who is homozygous for the HLA-Dw7 allele, are irradiated and
445. In setting up a complement fixation test to detect antibody then cultured with lymphocytes from person Z. It is found that
in the patient’s serum, the reactants should be added in what DNA synthesis is NOT stimulated. The proper conclusion to be
sequence? (Ag = antigen; C = complement; EA = antibody- drawn is that:
coated indicator erythrocytes.) (A) Person Z is homozygous for HLA-Dw7
(A) Ag + EA + C/wait/ + patient’s serum (B) Person Z is homozygous or heterozygous for HLA-Dw7
(B) C + patient’s serum + EA/wait/ + Ag (C) Person Z is heterozygous for HLA-Dw7
(C) Ag + patient’s serum + EA/wait/ + C (D) Person Z does not carry the HLA-Dw7 allele
(D) Ag + patient’s serum + C/wait/ + EA 452. A patient skin-tested with purified protein derivative (PPD) to
446. Proteins from two samples of animal blood, A and B, were tested determine previous exposure to Mycobacterium tuberculosis
by the double-diffusion (Ouchterlony) test in agar against antibody develops induration at the skin test site 48 hours later. Histo-
to bovine albumin. Which sample(s) contain horse blood? (An logically, the reaction site would MOST probably show:
explanation of the answer to this question is given on page 741.) (A) Eosinophils
(B) Neutrophils
(C) Helper T cells and macrophages
(D) B cells
453. Hemolytic disease of the newborn caused by Rh blood group
incompatibility requires maternal antibody to enter the fetal
bloodstream. Therefore, the mediator of this disease is:
(A) IgE antibody
(B) IgG antibody
(C) IgM antibody
(D) IgA antibody
454. An Rh-negative woman married to a heterozygous Rh-positive
(A) Sample A man has three children. The probability that all three of their
(B) Sample B children are Rh-positive is:
(C) Both samples (A) 1:2
(D) Neither sample (B) 1:4
447. Complement lyses cells by: (C) 1:8
(A) Enzymatic digestion of the cell membrane (D) Zero
(B) Activation of adenylate cyclase

mebooksfree.com
738 PART XIII USMLE (National Board) Practice Questions

455. Which one of the following statements BEST explains the (C) C6
relationship between inflammation of the heart (carditis) and (D) C9
infection with group A β-hemolytic streptococci? 463. The main advantage of passive immunization over active
(A) Streptococcal antigens induce antibodies cross-reactive with immunization is that:
heart tissue. (A) It can be administered orally.
(B) Streptococci are polyclonal activators of B cells. (B) It provides antibody more rapidly.
(C) Streptococcal antigens bind to IgE on the surface of heart (C) Antibody persists for a longer period.
tissue and histamine is released. (D) It contains primarily IgM.
(D) Streptococci are ingested by neutrophils that release prote- 464. On January 15, a patient developed an illness suggestive of
ases that damage heart tissue. influenza, which lasted 1 week. On February 20, she had a
456. Your patient became ill 10 days ago with a viral disease. similar illness. She had no influenza immunization during this
Laboratory examination reveals that the patient’s antibodies period. Her hemagglutination inhibition titer to influenza A
against this virus have a high ratio of IgM to IgG. What is your virus was 10 on January 18, 40 on January 30, and 320 on
conclusion? February 20. Which one of the following is the MOST appropri-
(A) It is unlikely that the patient has encountered this organism ate interpretation?
previously. (A) The patient was ill with influenza A on January 15.
(B) The patient is predisposed to IgE-mediated hypersensitivity (B) The patient was ill with influenza A on February 20.
reactions. (C) The patient was not infected with influenza virus.
(C) The information given is irrelevant to previous antigen (D) The patient has an autoimmune disease.
exposure. 465. An individual who is heterozygous for Gm allotypes contains
(D) It is likely that the patient has an autoimmune disease. two allelic forms of IgG in serum, but individual lymphocytes
457. If you measure the ability of cytotoxic T cells from an HLA-B27 produce only one of the two forms. This phenomenon, known
person to kill virus X–infected target cells, which one of the fol- as “allelic exclusion,” is consistent with:
lowing statements is CORRECT? (A) A rearrangement of a heavy chain gene on only one chromo-
(A) Any virus X–infected target cell will be killed. some in a lymphocyte
(B) Only virus X–infected cells of HLA-B27 type will be killed. (B) Rearrangements of heavy chain genes on both chromosomes
(C) Any HLA-B27 cell will be killed. in a lymphocyte
(D) No HLA-B27 cell will be killed. (C) A rearrangement of a light chain gene on only one chromo-
458. You have a patient who makes autoantibodies against his own some in a lymphocyte
red blood cells, leading to hemolysis. Which one of the follow- (D) Rearrangements of light chain genes on both chromosomes
ing mechanisms is MOST likely to explain the hemolysis? in a lymphocyte
(A) Perforins from cytotoxic T cells lyse the red cells. 466. Each of the following statements concerning class I MHC pro-
(B) Neutrophils release proteases that lyse the red cells. teins is correct EXCEPT:
(C) Interleukin-2 binds to its receptor on the red cells, which (A) They are cell surface proteins on virtually all cells.
results in lysis of the red cells. (B) They are recognition elements for cytotoxic T cells.
(D) Complement is activated, and membrane attack complexes (C) They are codominantly expressed.
lyse the red cells. (D) They are important in the skin test response to Mycobacte-
459. Your patient is a child who has no detectable T or B cells. This rium tuberculosis.
immunodeficiency is most probably the result of a defect in 467. Which one of the following is the BEST method of reducing the
(A) The thymus effect of graft-versus-host disease in a bone marrow recipient?
(B) The membrane attack complex of complement (A) Matching the complement components of donor and
(C) T cell–B cell interaction recipient
(D) Stem cells originating in the bone marrow (B) Administering alpha interferon
460. The role of dendritic cells during an antibody response is to: (C) Removing mature T cells from the graft
(A) Make antibody (D) Removing pre–B cells from the graft
(B) Lyse virus-infected target cells 468. Regarding Th-1 and Th-2 cells, which one of the following is
(C) Activate cytotoxic T cells LEAST accurate?
(D) Process antigen and present it (A) Th-1 cells produce gamma interferon and promote cell-
461. The structural basis of blood group A and B antigen specificity mediated immunity.
is: (B) Th-2 cells produce interleukin-4 and-5 and promote
(A) A single terminal sugar residue antibody-mediated immunity.
(B) A single terminal amino acid (C) Both Th-1 and Th-2 cells have both CD3 and CD4 proteins
(C) Multiple differences in the carbohydrate portion on their outer cell membrane.
(D) Multiple differences in the protein portion (D) Before naive Th cells differentiate into Th-1 or Th-2 cells,
462. Complement can enhance phagocytosis because of the presence they are double-positives (i.e., they produce both gamma
on macrophages and neutrophils of receptors for: interferon and interleukin-4).
(A) Factor D
(B) C3b

mebooksfree.com
PART XIII USMLE (National Board) Practice Questions 739

469. Each of the following statements concerning the variable 417. (A) 429. (C) 441. (B) 453. (B) 465. (A)
regions of heavy chains and the variable regions of light chains 418. (C) 430. (D) 442. (D) 454. (C) 466. (D)
in a given antibody molecule is correct EXCEPT: 419. (B) 431. (D) 443. (C) 455. (A) 467. (C)
(A) They have the same amino acid sequence. 420. (A) 432. (D) 444. (C) 456. (A) 468. (D)
(B) They define the specificity for antigen. 421. (B) 433. (B) 445. (D) 457. (B) 469. (A)
(C) They are encoded on different chromosomes. 422. (D) 434. (B) 446. (B) 458. (D) 470. (A)
(D) They contain the hypervariable regions. 423. (B) 435. (D) 447. (C) 459. (D) 471. (D)
470. Each of the following statements concerning class II MHC pro- 424. (D) 436. (D) 448. (D) 460. (D) 472. (A)
teins is correct EXCEPT: 425. (D) 437. (E) 449. (D) 461. (A) 473. (C)
426. (B) 438. (B) 450. (C) 462. (B) 474. (D)
(A) They are found on the surface of both B and T cells.
427. (A) 439. (A) 451. (B) 463. (B)
(B) They have a high degree of polymorphism.
428. (C) 440. (B) 452. (C) 464. (A)
(C) They are involved in the presentation of antigen by
macrophages.
(D) They have a binding site for CD4 proteins.
471. Which one of the following statements concerning immuno- DIRECTIONS (Questions 475–535): Select the ONE lettered option
globulin allotypes is CORRECT? that is MOST closely associated with the numbered items. Each lettered
(A) Allotypes are found only on heavy chains. option may be selected once, more than once, or not at all.
(B) Allotypes are determined by class I MHC genes. Questions 475–480
(C) Allotypes are confined to the variable regions.
(D) Allotypes are due to genetic polymorphism within a (A) T cells
species. (B) B cells
472. Each of the following statements concerning immunologic tol- (C) Macrophages
erance is correct EXCEPT: (D) B cells and macrophages
(E) T cells, B cells, and macrophages
(A) Tolerance is not antigen-specific (i.e., paralysis of the
immune cells results in a failure to produce a response 475. Major source of interleukin-1
against many antigens). 476. Acted on by interleukin-1
(B) Tolerance is more easily induced in T cells than in B cells. 477. Major source of interleukin-2
(C) Tolerance is more easily induced in neonates than in adults. 478. Express class I MHC proteins
(D) Tolerance is more easily induced by simple molecules than 479. Express class II MHC proteins
by complex ones. 480. Express surface immunoglobulin
473. Each of the following statements concerning a hybridoma cell is Questions 481–484
correct EXCEPT:
(A) Primary antibody response
(A) The spleen cell component provides the ability to form
(B) Secondary antibody response
antibody.
(B) The myeloma cell component provides the ability to grow 481. Appears more quickly and persists longer
indefinitely. 482. Relatively richer in IgG
(C) The antibody produced by a hybridoma cell is IgM, because 483. Relatively richer in IgM
heavy chain switching does not occur. 484. Typically takes 7 to 10 days for antibody to appear
(D) The antibody produced by a hybridoma cell is homogeneous Questions 485–488
(i.e., it is directed against a single epitope).
(A) Blood group A
474. Each of the following statements concerning haptens is correct
(B) Blood group O
EXCEPT:
(C) Blood groups A and O
(A) A hapten can combine with (bind to) an antibody. (D) Blood group AB
(B) A hapten cannot induce an antibody by itself; rather, it must
485. People with this type have circulating anti-A antibodies
be bound to a carrier protein to be able to induce antibody.
486. People with this type have circulating anti-B antibodies
(C) In both penicillin-induced anaphylaxis and poison ivy, the
487. People with this type are called “universal donors”
allergens are haptens.
488. People with this type are called “universal recipients”
(D) Haptens must be processed by CD8+ cells to become
immunogenic. Questions 489–494
Answers (Questions 387–474) (A) Variable region of light chain
(B) Variable region of heavy chain
387. (B) 393. (C) 399. (C) 405. (B) 411. (D) (C) Variable regions of light and heavy chains
388. (B) 394. (C) 400. (B) 406. (C) 412. (B) (D) Constant region of heavy chain
389. (A) 395. (A) 401. (C) 407. (C) 413. (C) (E) Constant regions of light and heavy chains
390. (D) 396. (B) 402. (A) 408. (C) 414. (A)
489. Determines immunoglobulin class
391. (C) 397. (C) 403. (B) 409. (B) 415. (B)
490. Determines allotypes
392. (B) 398. (D) 404. (D) 410. (D) 416. (D)
491. Determines idiotypes

mebooksfree.com
740 PART XIII USMLE (National Board) Practice Questions

492. Binding of IgG to macrophages Questions 510–513


493. Activation of complement by IgG
(A) IgA
494. Antigen-binding site
(B) IgE
Questions 495–498 (C) IgG
(D) IgM
The following double-immunodiffusion plate contains antibody pre- 510. Present in highest concentration in serum
pared against whole human serum in the center well. Identify the 511. Present in highest concentration in secretions
contents of each peripheral well from the following list (each well to be 512. Present in lowest concentration in serum
used once). (An explanation of the answer to this question is given on 513. Contains 10 heavy and 10 light chains
page 741.)
Questions 514–517
In this double-diffusion (Ouchterlony) assay, the center well contains
antibody against whole human serum. The peripheral (numbered)
wells each contain one of the following proteins:

495. Whole human serum


496. Human IgG
497. Baboon IgG (A) Human serum albumin at low concentration
498. Human transferrin (B) Human serum albumin at high concentration
(C) Human serum transferrin
Questions 499–501 (D) Sheep serum albumin
(A) Immediate hypersensitivity 514. Which protein is present in well No. 1?
(B) Cytotoxic hypersensitivity 515. Which protein is present in well No. 2?
(C) Immune-complex hypersensitivity 516. Which protein is present in well No. 3?
(D) Delayed hypersensitivity 517. Which protein is present in well No. 4? (An explanation of the
answer to this question is given on page 741.)
499. Irregular deposition of IgG along glomerular basement
-membrane Questions 518–521
500. Involves mast cells and basophils (A) Class I MHC proteins
501. Involves macrophages and helper T cells (B) Class II MHC proteins
Questions 502–505 518. Involved in the presentation of antigen to CD4-positive cells
519. Involved in the presentation of antigen to CD8-positive cells
(A) IgM 520. Involved in antibody responses to T-dependent antigens
(B) IgG 521. Involved in target cell recognition by cytotoxic T cells
(C) IgA
(D) IgE Questions 522–525
502. Crosses the placenta (A) Fab fragment of IgG
503. Can contain a polypeptide chain not synthesized by a B (B) Fc fragment of IgG
lymphocyte 522. Contains an antigen-combining site
504. Found in highest concentration in the milk of lactating women 523. Contains hypervariable regions
505. Binds firmly to mast cells and triggers anaphylaxis 524. Contains a complement-binding site
525. Is crystallizable
Questions 506–509
Questions 526–530
(A) Agglutination
(B) Precipitin test (A) Severe combined immunodeficiency disease (SCID)
(C) Immunofluorescence (B) X-linked hypogammaglobulinemia
(D) Enzyme immunoassay (C) Thymic aplasia
506. Concentration of IgG in serum (D) Chronic granulomatous disease
507. Surface IgM on cells in a bone marrow smear (E) Hereditary angioedema
508. Growth hormone in serum 526. Caused by a defect in the ability of neutrophils to kill
509. Type A blood group antigen on erythrocytes microorganisms
527. Caused by a development defect that results in a profound loss
of T cells

mebooksfree.com
PART XIII USMLE (National Board) Practice Questions 741

528. Caused by a deficiency in an inhibitor of the C1 component of survive, they will have an effect on the recipient. Answer D is incorrect
complement because the recipient is already tolerant to antigen A.
529. Caused by a marked deficiency of B cells
Explanation of question 430: Because the donor AB spleen cells will
530. Caused by a virtual absence of both B and T cells
not see any foreign antigen in the recipient, no graft-versus-host reac-
Questions 531–535 tion will occur; therefore, answer C is incorrect. The immune cells of
the newborn mouse do not have the capability to kill the donor cells;
(A) Systemic lupus erythematosus
therefore, answer A is incorrect. Answer D is more correct than
(B) Rheumatoid arthritis
answer B because the donor cells will survive and induce tolerance to
(C) Rheumatic fever
antigen A in the newborn recipient.
(D) Graves’ disease
(E) Myasthenia gravis Explanation of question 446: There is a line of identity between sam-
531. Associated with antibody to the thyroid-stimulating hormone ple A and bovine albumin; therefore, sample A is bovine albumin.
(TSH) receptor There is a line of identity between sample B and horse albumin; there-
532. Associated with antibody to IgG fore, sample B is horse albumin. The answer to the question is therefore
533. Associated with antibody to the acetylcholine receptor B. Note that there is a spur formed between the wells containing sample
534. Associated with antibody to DNA A and horse albumin and between the wells containing sample B and
535. Associated with antibody to Streptococcus pyogenes bovine albumin. The spur indicates partial identity between the two
proteins. Partial identity means that there are epitopes shared between
Answers (Questions 475–535) the two albumins but that, because they are from different species,
475. (C) 488. (D) 501. (D) 514. (B) 527. (C) there are epitopes unique to each protein, also. A spur is formed by the
476. (A) 489. (D) 502. (B) 515. (A) 528. (E) interaction of the subset of antibodies in the anti-bovine serum with
477. (A) 490. (E) 503. (C) 516. (D) 529. (B) the unique epitopes in bovine albumin. The other lines are formed by
478. (E) 491. (C) 504. (C) 517. (C) 530. (A) the interaction of the subset of antibodies in the anti-bovine serum
479. (D) 492. (D) 505. (D) 518. (B) 531. (D) with the epitopes shared by the two albumins.
480. (B) 493. (D) 506. (D) 519. (A) 532. (B)
Explanation of questions 495–498: The center well contains antibody
481. (B) 494. (C) 507. (C) 520. (B) 533. (E)
against whole human serum; therefore, well D must contain whole
482. (B) 495. (D) 508. (D) 521. (A) 534. (A)
human serum because there are multiple lines representing some of the
483. (A) 496. (C) 509. (A) 522. (A) 535. (C)
many proteins in whole human serum. There is a line of identity
484. (A) 497. (A) 510. (C) 523. (A)
between well C and a protein in whole human serum and a line of
485. (B) 498. (B) 511. (A) 524. (B)
partial identity with that same protein and well A. This indicates that
486. (C) 499. (C) 512. (B) 525. (B)
well C contains human IgG and well A contains baboon IgG. The con-
487. (B) 500. (A) 513. (D) 526. (D)
cept of partial identity is explained above in the discussion of ques-
tion 446. There is a line of nonidentity between wells B and C;
Explanation of question 398: Bovine serum albumin is the correct therefore, well B contains human transferrin, a protein immunologi-
answer because it activates helper T cells that are required to provide cally distinct from human IgG.
the interleukins needed for a secondary response. Penicillin alone is
incorrect because it is a hapten and cannot activate helper T cells. Explanation of questions 514–517: There is a line of identity between
Choices B and C refer to egg albumin which can activate helper T cells wells 1 and 2; therefore, they contain human serum albumin (HSA).
but not the ones that were activated by the initial stimulus that con- Note that the line of immunoprecipitate is very close to well 2. This line
tained penicillin bound to bovine serum albumin. would not form if well 2 contained the high concentration of HSA
because it would be a zone of antigen excess and the line only forms in
Explanation of question 429: Spleen cells from the adult donor A will a zone of equivalence. Therefore, well 2 contains the low concentration
recognize the B antigen on the recipient’s cells as foreign. Spleen cells and well 1 contains the high concentration of HSA. There is a line of
from the adult donor will contain mature CD4 and CD8 cells that will partial identity between wells 2 and 3; therefore, well 3 contains sheep
attack the recipient cells, causing a graft-versus-host reaction; there- serum albumin (SSA). There is a line of nonidentity between wells 1
fore, answer C is correct. Because the recipient is tolerant to antigen A, and 4 and wells 3 and 4; therefore, well 4 contains human transferrin,
the donor A spleen cells will not be destroyed; therefore, answer A is which is immunologically distinct from HSA and SSA.
incorrect. Answer B is incorrect because although the donor cells will

EXTENDED MATCHING QUESTIONS


DIRECTIONS (Questions 536–593): Each set of matching ques-
(D) Lipid A
tions in this section consists of a list of lettered options followed by
(E) 30S ribosomal subunit
several numbered items. For each numbered item, select the ONE let-
(F) G protein
tered option that is MOST closely associated with it. Each lettered
(G) Pilus
option may be selected once, more than once, or not at all.
(H) ADP-ribosylating enzyme
(A) Capsule (I) Flagellum
(B) Periplasmic space (J) Transposon
(C) Peptidoglycan

mebooksfree.com
742 PART XIII USMLE (National Board) Practice Questions

536. Is the site of action of lysozyme (J) Streptococcus agalactiae


537. Mediates adherence of bacteria to mucous membranes (K) Staphylococcus epidermidis
538. Is the toxic component of endotoxin (L) Streptococcus mutans
(A) Skin 551. The vaccine contains a single serotype of a capsular polysaccha-
(B) Colon ride coupled to a protein carrier
(C) Nose 552. Immunogen in the vaccine is a toxoid
(D) Stomach 553. Causes acute glomerulonephritis; is β-hemolytic
(E) Vagina 554. Causes urinary tract infections; grows in 6.5% NaCl
(F) Mouth 555. Causes neonatal meningitis; is bacitracin-resistant
(G) Outer third of urethra 556. Causes meningitis in adults; is α-hemolytic and optochin-sensitive
(H) Gingival crevice 557. Causes food poisoning; is coagulase-positive
(I) Pharynx (A) Escherichia coli
539. Anatomic location where Bacteroides fragilis is most commonly (B) Shigella sonnei
found (C) Salmonella typhi
540. Anatomic location where Actinomyces israelii is most com- (D) Salmonella enteritidis
monly found (E) Proteus mirabilis
(A) Toxic shock syndrome toxin (F) Pseudomonas aeruginosa
(B) Tetanus toxin (G) Vibrio cholerae
(C) Diphtheria toxin (H) Campylobacter jejuni
(D) Cholera toxin (I) Helicobacter pylori
(E) Coagulase (J) Bacteroides fragilis
(F) Botulinum toxin 558. Causes gastritis and peptic ulcer; produces urease
(G) Alpha toxin of Clostridium perfringens 559. Causes bloody diarrhea; does not ferment lactose and does not
(H) M protein produce H2S
(I) Endotoxin 560. Causes peritonitis; is an obligate anaerobe
(J) Verotoxin 561. Causes wound infections with blue-green pus; is oxidase-positive
541. Blocks release of acetylcholine 562. Comma-shaped rod; causes high-volume watery diarrhea
542. Its lipid component causes fever and shock by inducing tumor (A) Legionella pneumophila
necrosis factor (TNF) (B) Yersinia pestis
543. Causes fever and shock by binding to the T-cell receptor (C) Haemophilus influenzae
544. Inhibits protein synthesis by ADP-ribosylation of elongation (D) Corynebacterium diphtheriae
factor-2 (E) Pasteurella multocida
545. Increases cyclic AMP by ADP-ribosylation of a G protein (F) Bordetella pertussis
(A) Ampicillin (G) Brucella melitensis
(B) Nafcillin (H) Listeria monocytogenes
(C) Clindamycin (I) Clostridium perfringens
(D) Gentamicin (J) Neisseria gonorrhoeae
(E) Tetracycline 563. Gram-positive spore-forming rod that causes myonecrosis
(F) Amphotericin B 564. Gram-negative rod that is transmitted by cat bite
(G) Ciprofloxacin 565. Gram-negative rod that causes cough and lymphocytosis
(H) Rifampin (A) Mycobacterium tuberculosis
(I) Sulfonamide (B) Borrelia burgdorferi
(J) Erythromycin (C) Nocardia asteroides
546. Inhibits protein synthesis by blocking formation of the initia- (D) Treponema pallidum
tion complex so that no polysomes form (E) Coxiella burnetii
547. Inhibits DNA gyrase (F) Mycoplasma pneumoniae
548. Inhibits folic acid synthesis; analogue of para-aminobenzoic (G) Mycobacterium leprae
acid (H) Chlamydia trachomatis
549. Inhibits peptidoglycan synthesis; resistant to β-lactamase (I) Rickettsia rickettsii
550. Inhibits RNA polymerase (J) Leptospira interrogans
(A) Streptococcus pneumoniae 566. Spirochete that does not have an animal reservoir
(B) Streptococcus pyogenes 567. Obligate intracellular parasite that forms elementary bodies
(C) Haemophilus influenzae 568. Respiratory pathogen without a cell wall
(D) Salmonella typhi (A) Influenza virus
(E) Staphylococcus aureus (B) Adenovirus
(F) Enterococcus faecalis (C) Hepatitis A virus
(G) Clostridium tetani (D) Hepatitis B virus
(H) Bordetella pertussis (E) Herpes simplex virus
(I) Escherichia coli (F) Measles virus

mebooksfree.com
PART XIII USMLE (National Board) Practice Questions 743

(G) Human immunodeficiency virus (F) Candida albicans


(H) Rabies virus (G) Histoplasma capsulatum
(I) Rotavirus (H) Mucor species
569. Nonenveloped virus with a genome composed of single- (I) Microsporum canis
stranded, positive-polarity RNA 585. Dimorphic fungus that enters the body through puncture
570. Enveloped virus with a genome composed of two identical wounds in the skin
strands of positive-polarity RNA 586. Nonseptate mold that invades tissue, especially in acidotic
571. Enveloped virus with a genome composed of double-stranded patients
DNA and has a DNA polymerase in the virion 587. Yeast that forms pseudohyphae when it invades tissue
572. Enveloped virus with a genome composed of segmented, negative- (A) Giardia lamblia
polarity, single-stranded RNA (B) Plasmodium vivax
573. Nonenveloped virus with a genome composed of segmented (C) Leishmania donovani
double-stranded RNA (D) Entamoeba histolytica
(A) Herpes simplex virus type 1 (E) Toxoplasma gondii
(B) Rabies virus (F) Trypanosoma cruzi
(C) Varicella-zoster virus (G) Pneumocystis carinii
(D) Measles virus (H) Plasmodium falciparum
(E) Epstein–Barr virus (I) Naegleria fowleri
(F) Influenza virus (J) Trichomonas vaginalis
(G) Rubella virus 588. Acquired while swimming; causes meningitis
(H) Herpes simplex virus type 2 589. Transmitted by reduviid bug and invades cardiac muscle
(I) Mumps virus 590. Amastigotes found within macrophages
(J) Cytomegalovirus (A) Echinococcus granulosus
(K) Parainfluenza virus (B) Clonorchis sinensis
(L) Respiratory syncytial virus (C) Strongyloides stercoralis
574. Leading cause of congenital malformations; no vaccine available (D) Taenia solium
575. Causes a painful vesicular rash along the course of a thoracic (E) Necator americanus
nerve (F) Enterobius vermicularis
576. Causes encephalitis; killed vaccine available (G) Schistosoma haematobium
577. Causes pharyngitis, lymphadenopathy, and a positive hetero- (H) Wuchereria bancrofti
phil test (I) Trichinella spiralis
578. Causes retinitis and pneumonia in patients deficient in helper T (J) Taenia saginata
cells 591. Infection predisposes to bladder carcinoma
579. Causes encephalitis, especially in the temporal lobe 592. Ingestion of eggs can cause cysticercosis
580. Causes pneumonia primarily in infants; induces giant cells 593. Acquired by penetration of feet by larvae; causes anemia
581. Causes orchitis that can result in sterility
(A) Human papillomavirus Answers (Questions 536–593)
(B) Hepatitis A virus 536. (C) 548. (I) 560. (J) 572. (A) 584. (E)
(C) Rotavirus 537. (G) 549. (B) 561. (F) 573. (I) 585. (D)
(D) Adenovirus 538. (D) 550. (H) 562. (G) 574. (J) 586. (H)
(E) Hepatitis delta virus (HDV) 539. (B) 551. (C) 563. (I) 575. (C) 587. (F)
(F) Parvovirus B19 540. (H) 552. (G) 564. (E) 576. (B) 588. (I)
(G) Human immunodeficiency virus 541. (F) 553. (B) 565. (F) 577. (E) 589. (F)
(H) Hepatitis B virus 542. (I) 554. (F) 566. (D) 578. (J) 590. (C)
(I) Sin Nombre virus (Hantavirus) 543. (A) 555. (J) 567. (H) 579. (A) 591. (G)
(J) Human T-cell lymphotropic virus 544. (C) 556. (A) 568. (F) 580. (L) 592. (D)
(K) Prion 545. (D) 557. (E) 569. (C) 581. (I) 593. (E)
(L) Hepatitis C virus 546. (D) 558. (I) 570. (G) 582. (C)
582. Most important cause of diarrhea in infants 547. (G) 559. (B) 571. (D) 583. (H)
583. A vaccine containing purified viral protein is available
584. Defective virus with an RNA genome
(A) Coccidioides immitis
(B) Cryptococcus neoformans
(C) Blastomyces dermatitidis
(D) Sporothrix schenckii
(E) Aspergillus fumigatus

mebooksfree.com
744 PART XIII USMLE (National Board) Practice Questions

CLINICAL CASE QUESTIONS


DIRECTIONS (Questions 594–654): Select the ONE lettered answer (C) Culture of the sputum on Löwenstein-Jensen medium shows
that is BEST in each question. tan colonies after incubation for 4 weeks.
(D) Rapid plasma reagin test reveals the causative organism.
CASE 1. Your patient is a 20-year-old woman with the sudden onset of
fever to 104°F and a severe headache. Physical examination reveals CASE 5. Your patient is a 5-year-old girl with bloody diarrhea and no
nuchal rigidity. You suspect meningitis and do a spinal tap. Gram stain vomiting. There is no history of travel outside of San Francisco. Stool
of the spinal fluid reveals many neutrophils and many gram-negative culture grows both lactose-positive and lactose-negative colonies on
diplococci. EMB agar.
594. Of the following bacteria, which one is MOST likely to be the 599. Of the following organisms, which one is MOST likely to be the
cause? cause?
(A) Haemophilus influenzae (A) Shigella sonnei
(B) Neisseria meningitidis (B) Salmonella typhi
(C) Streptococcus pneumoniae (C) Campylobacter jejuni
(D) Pseudomonas aeruginosa (D) Helicobacter pylori
595. Additional history reveals that she has had several serious infec-
CASE 6. Your patient is a 25-year-old woman with acute onset of pain
tions with this organism previously. On the basis of this, which
in her left lower quadrant. On pelvic examination, there is a cervical
one of the following is the MOST likely predisposing factor?
exudate and tenderness in the left adnexa. You conclude that she has
(A) She is HIV antibody positive. pelvic inflammatory disease (PID) and order laboratory tests.
(B) She is deficient in CD8-positive T cells.
(C) She is deficient in one of the late-acting complement 600. Of the following, which one is the LEAST informative labora-
components. tory result?
(D) She is deficient in antigen presentation by her macrophages. (A) Gram stain of the cervical exudate shows gram-negative
diplococci within polys.
CASE 2. Your patient is a 70-year-old man with a long history of smok- (B) Culture of the cervical exudate on Thayer-Martin agar shows
ing who now has a fever and a cough productive of greenish sputum. oxidase-positive colonies.
You suspect pneumonia, and a chest X-ray confirms your suspicion. (C) Fluorescent-antibody test shows cytoplasmic inclusions.
596. If a Gram stain of the sputum reveals very small gram-negative (D) Complement fixation test shows a rise in antibody titer.
rods and there is no growth on a blood agar but colonies do
CASE 7. Your patient is a 22-year-old man with fever, fatigue, and a
grow on chocolate agar supplemented with NAD and heme,
new diastolic murmur. You suspect endocarditis and do a blood
which one of the following bacteria is the MOST likely cause?
culture.
(A) Chlamydia pneumoniae
(B) Legionella pneumophila 601. Which of the following statements is LEAST accurate?
(C) Mycoplasma pneumoniae (A) If he had dental surgery recently, one of the most likely
(D) Haemophilus influenzae organisms to grow would be a viridans group
streptococcus.
CASE 3. Your patient is a 50-year-old woman who returned yesterday
(B) If he is an intravenous drug user, one of the most likely
from a vacation in Peru, where there is an epidemic of cholera. She now
organisms to grow would be Candida albicans.
has multiple episodes of diarrhea.
(C) If he had colon surgery recently, one of the most likely organ-
597. Of the following, which one is MOST compatible with cholera? isms to grow would be Enterococcus faecalis.
(A) Watery diarrhea without blood, no polys in the stool, and (D) If he has a prosthetic aortic valve, one of the most likely
growth of curved gram-negative rods in the blood culture organisms to grow would be Streptococcus agalactiae.
(B) Watery diarrhea without blood, no polys in the stool, and no
organisms in the blood culture In fact, none of the above organisms grew in the blood culture. What
(C) Bloody diarrhea, polys in the stool, and growth of curved did grow was a gram-positive coccus arranged in clusters. When sub-
gram-negative rods in the blood culture cultured on blood agar, the colonies were surrounded by a zone of clear
(D) Bloody diarrhea, polys in the stool, and no organisms in the hemolysis, and a coagulase test was positive.
blood culture 602. In view of this, which one of the following is MOST accurate?
(A) He is probably an intravenous drug user.
CASE 4. Your patient is a 55-year-old man who is coughing up green-
(B) He probably lives on a farm and has had contact with preg-
ish blood-streaked sputum. For the past 2 weeks, he has had fever and
nant sheep.
night sweats. He thinks he has lost about 10 lb. On physical examina-
(C) He probably has a common sexually transmitted disease.
tion, there are crackles in the apex of the right lung, and a chest X-ray
(D) He probably has been camping and was bitten by a tick.
shows a cavity in that location.
598. Of the following, which one is the LEAST likely finding? CASE 8. Your patient is a 70-year-old woman who had a hysterectomy
(A) Gram stain of the sputum shows no predominant organism. for carcinoma of the uterus 3 days ago. She has an indwelling urinary
(B) Culture of the sputum on blood agar shows no predominant catheter in place and now has a fever to 39°C, and the urine in the
organism. collection bottle is cloudy. A Gram stain of the urine specimen shows

mebooksfree.com
PART XIII USMLE (National Board) Practice Questions 745

many neutrophils and gram-positive cocci in chains. You also do a 607. Regarding this organism, which one of the following is MOST
urine culture. accurate?
603. Which one of the following is the MOST likely set of findings (A) It has more than 100 serologic types.
on the urine culture? (B) It produces an exotoxin that inhibits elongation factor-2.
(A) β-Hemolytic colonies that are bacitracin-sensitive (C) It is commonly acquired by eating unpasteurized dairy
(B) α-Hemolytic colonies that are optochin-sensitive products.
(C) Nonhemolytic colonies that grow in 6.5% sodium chloride (D) There is a toxoid vaccine available against this organism.
(D) Nonhemolytic colonies that grow only anaerobically CASE 13. Ms. Jones calls to say that she, her husband, and their child
CASE 9. Your patient is a 27-year-old woman who was treated with have had nausea and vomiting for the past hour or so. Also, they have
oral ampicillin for cellulitis caused by Streptococcus pyogenes. Several had some non-bloody diarrhea. You ask when their last meal together
days later, she developed bloody diarrhea. You suspect that she may was, and she says they had a picnic lunch in the park about 3 hours ago.
have pseudomembranous colitis. They have no fever.

604. Regarding the causative organism of pseudomembranous coli- 608. Which one of the following is the MOST likely finding?
tis, which one of the following is the MOST accurate? (A) Gram stain of the leftover food would show many gram-
(A) It is an anaerobic gram-positive rod that produces positive cocci in clusters.
exotoxins. (B) Gram stain of the stool would show many gram-negative
(B) It is a comma-shaped gram-negative rod that grows best at diplococci.
41°C. (C) KOH prep of the leftover food would show many budding
(C) It is an obligate intracellular parasite that grows in cell cul- yeasts.
ture but not on blood agar. (D) Acid-fast stain of the stool would show many acid-fast rods.
(D) It is a yeast that forms germ tubes when incubated in human CASE 14. Your patient is a 9-year-old boy who was sent home from
serum at 37°C. school because his teacher thought he was acting strangely. This morn-
CASE 10. Your patient is a 10-year-old girl who has had pain in her left ing, he had a seizure and was rushed to the hospital. On physical
arm for the past 5 days. On physical examination, her temperature is examination, his temperature is 40°C and he has no nuchal rigidity. A
38°C, and there is tenderness of the humerus near her deltoid. On computed tomography (CT) scan is normal. A lumbar puncture is
X-ray of the humerus, an area of raised periosteum and erosion of bone done, and the spinal fluid protein and glucose are normal. A Gram
is seen. You do a blood culture. stain of the spinal fluid reveals no organisms and no polys. He is treated
with various antibiotics but becomes comatose and dies 2 days later.
605. Which one of the following is the MOST likely set of findings? The blood culture and spinal fluid culture grow no bacteria or fungi.
(A) Gram-negative rods that grow on EMB agar, forming purple On autopsy of the brain, eosinophilic inclusion bodies are seen in the
colonies and a green sheen cytoplasm of neurons.
(B) Gram-positive cocci that grow on blood agar, causing a clear
609. Of the following, which one is the MOST likely cause?
zone of hemolysis and are coagulase-positive
(C) Gram-positive rods that grow only anaerobically and form a (A) Prions
double zone of hemolysis on blood agar (B) JC virus
(D) Gram-negative diplococci that grow on blood agar, are (C) Rabies virus
oxidase-positive, and ferment maltose (D) Herpes simplex virus type 1

CASE 11. Your patient is a 30-year-old man who is HIV antibody CASE 15. Your patient is a 20-year-old man who was in a fist fight and
positive and has a history of Pneumocystis pneumonia 2 years ago. He suffered a broken jaw and lost two teeth. Several weeks later, he devel-
now has an ulcerating lesion on the side of his tongue. A Giemsa stain oped an abscess at the site of the trauma that drained to the surface of
of the biopsy specimen reveals budding yeasts within macrophages. A the skin, and yellowish granules were seen in the pus.
culture of the specimen grows an organism that is a budding yeast at 610. Regarding this disease, which one of the following is MOST
37°C but produces hyphae at 25°C. accurate?
606. Of the following, which one is the MOST likely organism to (A) The causative organism is a gram-positive rod that forms
cause this infection? long filaments.
(A) Coccidioides immitis (B) The causative organism is a comma-shaped gram-negative
(B) Aspergillus fumigatus rod that produces an exotoxin which increases cyclic AMP.
(C) Histoplasma capsulatum (C) The causative organism cannot be seen in the Gram stain but
(D) Cryptococcus neoformans can be seen in an acid-fast stain.
(D) A combination of gram-negative cocci and spirochetes cause
CASE 12. Your patient is a 10-year-old boy who is receiving chemo- this disease.
therapy for acute leukemia. He develops fever, headache, and a stiff
neck, and you make a presumptive diagnosis of meningitis and do a CASE 16. Your patient is a 25-year-old man who is HIV antibody
lumbar puncture. A Gram stain reveals a small gram-positive rod, positive and has a CD4 count of 120 cells (normal, 1000–1500). He has
and culture of the spinal fluid grows a β-hemolytic colony on blood had a mild headache for the past week and vomited once yesterday. On
agar. physical examination, he has a temperature of 38°C and mild nuchal
rigidity but no papilledema. The rest of the physical examination is
negative.

mebooksfree.com
746 PART XIII USMLE (National Board) Practice Questions

611. Of the following, which one is the MOST likely to be found CASE 19. Your patient is a 10-year-old girl who has leukemia and
on examination of the spinal fluid? is receiving chemotherapy through an indwelling venous catheter.
(A) Lymphs and gram-positive cocci resembling Streptococ- She now has a fever of 39°C but is otherwise asymptomatic. You
cus pneumoniae do a blood culture, and the laboratory reports growth of
(B) Lymphs and budding yeasts resembling Cryptococcus Staphylococcus epidermidis.
neoformans 616. Which one of the following results is LEAST likely to be
(C) Polys and anaerobic gram-negative rods resembling found by the clinical laboratory?
Bacteroides fragilis (A) Gram-positive cocci in clusters were seen on Gram
(D) Polys and septate hyphae resembling Aspergillus stain of the blood culture.
fumigatus (B) Subculture of the blood culture onto blood agar
CASE 17. Your patient is a 25-year-old woman with a sore throat revealed nonhemolytic colonies.
since yesterday. On physical examination, her throat is red, but no (C) A coagulase test on the colonies was negative.
exudate is seen. Two enlarged, tender cervical lymph nodes are (D) A catalase test on the colonies was negative.
palpable. Her temperature is 101°F. A throat culture reveals no CASE 20. Your patient is a 25-year-old woman with several pur-
β-hemolytic colonies. After receiving this result, you do another puric lesions indicative of bleeding into the skin. Her vital signs
physical examination, which reveals an enlarged spleen. A hetero- are as follows: temperature, 38°C; blood pressure, 70/40; pulse,
phil antibody test finds that sheep red blood cells are agglutinated 140; respiratory rate, 24. You think she has septic shock and do a
by the patient’s serum. blood culture.
612. Which one of the following is the MOST likely cause of 617. Which one of the following organisms is LEAST likely to
this disease? be the cause of her septic shock?
(A) Streptococcus pyogenes (A) Corynebacterium diphtheriae
(B) Corynebacterium diphtheriae (B) Neisseria meningitidis
(C) Epstein–Barr virus (C) Clostridium perfringens
(D) Influenza virus (D) Escherichia coli
CASE 18. Your patient is a 15-year-old boy with migratory poly- 618. Of the following mechanisms, which one is LEAST likely
arthritis, fever, and a new, loud cardiac murmur. You make a to be involved with the pathogenesis of her septic shock?
clinical diagnosis of rheumatic fever. (A) Increased amount of interleukin-1
613. Which one of the following laboratory results is MOST (B) Activation of the alternate pathway of complement
likely to be found in this patient? (C) Increased amount of tumor necrosis factor
(D) Increased amount of antigen–antibody complexes
(A) A blood culture is positive for Streptococcus pyogenes at
this time. CASE 21. Your patient is a 55-year-old man with severe cellulitis
(B) A throat culture is positive for Streptococcus pyogenes at of the right leg, high fever, and a teeth-chattering chill. He is a
this time. fisherman who was working on his boat in the waters off the Texas
(C) A Gram stain of the joint fluid shows gram-positive coast yesterday.
cocci in chains at this time.
619. Which one of the following organisms is MOST likely to
(D) An anti-streptolysin O assay is positive at this time.
be the cause of his disease?
614. Which one of the following modes of pathogenesis is
(A) Yersinia pestis
MOST compatible with a diagnosis of rheumatic fever?
(B) Vibrio vulnificus
(A) Bacteria attach to joint and heart tissue via pili, invade, (C) Pasteurella multocida
and cause inflammation. (D) Brucella melitensis
(B) Bacteria secrete exotoxins that circulate via the blood to
the joints and heart. CASE 22. Your patient is a 30-year-old woman with facial nerve
(C) Bacterial antigens induce antibodies that cross-react paralysis. She also has fever and headache but does not have a stiff
with joint and heart tissue. neck. On physical examination, she has a circular, erythematous,
(D) Bacterial endotoxin induces interleukin-1 and tumor macular rash on the back of her thigh. You suspect that she has
necrosis factor, which cause inflammation in joint and Lyme disease.
heart tissue.
620. Of the following, which one is the MOST appropriate test
615. Which one of the following approaches is MOST likely to to order to confirm a diagnosis of Lyme disease?
prevent endocarditis in patients with rheumatic fever?
(A) Blood culture to grow the organism
(A) They should take the streptococcal polysaccharide (B) Stain for inclusion bodies within cells involved in the
vaccine. rash
(B) They should take penicillin if they have dental surgery. (C) Test for serum antibody against the organism
(C) They should take the toxoid vaccine every 5 years. (D) Dark field microscopy
(D) They should take rifampin if they have abdominal
surgery.

mebooksfree.com
PART XIII USMLE (National Board) Practice Questions 747

CASE 23. Your patient is a 60-year-old man with confusion for 2 CASE 28. Your patient is a 35-year-old man who is HIV antibody
months. He has no history of fever or stiff neck. On physical positive and has a CD4 count of 85 cells. He recently had a seizure,
examination, he was ataxic and his coordination was abnormal. A and a magnetic resonance imaging (MRI) scan indicates a lesion
diagnosis of tertiary syphilis was made by the laboratory. in the temporal lobe. A brain biopsy specimen reveals multinucle-
621. Of the following tests, which one is the MOST appropriate ated giant cells with intranuclear inclusions.
to make a diagnosis of tertiary syphilis? 626. Of the following, which one is the MOST likely cause?
(A) Spinal fluid culture to grow the organism (A) Herpes simplex virus type 1
(B) Stain for inclusion bodies in the lymphocytes in the (B) Parvovirus B19
spinal fluid (C) Coxsackie virus
(C) Test for antibody in the spinal fluid that reacts with (D) Western equine encephalitis virus
cardiolipin
(D) ELISA for the antigen in the spinal fluid CASE 29. Your patient is a 40-year-old woman with a severe attack
of diarrhea that began on the airplane while she was returning from
CASE 24. Your patient is a 65-year-old man who had an adeno- a vacation in the Middle East. She had had multiple episodes of
carcinoma of the pancreas that was surgically removed. Several watery, nonbloody diarrhea and little vomiting. She is afebrile. A
blood transfusions were given, and he did well until 2 weeks later, stool culture reveals only lactose-fermenting colonies on EMB agar.
when fever, vomiting, and diarrhea began. Blood and stool cul- 627. Of the following, which one is the MOST likely cause?
tures were negative, and the tests for Clostridium difficile and
(A) Shigella sonnei
hepatitis B surface antigen were negative. A liver biopsy revealed
(B) Helicobacter pylori
intranuclear inclusion bodies.
(C) Escherichia coli
622. Of the following, which one is the MOST likely cause? (D) Pseudomonas aeruginosa
(A) Adenovirus
(B) Cytomegalovirus CASE 30. Your patient is a 20-year-old man with a sore throat for
(C) Hepatitis A virus the past 3 days. On physical examination, his temperature is 38°C,
(D) Rotavirus the pharynx is red, and several tender submaxillary nodes are
palpable.
CASE 25. Your patient is a 3-year-old girl with fever and pain in 628. Of the following, which one is the MOST likely organism
her right ear. On physical examination, the drum is found to be to cause this infection?
perforated, and a bloody exudate is seen. A Gram stain of the
(A) Streptococcus agalactiae (group B Streptococcus)
exudate reveals gram-positive diplococci.
(B) Streptococcus sanguis (a viridans group Streptococcus)
623. Of the following, which one is the MOST likely cause? (C) Parvovirus B19
(A) Streptococcus pyogenes (D) Epstein–Barr virus
(B) Staphylococcus aureus
(C) Corynebacterium diphtheriae You do a throat culture, and many small, translucent colonies that
(D) Streptococcus pneumoniae are β-hemolytic grow on blood agar. Gram stain of one of these
colonies reveals gram-positive cocci in chains.
CASE 26. Your patient is a 70-year-old man with a fever of 40°C 629. Of the following, which one is the MOST likely organism
and a very painful cellulitis of the right buttock. The skin appears to cause this infection?
necrotic, and there are several fluid-filled bullae. Crepitus can be
(A) Streptococcus pneumoniae
felt, indicating gas in the tissue. A Gram stain of the exudate
(B) Streptococcus pyogenes
reveals large gram-positive rods.
(C) Streptococcus agalactiae (group B Streptococcus)
624. Of the following, which one is the MOST likely cause? (D) Peptostreptococcus species
(A) Clostridium perfringens
(B) Bacillus anthracis CASE 31. Your patient is a 55-year-old woman with a lymphoma
(C) Corynebacterium diphtheriae who is receiving chemotherapy via intravenous catheter. She sud-
(D) Actinomyces israelii denly develops fever, shaking chills, and hypotension.
630. Of the following, which one is the LEAST likely organism
CASE 27. Your patient is a 45-year-old woman with a cadaveric to cause this infection?
renal transplant that is being rejected despite immunosuppressive
(A) Streptococcus pneumoniae
therapy. She is now in renal failure with a blood pH of 7.32. This
(B) Klebsiella pneumoniae
morning, she awoke with a pain near her right eye. On physical
(C) Mycoplasma pneumoniae
examination, her temperature is 38°C, and the skin near her eye is
(D) Proteus mirabilis
necrotic. A biopsy specimen of the lesion contains nonseptate
hyphae invading the blood vessels. 631. If a blood culture drawn from the patient described in case
31 grows a gram-negative rod, which one of the following
625. Of the following, which one is the MOST likely cause? is the LEAST likely organism to cause this infection?
(A) Histoplasma capsulatum (A) Bordetella pertussis
(B) Aspergillus fumigatus (B) Escherichia coli
(C) Cryptococcus neoformans (C) Pseudomonas aeruginosa
(D) Mucor species (D) Serratia marcescens

mebooksfree.com
748 PART XIII USMLE (National Board) Practice Questions

632. Of the following virulence factors, which one is the MOST 637. Of the following, which one is the MOST likely organism to
likely to cause the fever and hypotension seen in the patient cause this infection?
described in case 31? (A) Prevotella melaninogenica
(A) Pilus (B) Pseudomonas aeruginosa
(B) Capsule (C) Proteus mirabilis
(C) Lecithinase (D) Haemophilus influenzae
(D) Lipopolysaccharide
CASE 36. Your patient is a 32-year-old moving-van driver who lives in
CASE 32. Your patient is a 30-year-old woman who was part of a tour St. Louis. He arrived in San Francisco about 10 days ago after picking
group visiting a Central American country. The day before leaving, up furniture in Little Rock, Dallas, Albuquerque, and Phoenix. He now
several members of the group developed fever, abdominal cramps, and has a persistent cough and fever to 101°F, and he feels poorly. On
bloody diarrhea. physical examination, crackles are heard in the left lower lobe, and
633. Of the following, which one is the LEAST likely organism to chest X-ray reveals an infiltrate in that area.
cause this infection? 638. Of the following, which one is the LEAST accurate statement?
(A) Shigella dysenteriae (A) He probably has spherules containing endospores in his
(B) Salmonella enteritidis lung.
(C) Vibrio cholerae (B) If dissemination to the bone occurs, this indicates a failure of
(D) Campylobacter jejuni his cell-mediated immunity.
(C) He probably acquired this disease by inhaling arthrospores.
A stool culture reveals no lactose-negative colonies on the EMB agar. (D) The causative organism of this disease exists as a yeast in the
634. Which one of the following is the MOST likely organism to soil.
cause this infection?
CASE 37. Your patient is a 25-year-old man with an ulcerated lesion on
(A) Shigella dysenteriae
his penis that is not painful. You suspect that it may be a chancre.
(B) Salmonella enteritidis
(C) Vibrio cholerae 639. Which one of the following tests is the MOST appropriate to do
(D) Campylobacter jejuni with the material from the lesion?
(A) Dark field microscopy
CASE 33. Your patient is a 78-year-old man who had an episode of (B) Gram stain
acute urinary retention and had to be catheterized. He then underwent (C) Acid-fast stain
cystoscopy to determine the cause of the retention. Two days later, he (D) Culture on Thayer-Martin agar
developed fever and suprapubic pain. Urinalysis revealed 50 white
blood cells and 10 red blood cells per high-power field. Culture of the 640. Which one of the following tests is the MOST appropriate to do
urine revealed a thin film of bacterial growth over the entire blood agar with the patient’s blood?
plate, and the urease test was positive. (A) Culture on blood agar
635. Which one of the following is the MOST likely organism to (B) Assay for antibodies that react with cardiolipin
cause this infection? (C) Assay for neutralizing antibody in human cell culture
(D) Heterophil antibody test
(A) Escherichia coli
(B) Proteus mirabilis CASE 38. Your patient is a 6-year-old boy with papular and pustular
(C) Enterococcus faecalis skin lesions on his face. A serous, “honey-colored” fluid exudes from
(D) Moraxella catarrhalis the lesions. You suspect impetigo. A Gram stain of the pus reveals many
neutrophils and gram-positive cocci in chains.
CASE 34. Your patient is a 40-year-old man with a depigmented lesion
on his chest that appeared about a month ago. The skin of the lesion is 641. If you cultured the pus on blood agar, which one of the follow-
thickened and has lost sensation. He has lived most of his life in rural ing would you be MOST likely to see?
Louisiana. (A) Small β-hemolytic colonies containing bacteria that are
636. Of the following tests, which one is the MOST appropriate to do bacitracin-sensitive
to reveal the cause of this disease? (B) Small α-hemolytic colonies containing bacteria that are
resistant to optochin
(A) Perform a biopsy of the lesion and do an acid-fast stain
(C) Large nonhemolytic colonies containing bacteria that are
(B) Culture on Sabouraud’s agar and look for germ tubes
oxidase-positive
(C) Culture on blood agar anaerobically and do a Gram stain
(D) Small nonhemolytic colonies containing bacteria that grow
(D) Obtain serum for a Weil-Felix agglutination test
in 6.5% NaCl
CASE 35. Your patient is a 28-year-old man with third-degree burns
CASE 39. Your patient is a 66-year-old woman being treated with che-
over a large area of his back and left leg. This morning, he spiked a fever
motherapy for lymphoma. She develops fever to 38°C and a nonpro-
to 40°C and had two teeth-chattering chills. A blood culture grows a
ductive cough. A chest X-ray reveals an infiltrate. You treat her
gram-negative rod that is oxidase-positive and produces a blue-green
empirically with an appropriate antibiotic. The following day, several
pigment.
vesicles appear on her chest.

mebooksfree.com
PART XIII USMLE (National Board) Practice Questions 749

642. Which one of the following viruses is the MOST likely cause of 646. Of the following, which one is the MOST likely organism to
her disease? cause this infection?
(A) Measles virus (A) Helicobacter pylori
(B) Respiratory syncytial virus (B) Bacteroides fragilis
(C) Varicella-zoster virus (C) Salmonella typhi
(D) Rubella virus (D) Vibrio parahaemolyticus

CASE 40. Your patient is a 40-year-old woman with systemic lupus CASE 44. Your patient is an 18-year-old woman with a swollen left
erythematosus who is being treated with high-dose prednisone during ankle. Two days ago, when the ankle began to swell, she thought she
a flare of her disease. She develops a fever to 38°C and a cough produc- had twisted it playing soccer. However, today she has a fever to 38°C,
tive of a small amount of greenish sputum. On physical examination, and the ankle has become noticeably more swollen, warm, and red. Her
you hear coarse breath sounds in the left lower lobe. Chest X-ray other joints are asymptomatic. You aspirate fluid from the joint.
reveals an infiltrate in that region. Gram stain of the sputum reveals 647. Using the joint fluid, which one of the following procedures is
long filaments of gram-positive rods. MOST likely to provide diagnostic information?
643. Which one of the following organisms is the MOST likely cause (A) Acid-fast stain and culture on Löwenstein-Jensen medium
of this disease? (B) Gram stain and culture on chocolate agar
(A) Mycobacterium kansasii (C) Dark field microscopy and the VDRL test
(B) Listeria monocytogenes (D) India ink stain and culture on Sabouraud’s agar
(C) Nocardia asteroides
(D) Mycoplasma pneumoniae CASE 45. Your patient is a 6-year-old boy with a history of several
episodes of pneumonia. A sweat test revealed an increased amount of
CASE 41. Your patient is a 10-year-old girl with acute leukemia who chloride, indicating that he has cystic fibrosis. He now has a fever and
responded well to her first round of chemotherapy but not to the most is coughing up a thick, greenish sputum. A Gram stain of the sputum
recent one. In view of this, she had a bone marrow transplant and is on reveals gram-negative rods.
an immunosuppressive regimen. She is markedly granulocytopenic. 648. Of the following, which one is the MOST likely organism to
Ten days after the transplant, she spikes a fever and coughs up bloody, cause this infection?
purulent sputum. Chest X-ray shows pneumonia. A wet mount of the
(A) Pseudomonas aeruginosa
sputum shows septate hyphae with dichotomous (Y-shape) branching.
(B) Haemophilus influenzae
644. Which one of the following organisms is the MOST likely cause (C) Legionella pneumophila
of this disease? (D) Bordetella pertussis
(A) Histoplasma capsulatum
(B) Aspergillus fumigatus CASE 46. Your patient is a 7-year-old boy with fever, two episodes of
(C) Rhizopus nigricans vomiting, and a severe headache that began this morning. He has no
(D) Candida albicans diarrhea. On physical examination, his temperature is 39°C, and
nuchal rigidity is found. Examination of the spinal fluid revealed a
CASE 42. Your patient is a 30-year-old man with acute onset of fever white cell count of 800, of which 90% were lymphs, and a normal con-
to 40°C and a swollen, very tender right femoral node. His blood pres- centration of both protein and glucose. A Gram stain of the spinal fluid
sure is 90/50, and his pulse is 110. As you examine him, he has a teeth- revealed no bacteria.
chattering shaking chill. He returned from a camping trip in the 649. Of the following, which one is the MOST likely to cause this
Southern California desert 2 days ago. infection?
645. Regarding this disease, which one of the following is MOST (A) Chlamydia trachomatis
accurate? (B) Mycobacterium avium-intracellulare
(A) An aspirate of the node will reveal a small gram-negative rod (C) Coxsackie virus
with bipolar staining (appears like a “safety pin”). (D) Adenovirus
(B) The organism was probably acquired by eating food con-
taminated with rodent excrement. CASE 47. Your patient is a 22-year-old man who has been on a low-
(C) The aspirate of the node should be cultured on Löwenstein- budget trip to India, where he ate many of the local foods. He has had
Jensen agar and an acid-fast stain performed. a low-grade fever, anorexia, and mild abdominal pain for about a
(D) The organism causes disease primarily in people with month. You suspect that he may have typhoid fever.
impaired cell-mediated immunity. 650. If he does have typhoid fever, which one of the following is the
LEAST likely laboratory finding?
CASE 43. Your patient is a 62-year-old woman with a history of carci-
(A) Culture of the blood reveals gram-negative rods.
noma of the sigmoid colon that was removed 5 days ago. The surgery
(B) Culture of the stool grows lactose-negative colonies in EMB
was complicated by the escape of bowel contents into the peritoneal
agar.
cavity. She now has fever and pain in the perineum and left buttock. On
(C) His serum contains antibodies that agglutinate Salmonella
physical examination, her temperature is 39°C, and myonecrosis with a
typhi.
foul-smelling discharge is found. A Gram stain of the exudate reveals
(D) His serum contains antibodies that cause a positive Weil-
gram-negative rods.
Felix reaction.

mebooksfree.com
750 PART XIII USMLE (National Board) Practice Questions

CASE 48. Your patient is a 30-year-old man who is HIV antibody 653. Regarding the causative organism, what is the MOST
positive and has had two episodes of Pneumocystis pneumonia. He now likely appearance of a Gram stain of the exudate from the
complains of pain in his mouth and difficulty swallowing. On physical skin infection?
examination, you find several whitish plaques on his oropharyngeal (A) Gram-positive cocci in grapelike clusters
mucosa. (B) Gram-positive cocci in chains
651. Regarding the most likely causative organism, which one of the (C) Gram-positive diplococci
following statements is MOST accurate? (D) Gram-negative diplococci
(A) It is a filamentous gram-positive rod that is part of the nor- 654. What is the pathogenesis of the cloudy urine and facial
mal flora in the mouth. swelling?
(B) It is an anaerobic gram-negative rod that is part of the nor- (A) Toxin-mediated
mal flora in the colon. (B) Direct invasion by the bacteria
(C) It is a yeast that forms pseudohyphae when it invades tissue. (C) Immune complex-mediated
(D) It is a spirochete that grows only in cell culture. (D) Cell-mediated immunity (delayed hypersensitivity)

CASE 49. Your patient is a 20-year-old woman with a rash that began Answers (Questions 594–654)
this morning. She has been feeling feverish and anorexic for the past 594. (B) 607. (C) 620. (C) 633. (C) 646. (B)
few days. On physical examination, there is a papular rash bilaterally 595. (C) 608. (A) 621. (C) 634. (D) 647. (B)
over the chest, abdomen, and upper extremities including the hands. 596. (D) 609. (C) 622. (B) 635. (B) 648. (A)
There are no vesicles and no petechiae. Cervical and axillary lymph 597. (B) 610. (A) 623. (D) 636. (A) 649. (C)
nodes were palpable. Her temperature was 38°C. White blood count 598. (D) 611. (B) 624. (A) 637. (B) 650. (D)
was 9000 with a normal differential. 599. (A) 612. (C) 625. (D) 638. (D) 651. (C)
652. Of the following organisms, which one is the MOST likely cause 600. (D) 613. (D) 626. (A) 639. (A) 652. (D)
of her disease? 601. (D) 614. (C) 627. (C) 640. (B) 653. (B)
(A) Histoplasma capsulatum 602. (A) 615. (B) 628. (D) 641. (A) 654. (C)
(B) Coxiella burnetii 603. (C) 616. (D) 629. (B) 642. (C)
(C) Neisseria meningitidis 604. (A) 617. (A) 630. (C) 643. (C)
(D) Treponema pallidum 605. (B) 618. (D) 631. (A) 644. (B)
606. (C) 619. (B) 632. (D) 645. (A)
CASE 50. Your patient is a 10-year-old boy who fell, abraded the skin
of his thigh, and developed cellulitis (i.e., the skin was red, hot, and
tender). Several days later, the infection was treated with a topical anti-
biotic ointment, and the cellulitis gradually healed. However, 2 weeks
later, he told his mother that his urine was cloudy and reddish, and she
noted that his face was swollen. You suspect acute glomerulonephritis.

mebooksfree.com
PART XIV USMLE (NATIONAL BOARD)
PRACTICE EXAMINATION

This practice examination consists of two blocks, each a virus from a patient’s throat and find that its genome is
containing 40 microbiology and immunology questions. RNA. Furthermore, you find that the genome is the comple-
You should be able to complete each block in 50 minutes. ment of viral mRNA within the infected cell. Of the following,
The proportion of the questions devoted to bacteriology, which one is the MOST appropriate conclusion you could
draw?
virology, mycology, parasitology, and immunology is
approximately that of the USMLE. As in the USMLE, the (A) The virion contains a polymerase.
(B) The purified genome RNA is infectious.
questions are randomly assorted (i.e., they are not grouped
(C) The genome RNA is segmented.
according to subject matter).
(D) A single-stranded DNA is synthesized during replication.
All of the questions have between 4 and 10 answer (E) The genome RNA encodes a precursor polypeptide that
choices. Each question has a single “BEST” answer; there must be cleaved by a protease.
are no “EXCEPT” type questions. The answer choices are 3. A 25-year-old man has a history of four episodes of boils in
listed either in alphabetical order or in order of the length the last year. Boils are abscesses caused by Staphylococcus
of the answer. The answer key is located at the end of each aureus. Which one of the following is MOST likely to be
block. the underlying immunologic factor that predisposes him to
multiple episodes of boils?
(A) A deficient amount of the C8 component of complement
QUESTIONS in his plasma
(B) An inability of his macrophages to present antigen in
BLOCK ONE association with class I MHC proteins
(C) A failure to release granzymes from his cytotoxic T cells
Directions (Questions 1–40)—Select the ONE lettered (D) An insufficient amount of IgG in his plasma
answer that is BEST in each question. 4. You are reading an article that says that otitis media is com-
1. A 9-year-old girl was playing soccer when she began to monly caused by nonencapsulated strains of Haemophilus
limp. She has a pain in her leg and points to her upper thigh influenzae. You are surprised that nonencapsulated strains
when asked where it hurts. Her temperature is 101°F. X-ray can cause this disease. Which one of the following BEST
of the femur reveals that the periosteum is eroded. You explains why your surprise is justified?
order a blood culture. Which one of the following would be (A) Nonencapsulated strains would not have endotoxin.
the MOST likely blood culture findings? (B) Nonencapsulated strains cannot secrete exotoxin A.
(A) Gram-negative rods that grow on EMB agar, forming (C) Nonencapsulated strains should be easily phagocytized.
purple colonies and a green sheen (D) Nonencapsulated strains should be rapidly killed by
(B) Gram-positive cocci that grow on blood agar, causing a ultraviolet light.
clear zone of hemolysis, and are coagulase-positive (E) Nonencapsulated strains should be susceptible to killing
(C) Gram-positive rods that grow only anaerobically and by cytotoxic T cells.
form a double zone of hemolysis on blood agar 5. A 35-year-old man is HIV antibody positive and has a CD4
(D) Gram-negative diplococci that grow on chocolate agar, count of 50/mL (normal, 1000–1500). He has had a fever of
are oxidase-positive, and ferment maltose 101°F for a few weeks and “feels tired all the time.” He has
(E) Gram-positive cocci that grow on blood agar, causing no other symptoms, and findings on physical examination
a green zone of hemolysis, and are not inhibited by are normal. Complete blood cell count, urinalysis, and chest
optochin and bile X-ray are normal. Blood, stool, and urine cultures show no
2. Your summer research project is to study the viruses that growth. A bone marrow biopsy reveals granulomas, and a
cause upper respiratory tract infections. You have isolated culture grows an organism that is a budding yeast at 37°C

751

mebooksfree.com
752 PART XIV USMLE (National Board) Practice Examination

but produces hyphae at 25°C. Of the following, which one is As it passes through mucosal cells, it acquires a secretory
the MOST likely cause? piece that protects it from degradation by proteases.
(A) Aspergillus fumigatus (A) IgM
(B) Cryptococcus neoformans (B) IgG
(C) Mucor species (C) IgA
(D) Histoplasma capsulatum (D) IgD
(E) Coccidioides immitis (E) IgE
6. A 70-year-old woman has sustained third-degree burns 11. Mycobacterium tuberculosis (MTB) and Mycobacterium
over a significant area of her body. Despite appropriate burn avium-complex (MAC) are important causes of disease,
care in the hospital, she spiked a fever to 39°C, and the nurse especially in immunocompromised patients. (MAC is also
reports blue-green pus on the dressing covering the burned known as Mycobacterium avium-intracellulare.) Regarding
area. Gram stain of the pus reveals gram-negative rods, and MTB and MAC, which one of the following statements is
antibiotic sensitivity tests show resistance to most antibiotics. the MOST accurate?
Which one of the following organisms is MOST likely to (A) Cell-mediated immunity is the most important host
cause this disease? defense mechanism against MTB, whereas antibody-
(A) Nocardia asteroides mediated immunity is the most important host defense
(B) Vibrio vulnificus mechanism against MAC.
(C) Bacteroides fragilis (B) In the clinical laboratory, MAC can be distinguished from
(D) Haemophilus influenzae MTB by the fact that MAC forms colonies in 7 days,
(E) Pseudomonas aeruginosa whereas MTB does not.
7. A 20-year-old woman has had several episodes of high fever, (C) Multidrug-resistant strains of MAC are much less com-
shaking chills, and a severe headache. She has a hematocrit of mon than multidrug-resistant strains of MTB.
30%. She has recently returned from Africa, where she was a (D) MAC is found in the environment and is not transmitted
Peace Corps volunteer. Which one of the following is MOST from person to person, whereas MTB is found in humans
likely to be seen in the blood smear sample from this patient? and is transmitted from person to person.
(A) Acid-fast rods 12. In the laboratory, a virologist was studying the properties of
(B) Banana-shaped gametocytes mutant viruses. When she infected cells with mutant virus #1,
(C) Nonseptate hyphae no progeny viruses were produced. When she infected cells
(D) Spherules with mutant virus #2, no progeny viruses were produced. But
(E) Tachyzoites when she infected cells with both mutant virus #1 and mutant
8. Certain microorganisms, such as the protozoan Try- virus #2, progeny viruses of both virus #1 and virus #2 were
panosoma and the bacterium Neisseria gonorrhoeae, can produced. Which one of the following is the term that BEST
change their surface antigens quite frequently. This allows describes this phenomenon?
the organisms to evade our host defenses. Which one of (A) Phenotypic mixing
the following BEST explains how this frequent change in (B) Complementation
antigenicity occurs? (C) Reassortment
(A) It is due to the transposition of existing genes into an (D) Recombination
active expression site. 13. Your patient has been treated for endocarditis with penicillin G
(B) It is due to the acquisition of new fertility plasmids by for the past 2 weeks. She now has a fever and maculopapular
transduction. erythematous rash over her chest and abdomen. A urinalysis
(C) It is due to conjugation, during which the recipient shows significant protein in the urine. If the fever, rash, and
obtains new chromosomal genes. proteinuria are immunologic in origin, which one of the fol-
(D) It is due to new mutations that occur at “hot spots” in lowing is MOST likely to be involved?
the genome. (A) IgG and complement
9. A 60-year-old woman had an adenocarcinoma of the colon (B) IgE and histamine
that was surgically removed. Several blood transfusions (C) IL-2 and cytotoxic T cells
were given, and she did well until 3 weeks after surgery, (D) Gamma interferon and macrophages
when fever, vomiting, and diarrhea began. Blood and stool 14. Endotoxin is an important underlying cause of septic shock
cultures were negative for bacteria, and the tests for Clostrid- and death, especially in hospitalized patients. Regard-
ium difficile and hepatitis B surface antigen were negative. ing endotoxin, which one of the following is the MOST
A liver biopsy revealed intranuclear inclusion bodies. Which accurate?
one of the following is the MOST likely cause? (A) It acts by phosphorylating the G stimulating protein.
(A) Cytomegalovirus (B) It is a polypeptide with an A-B subunit configuration.
(B) Dengue virus (C) It induces the synthesis of tumor necrosis factor.
(C) Hepatitis A virus (D) It is found primarily in gram-positive rods.
(D) Rotavirus (E) It can be treated with formaldehyde to form an effective
(E) Yellow fever virus toxoid vaccine.
10. Which one of the immunoglobulins BEST fits the following 15. A 12-year-old girl had a seizure this morning and was
description: It is found in plasma as a dimer with a J chain. rushed to the hospital. On examination, her temperature

mebooksfree.com
PART XIV USMLE (National Board) Practice Examination 753

was 40°C, and she had no nuchal rigidity. Computed tomog- 20. Which one of the following is MOST likely to induce an
raphy (CT) scan revealed no abnormality. A spinal tap was IgM antibody response without the participation of helper
done, and the protein and glucose were normal. Gram stain T cells?
of the spinal fluid showed no organisms and no polys. She (A) Bacterial capsular polysaccharide
was treated with various antibiotics but became comatose (B) Toxic shock syndrome toxin
and died 2 days later. The routine blood culture and spinal (C) Penicillin–bovine serum albumin (BSA) complex
fluid culture grew no organism. On autopsy of the brain, (D) Tetanus toxoid
eosinophilic inclusion bodies were seen in the cytoplasm 21. A 25-year-old pregnant woman in the third trimester
of neurons. Of the following, which one is the MOST likely comes to the emergency room saying that about 12 hours
cause? ago she began to feel feverish and weak. On examination,
(A) Prions she has a temperature of 40°C but no other pertinent find-
(B) JC virus ings. A blood culture grows small gram-positive rods that
(C) Rabies virus cause β-hemolysis on a blood agar plate incubated in room
(D) Parvovirus B19 air. Which one of the following bacteria is the MOST likely
(E) Herpes simplex virus type 1 cause?
16. A 70-year-old woman presents with rapid onset of fever to (A) Clostridium perfringens
39°C and a cough productive of greenish sputum. She is not (B) Streptococcus pyogenes
hospitalized and not immunocompromised. A chest X-ray (C) Bacillus cereus
reveals a left lower lobe infiltrate. Of the following, which set (D) Listeria monocytogenes
of findings describes the MOST likely causative organism (E) Brucella abortus
found in the sputum culture? 22. Regarding the mode of action of antiviral drugs, which one
(A) Gram-positive diplococci that form an a-hemolytic of the following is MOST accurate?
colony (A) Amantadine inhibits influenza A virus by inhibiting the
(B) Gram-negative diplococci that form an oxidase-positive RNA polymerase carried by the virion.
colony (B) Foscarnet inhibits varicella-zoster virus by inhibiting
(C) Gram-positive rods that form a b-hemolytic colony the RNA polymerase carried by the virion.
(D) Gram-negative rods that form an oxidase-positive (C) Acyclovir action is greater in herpesvirus-infected cells
colony than in uninfected cells because herpesvirus-infected
(E) Gram-negative cocci that grow only anaerobically cells contain an enzyme that phosphorylates acyclovir
17. Regarding the function of chemokines in host defenses, very efficiently.
which one of the following is the MOST accurate? (D) Azidothymidine inhibits human immunodeficiency
(A) Chemokines bind to the T-cell receptor outside of the virus (HIV) by inhibiting viral mRNA synthesis more
antigen-binding site and activate many T cells. efficiently than cellular mRNA synthesis.
(B) Chemokines induce gene switching in B cells that (E) Indinavir blocks HIV replication by inhibiting the pro-
increases the amount of IgE synthesized, thereby predis- tease required for the envelope protein gp120 to bind to
posing to allergies. the CD8 protein on the surface of the T cell.
(C) Chemokines penetrate the membranes of target cells 23. Which one of the following diseases is MOST likely to be
during attack by cytotoxic T cells. caused by a delayed hypersensitivity reaction?
(D) Chemokines attract neutrophils to the site of bacterial (A) Serum sickness
infection, thereby playing a role in the inflammatory (B) Poststreptococcal glomerulonephritis
response. (C) Systemic lupus erythematosus
18. Which one of the following answer choices consists of (D) Hemolytic disease of the newborn
bacteria, BOTH of which produce exotoxins that act by (E) Contact dermatitis
ADP-ribosylation? 24. Members of the genus Mycobacterium stain better with
(A) Salmonella typhi and Vibrio cholerae the acid-fast stain than with the Gram stain. Which one of the
(B) Vibrio cholerae and Corynebacterium diphtheriae following is the BEST explanation for this finding?
(C) Salmonella typhi and Clostridium perfringens (A) They lack a cell wall; therefore, they cannot adsorb the
(D) Corynebacterium diphtheriae and Staphylococcus crystal violet.
aureus (B) They have a very thin cell wall that does not retain
(E) Clostridium perfringens and Streptococcus pyogenes the crystal violet.
19. Regarding hepatitis C virus (HCV) and hepatitis D virus (C) They have a thick polysaccharide capsule that prevents
(HDV), which one of the following is MOST accurate? entry of the iodine solution.
(A) HCV is transmitted by blood, but HDV is not. (D) They have a large amount of lipid in their cell wall that
(B) More than half of HCV infections result in a chronic prevents entry of the crystal violet.
carrier state. 25. A 50-year-old man with a cadaveric renal transplant is
(C) There is an effective vaccine against HCV but not rejecting the transplant despite immunosuppressive drugs.
against HDV. He is now in renal failure with a blood pH of 7.31. Yesterday,
(D) Both HCV and HDV are defective RNA viruses and he developed a pain near his left eye that has become pro-
require concurrent HBV infection to replicate. gressively more severe. On examination, his temperature is

mebooksfree.com
754 PART XIV USMLE (National Board) Practice Examination

37.5°C, and the skin near his eye is swollen and necrotic. (C) In the screening test for HIV infection, the enzyme-
Microscopic examination of a biopsy of the lesion reveals linked immunosorbent assay (ELISA) test detects the
non-septate hyphae with right-angle branching. Which one presence of antibody to the p24 protein of HIV.
of the following organisms is the MOST likely cause? (D) A major limitation to our ability to produce a vaccine
(A) Candida albicans against HIV is that there are many serologic types of the
(B) Coccidioides immitis viral p24 protein.
(C) Cryptococcus neoformans 30. Regarding haptens, which one of the following statements is
(D) Histoplasma capsulatum the MOST accurate?
(E) Mucor species (A) They are typically polypeptides that are resistant to
26. A 60-year-old woman had surgery for ovarian carcinoma proteolytic cleavage within the antigen-presenting
4 days ago and has an indwelling urinary catheter in place. cell.
She now spikes a fever to 39°C and has cloudy urine in the (B) They bind to class II MHC proteins but not to class I
collection bottle. Gram stain of the urine shows many polys MHC proteins.
and gram-positive cocci in chains. Which one of the follow- (C) They cannot induce antibodies unless they are bound to
ing would be the MOST likely finding in the urine culture? a carrier protein.
(A) α-Hemolytic colonies on the blood agar plate that are (D) They activate complement by binding to the Fc part of
optochin-sensitive the heavy chain of IgG.
(B) β-Hemolytic colonies on the blood agar plate that are 31. Your patient is a 20-year-old man with a urethral discharge.
bacitracin-sensitive Gram stain of the pus reveals many neutrophils but no bac-
(C) β-Hemolytic colonies on the blood agar plate that teria. Which one of the following organisms is the MOST
hydrolyze hippurate likely cause?
(D) Nonhemolytic colonies on the blood agar plate that (A) Treponema pallidum
grow in 6.5% sodium chloride (B) Haemophilus ducreyi
27. Your patient is a 40-year-old man with a history of confusion (C) Mycobacterium marinum
for the past 2 days and a grand mal seizure that occurred this (D) Candida albicans
morning. He is HIV antibody positive and has a CD4 count (E) Chlamydia trachomatis
of 100/mL. On examination, his temperature is 37.5°C, and 32. Regarding host defenses against viruses, which one of the
the findings of the remainder of the examination are within following is MOST accurate?
normal limits. Magnetic resonance imaging (MRI) reveals (A) IgA exerts its main antiviral effect by enhancing the
several “ring-enhancing” cavitary brain lesions. He has not cytopathic effect of natural killer cells—a process called
traveled outside of the United States, is employed as the antibody-dependent cellular cytotoxicity.
manager of a supermarket, is a strict vegetarian, and has (B) IgG plays a major role in neutralizing virus infectivity
several household pets, namely, a dog, a cat, a parrot, and a during the primary infection.
turtle. Which one of the following organisms is the MOST (C) Complexes of virus and IgE are the cause of the inflam-
likely cause? matory arthritis seen in several viral infections, such as
(A) Toxocara canis hepatitis B and rubella.
(B) Toxoplasma gondii (D) Alpha and beta interferons exert their antiviral action
(C) Taenia saginata by inducing a ribonuclease that degrades viral mRNA
(D) Trichinella spiralis and a protein kinase that inactivates protein
(E) Trypanosoma cruzi synthesis.
28. The emergence of antibiotic-resistant bacteria, especially in (E) Alpha and beta interferons exert their antiviral effect
enteric gram-negative rods, is an extremely important phe- against viruses with RNA genomes but not against
nomenon. The acquisition of resistance most commonly those with DNA genomes.
occurs by a process that involves a sex pilus and the sub- 33. Allergic rhinitis is characterized by sneezing, rhinorrhea,
sequent transfer of plasmids carrying one or more trans- nasal congestion, and itching of the eyes and nose. Persons
posons. Which one of the following is the name that BEST with allergic rhinitis have “X” that binds to high-affinity
describes this process? receptors on “Y.” On reexposure to antigen, the “Y” of patients
(A) Conjugation with allergic rhinitis degranulate, releasing “Z” and other
(B) Combination mediators. Which one of the following sets BEST describes
(C) Transformation X, Y, and Z?
(D) Transduction (A) X is IgE, Y is macrophages, and Z is tumor necrosis
(E) Translocation factor.
29. Regarding the diagnosis, treatment, and prevention of HIV, (B) X is IgE, Y is basophils, and Z is histamine.
which one of the following is the MOST accurate? (C) X is IgG, Y is eosinophils, and Z is histamine.
(D) X is IgG, Y is neutrophils, and Z is tumor necrosis
(A) The drug zidovudine (AZT) is a “chain terminating”
factor.
drug; that is, it inhibits the growing polypeptide chain
(E) X is IgA, Y is eosinophils, and Z is interleukin-5.
by causing misreading of the viral mRNA.
34. An outbreak of postsurgical wound infections caused by
(B) The drug lamivudine (3TC) acts by binding to the inte-
Staphylococcus aureus has occurred. The infection control
grase, which prevents integration of the viral DNA into
team was asked to determine whether the organism could
cellular DNA.

mebooksfree.com
PART XIV USMLE (National Board) Practice Examination 755

be carried by one of the operating room personnel. Using 39. Regarding parvovirus B19, which one of the following is the
your knowledge of normal flora, which one of the following MOST accurate?
body sites is the MOST likely location for this organism? (A) Parvovirus B19 has a double-stranded DNA genome
(A) Colon but requires a DNA polymerase in the virion because it
(B) Gingival crevice replicates in the cytoplasm.
(C) Nose (B) Parvovirus B19 is transmitted primarily by sexual
(D) Throat intercourse.
(E) Vagina (C) Parvovirus B19 causes severe anemia because it prefer-
35. A 35-year-old man who is HIV antibody positive and has entially infects erythrocyte precursors.
a CD4 count of 30 says, “I can’t remember the simplest (D) Patients infected by parvovirus B19 can be diagnosed in
things.” You are concerned about dementia. An MRI indi- the laboratory using the cold agglutinin test.
cates several widely scattered lesions in the brain. Over the (E) Patients with disseminated disease caused by parvovirus
next 4 months, he develops visual field defects, becomes B19 should be treated with acyclovir.
paralyzed, and dies. Autopsy reveals that many neurons of 40. Which one of the following laboratory tests would be the
the brain have lost myelin and contain intranuclear inclu- BEST in order to determine the number of CD4-positive
sions. Electron microscopy reveals the inclusions contain cells in a patient infected with HIV?
nonenveloped viruses. Which one of the following viruses (A) Agglutination
is the MOST likely cause? (B) Enzyme-linked immunosorbent assay (ELISA)
(A) Adenovirus (C) Flow cytometry
(B) Cytomegalovirus (D) Immunoelectrophoresis
(C) Herpes simplex virus (E) Ouchterlony gel assay
(D) JC virus
(E) Coxsackie virus
36. A 75-year-old man with substernal chest pain was found
to have angina pectoris caused by syphilitic aortitis that
affected his coronary arteries. Of the following, which one ANSWERS TO BLOCK ONE
is the MOST likely way that the diagnosis of syphilis was 1. (B) 9. (A) 17. (D) 25. (E) 33. (B)
made? 2. (A) 10. (C) 18. (B) 26. (D) 34. (C)
(A) Blood culture 3. (D) 11. (D) 19. (B) 27. (B) 35. (D)
(B) Culture on Thayer-Martin medium (chocolate agar 4. (C) 12. (B) 20. (A) 28. (A) 36. (C)
with antibiotics) 5. (D) 13. (A) 21. (D) 29. (C) 37. (B)
(C) Detecting antibodies to cardiolipin in his blood 6. (E) 14. (C) 22. (C) 30. (C) 38. (B)
(D) Detecting treponemal antigen in his blood 7. (B) 15. (C) 23. (E) 31. (E) 39. (C)
(E) Western blot assay 8. (A) 16. (A) 24. (D) 32. (D) 40. (C)
37. A 22-year-old woman has an erythematous rash on the malar
eminences of her face that gets worse when she is out in the
sun. She has lost about 10 lb and feels tired much of the time.
She took her temperature a few times, and it was 99°F. Physical
examination was normal except for the rash. Laboratory tests BLOCK TWO
revealed a hemoglobin of 11 and a white blood cell count
1. A 4-year-old girl has papular and pustular lesions on her
of 5500. Urinalysis showed albumin in the urine but no red
face. The lesions are exuding a honey-colored serous fluid.
cells, white cells, or bacteria. Which one of the following is
You make a clinical diagnosis of impetigo. A Gram stain
the MOST likely laboratory finding in this disease?
of the exudate reveals gram-positive cocci in chains, and a
(A) Decreased number of helper (CD4-positive) T cells
culture reveals β-hemolytic colonies on blood agar. For
(B) High level of antibodies to double-stranded DNA
which one of the following sequelae is she MOST at risk?
(C) Increased number of cytotoxic (CD8-positive) T cells
(D) Low level of C1 inhibitor (A) Bloody diarrhea
(E) Low microbicidal activity of neutrophils (B) Blurred vision
38. Regarding antimicrobial drugs that act by inhibiting nucleic (C) Paralysis of the facial nerve (Bell’s palsy)
acid synthesis in bacteria, which one of the following is the (D) Red blood cells and albumin in her urine
MOST accurate? (E) Rusty-colored sputum
(A) Quinolones, such as ciprofloxacin, inhibit the RNA 2. The purified genome of certain RNA viruses can enter a
polymerase in bacteria by acting as nucleic acid cell and elicit the production of progeny viruses (i.e., the
analogues. genome is infectious). Regarding these viruses, which one
(B) Rifampin inhibits the RNA polymerase in bacteria by of the following statements is MOST accurate?
binding to the enzyme and inhibiting messenger RNA (A) They have a segmented genome.
synthesis. (B) They have a polymerase in the virion.
(C) Sulfonamides inhibit the DNA polymerase in bacteria (C) Their genome RNA is double-stranded.
by causing chain termination of the elongating strand. (D) They encode a protease that cleaves a precursor
(D) Trimethoprim inhibits the DNA polymerase in bacteria polypeptide.
by preventing the unwinding of double-stranded DNA. (E) Their genome RNA has the same base sequence as mRNA.

mebooksfree.com
756 PART XIV USMLE (National Board) Practice Examination

3. A 77-year-old man with enterococcal endocarditis 7. The mother of a 4-year-old child notes that her child is
needed to be treated with penicillin G but had a history sleeping poorly and scratching his anal area. You suspect
of a severe penicillin reaction. He was therefore skin the child may have pinworms. Which one of the following
tested using penicilloyl-polylysine as the antigen. Which is the BEST method to make that diagnosis?
one of the following is MOST likely to occur in a positive (A) Examine the stool for the presence of cysts
skin test? (B) Examine the stool for the presence of trophozoites
(A) The antigen forms immune complexes with IgG. (C) Examine a blood smear for the presence of
(B) The antigen activates CD4-positive T cells and microfilaria
macrophages. (D) Determine the titer of IgE antibody against the
(C) The antigen activates the alternative pathway of organism
complement. (E) Examine transparent adhesive tape for the presence of
(D) The antigen activates CD8-positive T cells by binding to eggs
class I MHC proteins. 8. Regarding bacterial spores, which one of the following is
(E) The antigen cross-links IgE on the mast cells and causes the MOST accurate?
the release of histamine. (A) One spore germinates to form one bacterium.
4. Regarding the Gram stain, which one of the following is the (B) They are produced primarily within human red blood
MOST accurate? cells.
(A) After adding crystal violet and Gram’s iodine, both (C) They are killed by boiling at sea level but not at high
gram-positive bacteria and gram-negative bacteria will altitude.
appear blue. (D) They are produced by anaerobes only in the presence of
(B) If you forget to stain with the red dye (safranin or basic oxygen.
fuchsin), both gram-positive bacteria and gram-negative (E) They contain endotoxin, which accounts for their
bacteria will appear blue. ability to cause disease.
(C) If you forget to heat-fix, both gram-positive bacteria 9. A 22-year-old woman had fever to 100°F and anorexia for
and gram-negative bacteria will appear blue. the past 2 days, and this morning she appears jaundiced. On
(D) One reason why bacteria have a different color in this examination, her liver is enlarged and tender. She has a total
stain is because the gram-positive bacteria have lipid bilirubin of 5 mg/dL (normal, <1) and elevated transami-
in their membrane, whereas gram-negative bacteria nases. She received the complete course of the hepatitis B
do not. vaccine 2 years ago but has not had the hepatitis A vaccine.
5. A 35-year-old man with a CD4 count of 50 presents with The results of her hepatitis serologies are as follows: HAV-
a skin nodule on his chest. The nodule is about 3 cm in IgM negative, HAV-IgG positive, HBsAg negative, HBsAb
diameter and is not red, hot, or tender. He says it has been positive, HBcAb negative, HCV-Ab positive. Of the
slowly growing bigger for the past 3 weeks. You biopsy the following, which one is the MOST accurate?
nodule, and the pathologist calls to say that the patient has (A) She probably has hepatitis A now, probably has not been
disseminated cryptococcosis. Which one of the following infected with hepatitis B virus (HBV), and probably had
is the BEST description of what the pathologist saw in the hepatitis C in the past.
biopsy specimen? (B) She probably has hepatitis A now, probably has been
(A) Spherules infected with HBV in the past, and probably had
(B) Non-septate hyphae hepatitis C in the past.
(C) Germ tubes (C) She has been infected with hepatitis A virus (HAV) in
(D) Budding yeasts with a thick capsule the past, probably has not been infected with HBV, and
(E) Septate hyphae with low-angle branching probably has hepatitis C now.
6. A 22-year-old woman complains of a persistent nonproduc- (D) She has been infected with HAV in the past, probably
tive cough and a fever of 101°F that came on slowly over has hepatitis B now, and probably had hepatitis C in the
the last 4 days. Physical examination reveals some rales in past.
the left lung base. A patchy infiltrate is seen on chest X-ray. 10. Regarding the function of the different classes of antibod-
She works as a secretary in a law office and has not traveled ies, which one of the following statements is the MOST
recently. She is not immunocompromised and has not been accurate?
hospitalized recently. A sample of her serum agglutinates (A) IgA acts as an antigen receptor on the surface of B
red blood cells at 4°C but not at 37°C. Which one of the cells.
following BEST describes the organism that is the MOST (B) IgG activates the alternative pathway of complement,
likely cause of her disease? resulting in the production of C3a that degrades the
(A) A very small bacterium that has no cell wall bacterial cell wall.
(B) A gram-negative diplococcus with a large capsule (C) IgG binds to the bacterial surface and makes the bacteria
(C) An acid-fast rod that forms colonies within 7 days more easily ingested by phagocytes.
(D) A filamentous gram-positive rod that is weakly (D) IgM defends against worm parasites, such as
acid-fast hookworms.
(E) A spirochete that has never been grown on blood agar (E) IgE blocks the binding of viruses to the gut mucosa.

mebooksfree.com
PART XIV USMLE (National Board) Practice Examination 757

11. A 6-year-old boy fell and sustained a deep wound from a lesion was performed. A Giemsa stain of the tissue shows
rusty nail that penetrated his thigh. His mother removed multinucleated giant cells with intranuclear inclusion bod-
the nail and cleaned the wound with soap and water. The ies. Which one of the following is the MOST likely caus-
next morning, he had a temperature of 102°F, and his ative organism?
thigh was very painful and swollen. In the emergency (A) Adenovirus
room, crepitus (gas in the tissue) was noted. A Gram stain (B) Coxsackie virus
of exudate from the wound area revealed large gram- (C) Cytomegalovirus
positive rods. Which one of the following is the MOST (D) Herpes simplex virus type 1
likely cause? (E) Influenza virus
(A) Actinomyces israelii (F) Measles virus
(B) Clostridium perfringens (G) Parvovirus B19
(C) Clostridium tetani (H) Poliovirus
(D) Listeria monocytogenes (I) Prion
(E) Mycobacterium fortuitum-chelonei complex (J) Rabies virus
(F) Nocardia asteroides 16. An 80-year-old man had a carcinoma of the colon removed
(G) Pseudomonas aeruginosa 3 days ago. He was doing well until this morning, when he
12. The two most common types of viral vaccines are killed vac- spiked a fever to 39°C and complained of severe abdominal
cines and live, attenuated vaccines. Regarding these vaccines, pain. Examination revealed a “board-like” abdomen indica-
which one of the following statements is the MOST accurate? tive of peritonitis. He was taken to the operating room, where
(A) Killed vaccines induce a longer-lasting response than it was discovered that his anastomosis had broken down and
do live, attenuated vaccines. bowel contents had spilled into the peritoneal cavity. A foul-
(B) Killed vaccines are no longer used in this country smelling exudate was observed. A Gram stain of the perito-
because they do not induce secretory IgA. neal exudate revealed many gram-negative rods. Which one
(C) Killed vaccines induce a broader range of immune of the following sets of bacteria is the MOST likely cause of
responses than do live, attenuated vaccines. this infection?
(D) Killed vaccines are safer to give to immunocompro- (A) Escherichia coli and Brucella melitensis
mised patients than are live, attenuated vaccines. (B) Enterobacter cloacae and Salmonella enteritidis
13. Regarding immediate (type I) and immune complex (type III) (C) Fusobacterium nucleatum and Bacteroides fragilis
hypersensitivities, which one of the following is the MOST (D) Haemophilus influenzae and Actinomyces israelii
accurate? (E) Shigella dysenteriae and Serratia marcescens
(A) IgE is involved in both immediate and immune com- 17. Regarding the primary and secondary antibody responses,
plex hypersensitivities. which one of the following statements is MOST accurate?
(B) Complement is involved in both immediate and (A) The IgM made in the primary response is made primar-
immune complex hypersensitivities. ily by memory B cells.
(C) Less antigen is typically needed to trigger an immediate (B) The lag phase is shorter in the primary response than in
reaction than an immune complex reaction. the secondary response.
(D) Neutrophils play a more important role in immediate (C) In the primary response, memory B cells are produced,
reactions than in immune complex reactions. but memory T cells are not.
14. Disease caused by which one of the following bacteria can (D) Antigen must be processed and presented in the pri-
be prevented by a toxoid vaccine? mary response but not in the secondary response.
(A) Actinomyces israelii (E) The amount of IgG made in the secondary response is
(B) Bacteroides fragilis greater than the amount made in the primary response.
(C) Borrelia burgdorferi 18. A 70-year-old man who is receiving chemotherapy for
(D) Corynebacterium diphtheriae leukemia develops a fever to 40°C and has two episodes
(E) Haemophilus influenzae of teeth-chattering chills, and his blood pressure drops to
(F) Listeria monocytogenes 80/20 mmHg. Of the following factors, which one is MOST
(G) Neisseria meningitidis likely to be the cause of his fever, chills, and hypotension?
(H) Salmonella typhi (A) Coagulase
(I) Streptococcus pneumoniae (B) Dipicolinic acid
(J) Yersinia pestis (C) Glycocalyx
15. A 50-year-old woman has had a gradual onset of headaches (D) Lipid A
that have become increasingly more severe during the past (E) Mycolic acid
3 weeks. On examination, she is confused regarding time, (F) Pili
place, and person, and she is febrile to 39°C. Her spinal (G) Polysaccharide capsule
fluid reveals a normal glucose, normal protein, and 17 cells, 19. A 22-year-old woman presents with “the worst sore throat
all of which were lymphocytes. Gram stain of the spinal I’ve ever had.” She also complains of fatigue and anorexia.
fluid shows no organism. An MRI reveals a 2-cm radio- She is not immunocompromised and has not been hospi-
lucent lesion in the temporal lobe. A biopsy of the brain talized recently. On examination, she is febrile to 38°C, the

mebooksfree.com
758 PART XIV USMLE (National Board) Practice Examination

pharynx is inflamed, and there are a few tender cervical is the MOST accurate regarding the pathogenesis of this
nodes bilaterally. There are no white lesions on the tongue disease?
or pharynx. A throat culture grows a-hemolytic colonies (A) It is caused by sensitized CD4-positive T lymphocytes
on blood agar that are optochin-resistant. Of the following, and macrophages invading the joints.
which one is the MOST likely cause? (B) It is caused by antibody against human IgG-forming
(A) Candida albicans immune complexes within the joints.
(B) Epstein–Barr virus (C) It is caused by the release of mediators from mast cells
(C) Parvovirus B19 when environmental agents cross-link adjacent IgEs
(D) Pneumocystis carinii within the joints.
(E) Poliovirus (D) It is caused by superantigens inducing the release of
(F) Serratia marcescens large amounts of lymphokines from helper T cells
(G) Streptococcus mutans within the joints.
(H) Streptococcus pneumoniae 24. Listed below are five bacteria paired with a mode of trans-
(I) Streptococcus pyogenes mission. Which one of the pairings is MOST accurate?
(J) Strongyloides stercoralis (A) Borrelia burgdorferi—mosquito bite
20. Regarding the complement pathway, which one of the follow- (B) Coxiella burnetii—bat guano
ing is MOST accurate? (C) Haemophilus influenzae—penetrating wound contami-
(A) C5a mediates chemotaxis and attracts neutrophils to nated with soil
the site of infection. (D) Rickettsia rickettsii—contaminated food
(B) C5b plays an important role in the opsonization of (E) Yersinia pestis—flea bite
gram-negative bacteria. 25. A 70-year-old man with leukemia initially responded to
(C) C3a is a decay-accelerating factor that causes the rapid chemotherapy but now is refractory. He therefore under-
decay and death of bacteria. went a bone marrow transplant and is now receiving large
(D) C1 binds to the surface of gram-positive bacteria, which doses of cyclosporine A and prednisone. Three weeks after
initiates the classic pathway. the transplant, he became febrile to 39°C and began cough-
(E) The membrane attack complex is produced in the classic ing up purulent sputum. A chest X-ray revealed pneumonia.
pathway but not in the alternative pathway. A Gram stain of the sputum did not reveal a predominant
21. A 65-year-old woman had symptoms of dementia. An MRI organism, but a KOH prep of the sputum revealed septate
revealed significant cortical atrophy. It was determined that hyphae with parallel walls and low-angle branching. Of the
her intraventricular pressure was very high, and a ventric- following organisms, which one is MOST likely to be the
uloperitoneal shunt (from the brain, tunneling under the cause of this pneumonia?
skin into the peritoneal cavity) was placed to relieve the (A) Aspergillus fumigatus
pressure. Three weeks later, she developed a fever to 38°C, (B) Candida albicans
malaise, and anorexia but no other symptoms. Of the fol- (C) Coccidioides immitis
lowing, which one BEST describes the MOST likely organ- (D) Cryptococcus neoformans
ism causing her current symptoms? (E) Rhizopus nigricans
(A) A gram-positive coccus that does not clot plasma 26. Your patient is a 20-year-old woman with severe diar-
(B) A curved gram-negative rod that produces urease rhea that began yesterday. She has just returned from a
(C) An acid-fast rod that does not grow on bacteriologic media 3-week trip to Peru, where she ate some raw shellfish at
(D) An obligate intracellular parasite that forms a cytoplasmic the farewell party. She now has watery diarrhea, perhaps
inclusion body 20 bowel movements a day, and is feeling quite weak and
(E) A spirochete that induces an antibody that agglutinates dizzy. Her stool is guaiac-negative, a test that determines
a lipid from a cow’s heart whether there is blood in the stool. A Gram stain of the
22. Two mutants of poliovirus, one mutated at gene X and stool reveals curved gram-negative rods. Of the following
the other mutated at gene Y, have been isolated. If a cell is organisms, which one is MOST likely to be the cause of
infected with each mutant alone, no virus is produced. If a her diarrhea?
cell is infected with both mutants, which one of the follow- (A) Bacteroides fragilis
ing is MOST likely to occur? (B) Campylobacter jejuni
(A) Complementation between the mutant gene products (C) Entamoeba histolytica
may occur, and, if so, both X and Y progeny viruses will (D) Helicobacter pylori
be made. (E) Shigella dysenteriae
(B) Phenotypic mixing may occur, and, if so, both X and Y (F) Vibrio cholerae
progeny viruses will be made. (G) Yersinia enterocolitica
(C) Reassortment of the genome segments may occur, and, 27. A 50-year-old man has had low-grade, persistent head-
if so, both X and Y progeny viruses will be made. aches for several months. In the last few days, nausea,
(D) The genome may be transcribed into DNA, and, if so, vomiting, and blurred vision have occurred. An MRI
both X and Y viruses will be made. reveals several cystlike lesions in the brain parenchyma.
23. A 40-year-old woman has a history of chronic inflamma- The patient lived for many years on one of the small
tion of the small joints of the hands bilaterally. You suspect Caribbean islands. On the basis of a positive serologic
rheumatoid arthritis. Which one of the following statements test, a diagnosis of neurocysticercosis was made. Of the

mebooksfree.com
PART XIV USMLE (National Board) Practice Examination 759

following, which one is the MOST likely mode by which following, which one is the MOST likely result observed in
this disease was acquired? the laboratory analysis of the spinal fluid?
(A) Sandfly bite (A) Gram-negative rods that grew only on Thayer-Martin
(B) Mosquito bite medium
(C) Sexual intercourse (B) A motile spirochete that formed beta-hemolytic colo-
(D) Ingestion of the larvae of the organism in raw fish nies on blood agar
(E) Ingestion of the eggs of the organism in contaminated food (C) Gram-positive cocci that formed alpha-hemolytic colo-
(F) Penetration of the skin by the organism while walking nies on blood agar
bare-footed (D) Gram-positive rods that grew only on chocolate agar
(G) Penetration of the skin by the organism while bathing supplemented with X and V factors
in fresh water (E) No organism was seen using Gram stain, but tissue
28. A 30-year-old woman with a previous history of rheumatic stains revealed cytoplasmic inclusion bodies
fever now has a fever for the past 2 weeks. Physical exami- 32. Regarding prions and prion-caused diseases, which one of
nation reveals a new heart murmur. You suspect endocar- the following is MOST accurate?
ditis and do a blood culture, which grows a viridans group (A) Prions are highly resistant to both ultraviolet light and
streptococcus later identified as Streptococcus sanguinus. to boiling but are inactivated by hypochlorite.
Of the following body sites, which one is the MOST likely (B) Prions are protein-containing particles surrounded by a
source of this organism? lipoprotein envelope with a DNA polymerase in the
(A) Colon envelope.
(B) Mouth (C) The diagnosis of prion-caused diseases such as
(C) Skin Creutzfeldt-Jakob disease is typically made by observ-
(D) Stomach ing cytopathic effect in cell culture.
(E) Vagina (D) Creutzfeldt-Jakob disease occurs primarily in children
29. Regarding poliovirus, which one of the following is MOST younger than the age of 2 years because they cannot
accurate? mount an adequate immune response to the prion
(A) Poliovirus remains latent within sensory ganglia, and protein.
reactivation occurs primarily in immunocompromised 33. A 2-year-old boy has had several infections of the sinuses
patients. and lungs and is being evaluated to determine whether he
(B) When the live, attenuated virus in the oral vaccine rep- has chronic granulomatous disease. Regarding this disease,
licates, revertant mutants can occur that can cause para- which one of the following is the MOST accurate?
lytic polio. (A) There is a deficiency in NADPH oxidase activity.
(C) The widespread use of the killed vaccine in the coun- (B) The defect is primarily in antigen-presenting cells such
tries of North and South America has led to the virtual as macrophages.
elimination of paralytic polio in those areas. (C) Pneumocystis jiroveci infections are common in patients
(D) The current recommendation is to give the live, attenu- with this disease.
ated vaccine for the first three immunizations to pre- (D) The diagnosis is primarily made by ELISA, in which
vent the child from acting as a reservoir, followed by antibody against the affected cell component is detected.
boosters using the killed vaccine. 34. Regarding Chlamydiae, which one of the following is MOST
30. Regarding ABO and Rh blood types, which one of the fol- accurate?
lowing is the MOST accurate? (A) They are gram-positive rods that do not form spores.
(A) People with type O are called universal recipients (B) They exhibit swarming motility on a blood agar plate.
because they have antibodies against H substance but (C) Their life cycle consists of a metabolically inactive par-
not against A and B antigens. ticle in the extracellular phase.
(B) If the father is Rh-positive and the mother is Rh-negative, (D) They can replicate only within cells because they lack
hemolytic disease of the newborn only occurs when the the ability to produce certain essential mRNAs.
child is Rh-negative. (E) They replicate in the nucleus of infected cells, where
(C) People who are Rh-negative usually have antibodies to they form inclusions that are useful diagnostically.
the Rh antigen because they are exposed to cross- 35. Regarding human papillomavirus (HPV), which one of the
reacting antigen located on bacteria in the colon. following is MOST accurate?
(D) If type A blood is transfused into a person with type B (A) Blood and blood products are an important mode of
blood, complement will be activated, and the mem- transmission of HPV.
brane attack complex will cause lysis of the type A red (B) HPV is an enveloped virus with a genome composed of
cells. double-stranded RNA.
31. A 25-year-old man was in a motorcycle accident 3 days ago, (C) Amantadine is a chain-terminating drug that inhibits
in which he sustained severe head trauma. He has had spi- HPV replication by blocking DNA synthesis.
nal fluid leaking from his nose since the accident and now (D) HPV induces the formation of koilocytes in the skin that
develops a severe headache. His temperature is 39°C, and are an important diagnostic feature of HPV infection.
on examination you find nuchal rigidity. You do a lum- (E) The P2 capsid protein of HPV activates the c-sarc onco-
bar puncture and find that the spinal fluid is cloudy and gene in human cells, which is the process by which
contains 5000 WBC/mL, 90% of which are polys. Of the HPV predisposes to malignancy.

mebooksfree.com
760 PART XIV USMLE (National Board) Practice Examination

36. Regarding Lyme disease, which one of the following is 39. Regarding human immunodeficiency virus (HIV), which
MOST accurate? one of the following is MOST accurate?
(A) The causative organism is a small gram-positive rod. (A) The term viral load refers to the concentration of HIV
(B) Mice are the main reservoir of the causative organism. RNA in the patient’s blood plasma.
(C) The Lyme disease vaccine contains toxoid as the (B) Both zidovudine and lamivudine block HIV replication
immunogen. by inhibiting cleavage of the precursor polypeptide by
(D) Fleas are the principal mode of transmission of the the virion-encoded protease.
causative organism. (C) The antigenicity of the GAG protein of HIV is highly
(E) The diagnosis in the clinical laboratory is typically variable, which is a significant impediment to the devel-
made by culturing the organism on chocolate agar. opment of a vaccine against HIV.
37. Regarding Bruton’s agammaglobulinemia, which one of the (D) The Western blot test for antibodies to HIV has more
following is the MOST accurate? false-positive results than the ELISA test.
(A) VDJ gene switching does not occur. 40. Regarding Th-1 and Th-2 cells, which one of the following
(B) There is very little IgG, but IgM and IgA levels are normal. is MOST accurate?
(C) The number of B cells is normal, but they cannot dif- (A) Th-1 cells produce gamma interferon and promote cell-
ferentiate into plasma cells. mediated immunity.
(D) There is a defect in a tyrosine kinase, one of the (B) Th-2 cells produce interleukin-12, which inhibits the
enzymes in the signal transduction pathway. formation of Th-1 cells.
(E) Viral infections are more common in patients with this (C) Both Th-1 and Th-2 cells have class II MHC proteins on
disease than are pyogenic bacterial infections. their outer cell membrane.
38. A 20-year-old woman presents with a history of vaginal dis- (D) Before they differentiate into Th-1 or Th-2 cells, naïve
charge for the past 3 days. On pelvic examination, you see a Th cells are double-positives (i.e., they produce both
mucopurulent exudate at the cervical os, and there is tender- gamma interferon and interleukin-4).
ness on palpation of the right fallopian tube. You do a Gram
stain and culture on the cervical discharge. The culture is done
on Thayer-Martin medium, which is a chocolate agar that con-
tains antibiotics that inhibit the growth of normal flora. Of the
following, which findings are the MOST likely to be found? ANSWERS TO BLOCK TWO
(A) A Gram stain reveals many neutrophils and spirochetes, 1. (D) 9. (C) 17. (E) 25. (A) 33. (A)
and culture on Thayer-Martin medium reveals no colonies. 2. (E) 10. (C) 18. (D) 26. (F) 34. (C)
(B) A Gram stain reveals many neutrophils and gram-variable 3. (E) 11. (B) 19. (B) 27. (E) 35. (D)
rods, and culture on Thayer-Martin medium reveals 4. (A) 12. (D) 20. (A) 28. (B) 36. (B)
β-hemolytic colonies. 5. (D) 13. (C) 21. (A) 29. (B) 37. (D)
(C) A Gram stain reveals many neutrophils and gram-negative 6. (A) 14. (D) 22. (A) 30. (D) 38. (C)
diplococci, and culture on Thayer-Martin medium reveals 7. (E) 15. (D) 23. (B) 31. (C) 39. (A)
oxidase-positive colonies. 8. (A) 16. (C) 24. (E) 32. (A) 40. (A)
(D) A Gram stain reveals many neutrophils but no gram-
negative diplococci are seen, and culture on Thayer-
Martin medium reveals coagulase-positive colonies.

mebooksfree.com

You might also like